You are on page 1of 232

Question

ICAP
Bank

Financial accounting
and reporting II
Fourth edition published by
Emile Woolf Limited
Bracknell Enterprise & Innovation Hub
Ocean House, 12th Floor, The Ring
Bracknell, Berkshire, RG12 1AX United Kingdom
Email: info@ewiglobal.com
www.emilewoolf.com

© Emile Woolf International, August 2017

All rights reserved. No part of this publication may be reproduced, stored in a retrieval system, or transmitted,
in any form or by any means, electronic, mechanical, photocopying, recording, scanning or otherwise, without
the prior permission in writing of Emile Woolf Publishing Limited, or as expressly permitted by law, or under
the terms agreed with the appropriate reprographics rights organisation.

You must not circulate this book in any other binding or cover and you must impose the same condition on
any acquirer.

Notice
Emile Woolf International has made every effort to ensure that at the time of writing the contents of this study
text are accurate, but neither Emile Woolf International nor its directors or employees shall be under any
liability whatsoever for any inaccurate or misleading information this work could contain.

© Emile Woolf International ii The Institute of Chartered Accountants of Pakistan


Certificate in Accounting and Finance
Financial accounting and reporting II

C
Contents
Page

Question and Answers Index v


Questions
Section A Questions 1
Answers
Section B Answers 85

© Emile Woolf International iii The Institute of Chartered Accountants of Pakistan


Financial accounting and reporting II

© Emile Woolf International iv The Institute of Chartered Accountants of Pakistan


Certificate in Accounting and Finance
Financial accounting and reporting II

I
Index to questions and answers
Question Answer
page page

CHAPTER 2 – IAS 1: PRESENTATION OF FINANCIAL STATEMENTS

2.1 LARRY 2 86

2.2 MINGORA IMPORTS LIMITED 3 87

2.3 BARRY 4 89

2.4 OSCAR INC 6 91

2.5 CLIFTON PHARMA LIMITED 7 93

2.6 SARHAD SUGAR LIMITED 8 95

2.7 BSZ LIMITED 10 98

2.8 YASIR INDUSTRIES LIMITED 11 101

2.9 SHAHEEN LIMITED 13 105

2.10 MOONLIGHT PAKISTAN LIMITED 14 107

2.11 FIGS PAKISTAN LIMITED 15 109

CHAPTER 3 – IAS 7: STATEMENTS OF CASH FLOWS

3.1 KLEA 18 113

3.2 STANDARD INC 19 115

3.3 FALLEN 20 118

3.4 BIN QASIM MOTORS LIMITED 22 120

3.5 ITTEHAD MANUFACTURING LTD 24 123

3.6 WASEEM INDUSTRIES LIMITED 26 125

3.7 JALIB INDUSTRIES LIMITED 28 128

© Emile Woolf International v The Institute of Chartered Accountants of Pakistan


Financial accounting and reporting II

Question Answer
page page

3.8 APOLLO INDUSTRY LIMITED 29 130

3.9 MARVEL ENGINEERING LIMITED 31 131

CHAPTER 4 – CONSOLIDATED ACCOUNTS: STATEMENTS OF FINANCIAL


POSITION – BASIC APPROACH

4.1 HALL 33 133

4.2 HASSLE 33 134

4.3 HYMN 34 135

4.4 HANG 34 136

4.5 HASH 35 138

CHAPTER 5 – CONSOLIDATED ACCOUNTS: STATEMENTS OF FINANCIAL


POSITION – COMPLICATIONS

5.1 HAIL 36 140

5.2 HAIRY 37 141

5.3 HARD 38 143

5.4 HALE 39 145

5.5 HELLO 39 146

5.6 HASAN LIMITED 40 148

CHAPTER 6 – CONSOLIDATED ACCOUNTS: STATEMENTS OF


COMPREHENSIVE INCOME

6.1 HARRY 42 152

6.2 HORNY 43 153

6.3 HERON 43 155

6.4 HANKS 45 156

CHAPTER 7 – IAS 16: PROPERTY, PLANT AND EQUIPMENT

7.1 ROONEY 47 160

7.2 EHTISHAM 47 161

7.3 CARLY 48 163

7.4 ADJUSTMENTS LIMITED 49 164

7.5 FAM 49 165

7.6 HUMAYUN CHEMICALS LIMITED 50 167

7.7 FARADAY PHARMACEUTICAL LIMITED 50 168

7.8 SCIENTIFIC PHARMA LIMITED 51 169

© Emile Woolf International vi The Institute of Chartered Accountants of Pakistan


Index to questions and answers

Question Answer
page page

CHAPTER 8 – IAS 36: IMPAIRMENT OF ASSETS

8.1 ABA LIMITED 52 171

8.2 HUSSAIN ASSOCIATES LTD 52 172

8.3 IMPS 53 173

CHAPTER 9 – IAS 38: INTANGIBLE ASSETS

9.1 FAZAL 55 175

9.2 HENRY 55 175

9.3 TOBY 55 176

9.4 BROOKLYN 56 177

9.5 ZOUQ INC 57 178

9.6 STAR-BRIGHT PHARMACEUTICAL LIMITED 57 179

9.7 RAISIN INTERNATIONAL 58 180

CHAPTER 10 – IFRS 16: LEASES

10.1 X LTD 59 181

10.2 PROGRESS LTD 59 181

10.3 MIRACLE TEXTILE LIMITED 59 183

10.4 ACACIA LTD 60 184

10.5 SHOAIB LEASING LIMITED 60 185

10.6 AKBAR LTD. 60 187

10.7 ALI LIMITED 61 187

10.8 MOAZZAM TEXTILE MILLS LIMITED 61 188

CHAPTER 11 – IAS 37: PROVISIONS CONTINGENT LIABILITIES AND


CONTINGENT ASSETS AND IAS 10: EVENTS AFTER THE REPORTING
PERIOD
11.1 BADAR 62 191

11.2 GEORGINA 62 191

11.3 EARLEY INC 62 192

11.4 ACCOUNTING TREATMENT 63 193

11.5 J-MART LIMITED 64 194

11.6 AKBER CHEMICALS LIMITED 64 195

11.7 QALLAT INDUSTRIES LIMITED 65 196

11.8 SKYLINE LIMITED 65 196

© Emile Woolf International vii The Institute of Chartered Accountants of Pakistan


Financial accounting and reporting II

Question Answer
page page

11.9 WALNUT LIMITED 66 197

11.10 ATTOCK TECHNOLOGIES LIMITED 66 197

CHAPTER 12 – IAS 8: ACCOUNTING POLICIES, CHANGES IN ACCOUNTING


ESTIMATES AND ERRORS

12.1 WONDER LIMITED 68 198

12.2 DUNCAN 68 199

12.3 MOHANI MANUFACTURING LIMITED 69 200

CHAPTER 13 – IAS 12: INCOME TAXES

13.1 FRANCESCA 70 201

13.2 SHEP (I) 70 201

13.3 SHEP (II) 71 202

13.4 SHEP (III) 71 203

13.5 SHEP (IV) 72 204

13.6 WAQAR LIMITED 73 205

13.7 SHAKIR INDUSTRIES 73 207

13.8 MARS LIMITED 74 208

13.9 BILAL ENGINEERING LIMITED 75 210

13.10 GALAXY INTERNATIONAL 75 212

13.11 APRICOT LIMITED 76 213

CHAPTER 14: IAS 33: EARNINGS PER SHARE

14.1 AIRCON LTD 77 214

CHAPTER 15 – IAS 23: BORROING COST

15.1 SPIN INDUSTRIES LIMITED 79 216

15.2 GRANITE CORPORATION 79 217

15.3 IMRAN LIMITED 80 218

15.4 QURESHI STEEL LIMITED 81 219

CHAPTER 16 – ETHICAL ISSUES IN FINANCIAL REPORTING

16.1 ETHICAL ISSUES 82 221

16.2 SINDH INDUSTRIES LTD 82 221

© Emile Woolf International viii The Institute of Chartered Accountants of Pakistan


Certificate in Accounting and Finance
Financial accounting and reporting II

SECTION
A
Questions

© Emile Woolf International 1 The Institute of Chartered Accountants of Pakistan


Financial accounting and reporting II

CHAPTER 1 – LEGAL BACKGROUND TO THE PREPARATION OF FINANCIAL


STATEMENTS
There are no questions specific to chapter one because the learning outcomes in this area concern the
preparation of financial statements. The relevant questions have been given in chapter 2 of this question
bank.

CHAPTER 2 – IAS 1: PRESENTATION OF FINANCIAL STATEMENTS

2.1 LARRY
The trial balance of Larry at 31 December 2015 is as follows.
Rupees in million
Dr Cr
Administration charges 342
Bank account 89
Cash 2
Payables’ ledger 86
Accumulated amortisation on patents at 31 December 2015 5
Accumulated depreciation at 31 December 2015 918
Receivables’ ledger 189
Distribution expenses 175
Property, plant and equipment at cost 2,830
Interest received 20
Issued share capital 400
Loan 18
Patents at cost 26
Accumulated profits 1,562
Purchases 2,542
Sales 3,304
Inventories at 31 December 2014 118
6,313 6,313

The following information is also relevant.


(1) Inventories on 31 December 2015 amounted to Rs. 127 million.
(2) Current tax of Rs. 75 million is to be provided.
(3) The loan is repayable by equal annual instalments over three years.

Required
Prepare an statement of profit or loss (analysing expenses by function) for the year ended 31
December 2015 and a statement of financial position as at that date.

© Emile Woolf International 2 The Institute of Chartered Accountants of Pakistan


Questions

2.2 MINGORA IMPORTS LIMITED


The trial balance of Mingora Imports Limited at 31 December 2015 is as follows.
Rupees in million
Dr Cr
Patent rights 60
Work-in-progress, 1 January 2015 125
Leasehold buildings at cost 300
Ordinary share capital 600
Sales 1,740
Staff costs 260
Accumulated depreciation on buildings, 1 January 2015 60
Inventories of finished games, 1 January 2015 155
Consultancy fees 44
Directors’ salaries 360
Computers at cost 50
Accumulated depreciation on computers, 1 January 2015 20
Dividends paid 125
Cash 440
Receivables 420
Trade payables 92
Sundry expenses 294
Accumulated profits, 1 January 2015 121

2,633 2,633

The following information is also relevant.


(1) Closing inventories of finished goods are valued at Rs. 180 million. Work-in-progress has
increased to Rs. 140 million.
(2) The patent rights relate to a computer program with a three year lifespan.
(3) On 1 January 2015 buildings were revalued to Rs. 360 million. This has not yet been reflected
in the accounts. Computers are depreciated over five years. Buildings are now to be
depreciated over 30 years.
(4) An allowance for bad debts (irrecoverable debts) of 5% is to be created.
(5) There is an estimated bill for current tax of Rs. 120 million which has not yet been recognised.

Required
Prepare an statement of profit or loss (analysing expenses by nature for the year ended 31
December 2015 and a statement of financial position as at that date.

© Emile Woolf International 3 The Institute of Chartered Accountants of Pakistan


Financial accounting and reporting II

2.3 BARRY
Barry has prepared the following draft financial statements for your review
Statement of profit or loss for year to 31st August 2015
Rs. in ‘000
Sales revenue 30,000
Raw materials consumed (9,500)
Manufacturing overheads (5,000)
Increase in inventories of work in progress and finished goods 1,400
Staff costs (4,700)
Distribution costs (900)
Depreciation (4,250)
Interest expense (350)
6,700

Statement of financial position as at 31st August 2015


Rs. in ‘000 Rs. in ‘000
Assets
Non-current
Freehold land and buildings 20,000
Plant and machinery 14,000
Fixtures and fittings 5,600
39,600
Current assets
Prepayments 200
Trade receivables 7,400
Cash at bank 700
Inventories 4,600
12,900
Total assets 52,500
Equity and liabilities
Equity shares of Rs. 1 each 21,000
Accumulated profit 14,000
Share premium 2,000
Total equity 37,000
Revaluation surplus 5,000
Current liabilities 5,300
Non-current liabilities
8% Debentures 2019 5,200
Total equity and liabilities 52,500

© Emile Woolf International 4 The Institute of Chartered Accountants of Pakistan


Questions

Additional information
1 Income tax of Rs. 2.1 million has yet to be provided for on profits for the current year. An
unpaid under-provision for the previous year’s liability of Rs. 400,000 has been identified on
5th September 2015 and has not been reflected in the draft accounts.
2 There have been no additions to, or disposals of, non-current assets in the year but the assets
under construction have been completed in the year at an additional cost of Rs. 50,000.
These related to plant and machinery.
The cost and accumulated depreciation of non-current assets as at 1st September 2014 were
as follows:
Cost Depreciation
Rs. in ‘000 Rs. in ‘000
Freehold land and buildings 19,000 3,000
(land element Rs. 10 million)
Plant and machinery 20,100 4,000
Fixtures and fittings 10,000 3,700
Assets under construction 400 -

3 There was a revaluation of land and buildings during the year, creating the revaluation surplus
of Rs. 5 million (land element Rs. 1 million). The effect on depreciation has been to increase
the buildings charge by Rs. 300,000. Barry adopts a policy of transferring the revaluation
surplus included in equity to retained earnings as it is realised.
4 Staff costs comprise 70% factory staff, 20% general office staff and 10% goods delivery staff
5 An analysis of depreciation charge shows the following:
Rs. in ‘000
Buildings (50% production, 50% administration) 1,000
Plant and machinery 2,550
Fixtures and fittings (30% production, 70% administration) 700

Required
Prepare the following information in a form suitable for publication for Barry’s financial statements for
the year ended 31st August 2015.
‰ Statement of profit or loss
‰ Statement of financial position
‰ Reconciliation of opening and closing property, plant and equipment

© Emile Woolf International 5 The Institute of Chartered Accountants of Pakistan


Financial accounting and reporting II

2.4 OSCAR INC


The following trial balance has been extracted from the books of accounts of Oscar Inc as at 31
March 2015.

Rs. in ‘000
Dr Cr
Administrative expenses 210
Share capital 600
Receivables 470
Bank overdraft 80
Income tax (overprovision in 2014) 25
Provision 180
Distribution costs 420
Non-current investments 560
Investment income 75
Plant and machinery
At cost 750
Accumulated depreciation (at 31 March 2015) 220
Retained earnings (at 1 April 2014) 180
Purchases 960
Inventory (at 1 April 2014) 140
Trade payables 260
Sales revenue 2,010
Interim dividend paid 120
3,630 3,630

Additional information
(1) Inventory at 31 March 2015 was valued at Rs. 150,000.
(2) The income tax charge based on the profits on ordinary activities is estimated to be Rs.
74,000.
(3) The provision is to be increased by Rs. 16,000.
(4) There were no purchases or disposals of fixed assets during the year.

Required
Prepare the company’s statement of profit or loss for the year to 31 March 2015 and a statement of
financial position as at that date in accordance with IAS 1.

© Emile Woolf International 6 The Institute of Chartered Accountants of Pakistan


Questions

2.5 CLIFTON PHARMA LIMITED


The following trial balance relates to Clifton Pharma Limited, a public listed company, at 30
September 2015.

Rs. in ‘000
Dr Cr
Cost of sales 134,000
Operating expenses 35,000
Loan interest paid (see note (1)) 1,500
Rental of vehicles (see note (2)) 8,600
Revenue 338,300
Investment income 2,000
Leasehold property at cost (see note (4)) 250,000
Plant and equipment at cost 197,000
Accumulated depreciation at 1 October 2014:
- leasehold property 40,000
- plant and equipment 47,000
Investments 92,400
Share capital 280,000
Share premium 20,000
Retained earnings at 1 October 2014 19,300
Loan notes (see note (1)) 50,000
Deferred tax balance at 1 October 2014 (see note (5)) 20,000
Inventory at 30 September 2015 23,700
Trade receivables 76,400
Trade payables 14,100
Bank 12,100
830,700 830,700

The following notes are relevant


(1) The effective interest rate on the loan notes is 6% per year.
(2) There are two separate contracts for rental of vehicles. A recent review by the finance
department of these contracts has reached the conclusion that Rs. 7 million of the total rental
cost of vehicles relates to a lease rather than rental arrangement.
The lease was entered into on 1 October 2014 which was when the Rs. 7 million was paid:
the lease agreement is for a four-year period in total, and there will be three more annual
payments in advance of Rs. 7 million, payable on 1 October in each year. The vehicles in the
lease agreement had a fair value of Rs. 24 million at 1 October 2014 and they should be
depreciated using the straight line method to a nil residual value. The interest rate implicit in
the lease is 10% per year. The other contract for vehicle rental is also a lease and the rental
payment should be charged to operating expenses. (Note: You are not required to calculate
the present value of the lease payments.)

© Emile Woolf International 7 The Institute of Chartered Accountants of Pakistan


Financial accounting and reporting II

(3) Other plant and equipment is depreciated at 20% per year by the reducing balance method.
All depreciation of property, plant and equipment should be charged to cost of sales.
(4) The leasehold property has a 25-year life and is amortised at a straight-line rate. On 30
September 2015 the leasehold property was re-valued to Rs. 220 million and the directors
wish to incorporate this re-valuation in the financial statements.
(5) The provision for income tax for the year ended 30 September 2015 has been estimated at
Rs. 18 million. At 30 September 2015 there are taxable temporary differences of Rs. 92
million. The rate of income tax on profits is 25%.

Required
(a) Prepare an statement of profit or loss for Clifton Pharma Limited for the year to 30 September
2015
(b) Prepare a statement of financial position (balance sheet) for Clifton Pharma Limited as at 30
September 2015

2.6 SARHAD SUGAR LIMITED


The following trial balance relates to Sarhad Sugar Limited at 30 September 2015:

Rs. in ‘000
Dr Cr
Leasehold property – at valuation 1 October 2014 (note (i)) 50,000
Plant and equipment – at cost (note (i)) 76,600
Plant and equipment – accumulated depreciation at 1 October 2014 24,600
Capitalised development expenditure – at 1 October 2014 (note (ii)) 20,000
Development expenditure – accumulated amortisation at 1 October 2014 6,000
Closing inventory at 30 September 2015 20,000
Trade receivables 43,100
Bank 1,300
Trade payables and provisions (note (iii)) 23,800
Revenue (note (i)) 300,000
Cost of sales 204,000
Distribution costs 14,500
Administrative expenses (note (iii)) 22,200
Interest on bank borrowings 1,000
Equity dividend paid 6,000
Research and development costs (note (ii)) 8,600
Share capital 70,000
Retained earnings at 1 October 2014 24,500
Deferred tax (note (v)) 5,800
Revaluation surplus (Leasehold property) 10,000
466,000 466,000

© Emile Woolf International 8 The Institute of Chartered Accountants of Pakistan


Questions

The following notes are relevant:


(i) Non-current assets – tangible:
The leasehold property had a remaining life of 20 years at 1 October 2014. The company’s
policy is to revalue its property at each year end and at 30 September 2015 it was valued at
Rs. 43 million.
On 1 October 2014 an item of plant was disposed of for Rs. 2·5 million cash. The proceeds
have been treated as sales revenue by Sarhad Sugar Limited. The plant is still included in the
above trial balance figures at its cost of Rs. 8 million and accumulated depreciation of Rs. 4
million (to the date of disposal).
All plant is depreciated at 20% per annum using the reducing balance method.
Depreciation and amortisation of all non-current assets is charged to cost of sales.
(ii) Non-current assets – intangible:
In addition to the capitalised development expenditure (of Rs. 20 million), further research and
development costs were incurred on a new project which commenced on 1 October 2014.
The research stage of the new project lasted until 31 December 2014 and incurred Rs. 1·4
million of costs. From that date the project incurred development costs of Rs. 800,000 per
month. On 1 April 2015 the directors became confident that the project would be successful
and yield a profit well in excess of its costs. The project is still in development at 30
September 2015.
Capitalised development expenditure is amortised at 20% per annum using the straight-line
method. All expensed research and development is charged to cost of sales.
(iii) Sarhad Sugar Limited is being sued by a customer for Rs. 2 million for breach of contract over
a cancelled order. Sarhad Sugar Limited has obtained legal opinion that there is a 20%
chance that Sarhad Sugar Limited will lose the case. Accordingly Sarhad Sugar Limited has
provided Rs. 400,000 (Rs. 2 million x 20%) included in administrative expenses in respect of
the claim. The unrecoverable legal costs of defending the action are estimated at Rs.
100,000. These have not been provided for as the legal action will not go to court until next
year.
(iv) The directors have estimated the provision for income tax for the year ended 30 September
2015 at Rs. 11·4 million. The required deferred tax provision at 30 September 2015 is Rs. 6
million.

Required
(a) Prepare the statement of profit or loss for the year ended 30 September 2015
(b) Prepare the statement of financial position as at 30 September 2015.
Note: notes to the financial statements are not required.

© Emile Woolf International 9 The Institute of Chartered Accountants of Pakistan


Financial accounting and reporting II

2.7 BSZ LIMITED


The post-closing trial balance of BSZ Limited, a listed company, as at June 30, 2017 is given
below:
Debit Credit
Rs. in million
Cash at banks – current accounts 7
Cash at banks – in saving accounts 22
Stocks in trade – closing 90
Accounts receivable 60
Provision for bad debts 3
Advances to suppliers 16
Advances to staff 6
Short term deposits 11
Prepayments 4
Sales tax receivable 12
Freehold land – at revalued amount 375
Furniture and fixtures - cost 27
Accumulated depreciation – Furniture and fixtures 8
Machines - cost 85
Accumulated depreciation – Machines 27
Building on freehold land – cost 150
Accumulated depreciation – Building 26
Computer software – cost 10
Accumulated amortization – Computer software 2
Deferred taxation 40
Short term loan 85
Accounts payable 75
Accrued liabilities 7
Provision for taxation 17
Issued, subscribed and paid up capital (Rs. 10 each) 400
Surplus on revaluation of fixed assets 120
Accumulated profits 65
875 875

Additional Information
(i) The first revaluation of freehold land was carried out in 2013 and resulted in a surplus of
Rs. 120 million. The valuation was carried out under market value basis by an independent
valuer, Mr. Dee, Chartered Civil Engineer of M/s SSS Consultants (Pvt.) Ltd., Islamabad.

© Emile Woolf International 10 The Institute of Chartered Accountants of Pakistan


Questions

(ii) The details relating to additions, disposal and depreciation/amortization of fixed assets,
during the year 2017 are given below:
‰ The company uses the straight line method for charging depreciation and amortization.
The building is depreciated at a rate of 5% whereas 10% is charged on machines,
furniture and fixtures and computer software.
‰ Construction on third floor of the building commenced on March 1, 2017 and is
expected to be completed on September 30, 2017. The cost incurred during the year
i.e. Rs. 20 million was capitalised on June 30, 2017.
‰ Furniture and fixtures worth Rs. 8 million were purchased on April 1, 2017.
‰ A machine was sold on February 28, 2017 to NJ Enterprise at a price of Rs. 13 million.
At the time of disposal, the cost and written down value of the machine was Rs. 15
million and Rs. 10 million respectively.
(iii) 50% of the accounts receivable were secured and considered good. 10% of the
unsecured accounts receivable were considered doubtful. Bad debts expenses for the year
amounted to Rs. 1.0 million. An amount of Rs. 1.4 million was written off during the year.
(iv) All advances given to suppliers are considered good and include an amount of Rs. 4.0
million paid for goods which will be supplied on December 31, 2018.
(v) Cash at banks in saving accounts carry interest / mark-up ranging from 3% to 7% per
annum.
(vi) The authorised share capital of the company is Rs. 500 million.

Required
Prepare the statement of financial position as at June 30, 2017 along with the relevant notes
showing all possible disclosures as required under the International Accounting Standards and
the Companies Act, 2017.
(Comparative figures and the note on accounting policies are not required.)

2.8 YASIR INDUSTRIES LIMITED


The following trial balance related to Yasir Industries Limited (YIL) for the year ended June 30, 2017:

Dr Cr
Rs. in million
Ordinary share capital (Rs. 10 each) - 120.00
Retained earnings - 10.20
Sales - 472.40
Purchases 175.70 -
Production labour 61.00
Manufacturing overheads 39.00
Inventories (July 1, 2016) 38.90
Administrative expenses 40.00 -
Distribution expenses 19.80 -
Financial charges 0.30 -

© Emile Woolf International 11 The Institute of Chartered Accountants of Pakistan


Financial accounting and reporting II

Dr Cr
Rs. in million
Cash and bank - 13.25
Trade creditors - 30.40
Accrued expenses - 16.20
10% redeemable preference shares - 40.00
Debentures - 80.00
Deferred tax (July 1, 2016) - 6.00
Suspense account 30.00 -
Leasehold property - at cost 230.00 -
Machines – at cost 168.60 -
Software – at cost 20.00 -
Acc. depreciation – Leasehold property (June 30, 2017) - 40.25
Acc. depreciation – Machines (June 30, 2017) - 48.60
Acc. amortization – Software (June 30, 2017) - 12.00
Trade receivables 66.00 -
889.30 889.30

Additional Information
(i) Sales include an amount of Rs. 27 million, made to a customer under sale or return
agreement. The sale has been made at cost plus 20% and the expiry date for the return of
these goods is July 31, 2017.
(ii) The value of inventories at June 30, 2017 was Rs. 42 million.
(iii) A fraud of Rs. 30 million was discovered in October 2016. A senior employee of the company
who left in June 2016, had embezzled the funds from YIL’s bank account. The chances of
recovery are remote. The amount is presently appearing in the suspense account.
(iv) On January 1, 2017 YIL issued debenture certificates which are repayable in 2022. Interest is
paid on these at 12% per annum.
(v) Financial charges comprise bank charges and bank commission.
(vi) The provision for current taxation for the year ended June 30, 2017 after making all the above
adjustments is estimated at Rs. 16.5 million.
(vii) The carrying value of YIL’s net assets as on June 30, 2017 exceeds their tax base by Rs. 30
million. The income tax rate applicable to the company is 30%.
(viii) On July 1, 2016, the leasehold property having a useful life of 40 years was revalued at Rs.
238 million. No adjustment in this regard has been made in the books.
(ix) Depreciation of leasehold property is charged using the straight line method. 50% of
depreciation is allocated to manufacturing, 30% to administration and 20% to selling and
distribution.
Required
In accordance with the requirements of the Companies Act, 2017 and International Accounting
Standards, prepare the:
(a) statement of financial position as of June 30, 2017.
(b) statement of profit or loss for the year ended June 30, 2017.
(Comparative figures and notes to the financial statements are not required.)

© Emile Woolf International 12 The Institute of Chartered Accountants of Pakistan


Questions

2.9 SHAHEEN LIMITED


Following is the trial balance of Shaheen Limited (SL) as at June 30, 2017:
Rs. in ‘000
Dr Cr
Sales revenue 200,000
Manufacturing costs 100,000
Selling and distribution costs 35,000
Administrative costs 30,000
Opening inventories 23,000
Interest on borrowings 5,000
Provision for income tax 2,000
Advance income tax paid 6,000
Property, plant and equipment 86,000
Accumulated depreciation on property, plant and equipment 12,000
Export licence 6,000
Trade receivables 37,800
Cash and bank balances 4,725
Other receivable and prepayments 14,000
Trade payables 12,000
Provisions for litigation 5,000
Long term borrowings 31,525
Deferred tax 5,000
Share capital (Rs. 10 each and fully paid) 60,000
Retained earnings 20,000
347,525 347,525

Additional information
(i) Sales last year (year ended 30 June 2016) included goods invoiced at Rs 10 million which
were sent to a customer on June 25, 2016 under a sale or return agreement, at cost plus
20%. The goods were returned on August 25, 2016. No correction has been made for the
return.
(ii) The export licence has been obtained for exporting a new product and is effective for five
years up to December 31, 2021. However, the exports commenced from July 1, 2017.
(iii) Closing inventories are valued at Rs. 30 million.
(iv) Details of property, plant and equipment are as follows:

Plant and
Land Buildings equipment
Rs in ‘000
Cost as at June 30, 2016 20,000 36,000 30,000
Fully depreciated amounts included in cost 3,000
Estimated useful life at the date of purchase 20 years 10 years

The company uses straight line method for charging depreciation. Depreciation is allocated
to manufacturing, distribution and administrative costs at 75%, 15% and 10% respectively.

© Emile Woolf International 13 The Institute of Chartered Accountants of Pakistan


Financial accounting and reporting II

(v) Rs. 6 million of the long term borrowings is of current maturity (i.e. will be repaid within 12
months).
(vi) During the year Rs. 5 million was paid in full and final settlement of income tax liability
against which a provision of Rs. 7.0 million had been made in the previous year. Current
year’s taxable income exceeds accounting income by Rs. 5 million of which 0.8 million are
permanent differences. Applicable tax rate for the company is 35%.
(vii) On July 30, 2017 the board of directors proposed a final dividend at 15% for the year ended
June 30, 2017 (2016: at 20%)

Required
In accordance with the requirements of the Companies Act, 2017 and International Financial
Reporting Standards, prepare:
(a) The statement of financial position as of June 30, 2017
(b) The statement of profit or loss for the year ended June 30, 2017
(c) The statement of changes in equity for the year ended June 30, 2017.
(Comparative figures and notes to the financial statements are not required)

2.10 MOONLIGHT PAKISTAN LIMITED


Following is the summarised trial balance of Moonlight Pakistan Limited (MPL), a listed company, for
the year ended December 31, 2017:

Rs. in million
Debit Credit
Land and buildings - at cost 2,600 -
Plants – at cost 2,104 -
Trade receivables 702 -
Stock in trade at December 31, 2017 758 -
Cash and bank 354 -
Cost of sales 1,784 -
Selling expenses 220 -
Administrative expenses 250 -
Financial charges 210 -
Accumulated depreciation as on January 1, 2017 – Buildings - 400
Accumulated depreciation as on January 1, 2017 – Plants - 670
Ordinary shares of Rs. 10 each fully paid - 1,200
Retained earnings as at January 1, 2017 - 510
12% Long term loan - 1,600
Provision for gratuity - 8
Deferred tax on January 1, 2017 - 22
Trade payables - 544
Right subscription received - 420
Revenue - 3,608
8,982 8,982

© Emile Woolf International 14 The Institute of Chartered Accountants of Pakistan


Questions

Additional Information
(i) The land and buildings were acquired on January 1, 2013. The cost of land was Rs. 600
million. On January 1, 2017 a professional valuation firm valued the buildings at Rs. 1,840
million with no change in the value of land. The estimated life at acquisition was 20 years
and the remaining life has not changed as a result of the valuation. 60% of depreciation on
buildings is allocated to manufacturing, 25% to selling and 15% to administration.
(ii) Plant is depreciated at 20% per annum using the reducing balance method.
(iii) On March 31, 2017 MPL made a bonus issue of one share for every six held. The issue
has not been recorded in the books of account.
(iv) Right shares were issued on September 1, 2017 at Rs. 12 per share.
(v) The interest on long term loan is payable on the first day of July and January. No accrual has
been made for the interest payable on January 1, 2015.
(vi) MPL operates an unfunded gratuity scheme for all its eligible employees. The provision
required as on December 31, 2017 is estimated at Rs. 23 million. Rs. 3 million were paid
during the year and debited to the provision for gratuity account. Cost of gratuity is allocated
to production, selling and administration expenses in the ratio of 60%: 20% : 20%.
(vii) The tax charge for the current year after making all related adjustments is estimated at Rs. 37
million. The timing differences related to taxation are estimated to increase by Rs. 80 million,
over the last year. The applicable income tax rate is 35%.

Required
In accordance with the requirements of Companies Act, 2017 and International Financial
Reporting Standards, prepare the following:
(a) Statement of Financial Position as of December 31, 2017.
(b) Statement of profit or loss for the year ended December 31, 2017.
(Comparative figures and notes to the financial statements are not required)

2.11 FIGS PAKISTAN LIMITED


Figs Pakistan Limited is a listed company engaged in the business of manufacturing and marketing
of personal care and food products. Following is an extract from its trial balance for the year ended
31 December 2017:

Debit Credit
Rs. in million
Sales - Manufactured goods 56,528
Sales - Imported goods 1,078
Scrap sales 16
Dividend income 12
Return on savings account 2
Sales tax - Imported goods 53
Sales tax - Manufactured goods 10,201
Sales discount 2,594
Raw material stock as on 1 January 2017 1,751
Work in process as on 1 January 2017 73
Finished goods (manufactured) as on 1 January 2017 1,210
Finished goods (imported) as on 1 January 2017 44

© Emile Woolf International 15 The Institute of Chartered Accountants of Pakistan


Financial accounting and reporting II

Debit Credit
Rs. in million
Purchases - Raw material 22,603
Purchases - Imported goods 658
Stores and spares consumed 180
Salaries, wages and benefits 2,367
Utilities 734
Depreciation and amortization 1,287
Stationery and office expenses 230
Repairs and maintenance 315
Advertisement and sales promotion 4,040
Outward freight and handling 1,279
Legal and professional charges 71
Auditor's remuneration 13
Donations 34
Workers Profit Participation Fund 257
Worker Welfare Fund 98
Loss on disposal of property, plant and equipment 10
Financial charges on short term borrowings 133
Exchange loss 22
Financial charges on lease 11
Additional information
(i) The position of inventories as at 31 December 2017 was as follows:

Rs. m
Raw material 2,125
Work in process 125
Finished goods (manufactured) 1,153
Finished goods (imported) 66

(ii) The basis of allocation of various expenses among cost of sales, distribution costs and
administrative expenses are as follows:

Cost of Distribution Administrative


sales costs expenses
% % %
Salaries, wages and benefits 55 30 15
Depreciation and amortization 70 20 10
Stationery and office expenses 25 40 35
Repairs and maintenance / Utilities 85 5 10

© Emile Woolf International 16 The Institute of Chartered Accountants of Pakistan


Questions

(iii) Salaries, wages and benefits include contributions to provident fund (defined contribution
plan) and gratuity fund (defined benefit plan) amounting to Rs. 54 million and Rs. 44 million
respectively.
(iv) Auditor’s remuneration includes taxation services and out-of-pocket expenses amounting to
Rs. 4 million and Rs. 1 million respectively.
(v) Donations include Rs. 5 million given to Dates Cancer Foundation (DCF). One of the
company’s directors, Mr. Peanut is a trustee of DCF.
(vi) The tax charge for the current year after making all related adjustments is estimated at Rs.
1,440 million. Taxable temporary differences of Rs. 3,120 originated in the year million, over
the last year. The applicable income tax rate is 35%.
(vii) 274 million ordinary shares were outstanding as on 31 December 2017.
(viii) There is no other comprehensive income for the year.

Required
Prepare the statement of profit or loss for the year ended 31 December 2017 along with the relevant
notes showing required disclosures as per the Companies Act, 2017 and International Financial
Reporting Standards. Comparatives are not required.

© Emile Woolf International 17 The Institute of Chartered Accountants of Pakistan


Financial accounting and reporting II

CHAPTER 3 – IAS 7: STATEMENTS OF CASH FLOWS

3.1 KLEA
The statement of financial position and statement of profit or loss for Klea for the year to 31st March
2015 are provided below.
Statement of financial position as at 31st March 2015
2015 2014
Rs. in ‘000
Assets
Non-current assets
Intangible assets 300 200
Property, plant and equipment 3,450 1,600
Financial assets 400 200
4,150 2,000
Current assets
Inventory 3,200 2,000
Trade receivables 2,400 2,000
Cash and cash equivalents 32 580
5,632 4,580
Total assets 9,782 6,580
Equity and liabilities
Equity
Issued share capital 3,000 2,000
Share premium account 838 560
Retained earnings 910 354
Total equity 4,748 2,914
Revaluation surplus 1,000 -

Non-current liabilities
Interest-bearing loans and liabilities 1,600 2,000
Current liabilities
Bank overdraft 414 -
Trade payables 1,600 1,266
Taxation 420 400
2,434 1,666
Total liabilities 4,034 3,666
Total equity and liabilities 9,782 6,580

Statement of profit or loss for the year ended 31st March 2015
Rs. in ‘000
Revenue 10,000
Other income 100
Change in inventory of finished goods and WIP 1,300
Raw materials and consumables used 4,000
Employee benefits costs 3,000
Depreciation and amortisation expense 800
Other expenses 1,724

© Emile Woolf International 18 The Institute of Chartered Accountants of Pakistan


Questions

Total expenses (9,524)


1,876
Finance costs (320)
Finance income 50
Profit before tax 1,606
Income tax expense (650)
Profit for the year 956

Additional information
(i) Non-current assets Rs. in ‘000
2015 2014
Cost Deprec’n Cost Deprec’n
Intangible assets 700 400 400 200
Property, plant and equipment 5,000 1,550 3,000 1,400
(ii) At 1 April 2014 land was revalued from Rs. 1million to Rs. 2 million.
(iii) During the year, plant and machinery costing Rs. 600,000 and depreciated by Rs. 500,000
was sold for Rs. 150,000.
(iv) The interest bearing loans relate to debentures which were issued at their nominal value. Rs.
400,000 of these debentures were redeemed at par during the year.
(v) Ordinary shares were issued for cash during the year.
(vi) Rs. 100,000 of current asset investments held as cash equivalents were sold during the year
for Rs. 94,000.
(vii) Dividends paid in the year were Rs. 200,000 relating to the 2014 proposed dividend and a Rs.
200,000 interim dividend for 2015.

Required
Prepare a statement of cash flows for Klea for the year ended 31 March 2015 in accordance with
IAS 7 using the indirect method.

3.2 STANDARD INC


The summarised statements of financial position of Standard Inc at 31 December 2014 and 2015
are as follows.
2015 2014
Rs. in ‘000 Rs. in ‘000
Issued share capital 150,000 100,000
Share premium 35,000 15,000
Retained earnings 41,000 14,000
Long-term loans 30,000 70,000
Payables 48,000 34,000
Bank overdraft – 14,000
Tax payable 33,000 21,500
Proposed dividends 15,000 7,500
Depreciation
Plant and machinery 54,000 45,000
Fixtures and fittings 15,000 13,000
———— ————
421,000 334,000
———— ————

© Emile Woolf International 19 The Institute of Chartered Accountants of Pakistan


Financial accounting and reporting II

Freehold property at cost 130,000 110,000


Plant and machinery at cost 151,000 120,000
Fixtures and fittings at cost 29,000 24,000
Inventories 51,000 37,000
Trade receivables 44,000 42,800
Long-term investments 4,600 –
Cash at bank 11,400 200
———— ————
421,000 334,000
———— ————
The following information is relevant:
(a) There had been no disposal of freehold property in the year.
(b) A machine tool which had cost Rs. 8,000,000 (in respect of which Rs. 6,000,000 depreciation
had been provided) was sold for Rs. 3,000,000, and fixtures which had cost Rs. 5,000,000 (in
respect of which depreciation of Rs. 2,000,000 had been provided) were sold for Rs.
1,00,0000. Profits and losses on those transactions had been dealt with through the statement
of profit or loss.
(c) The statement of profit or loss charge in respect of tax was Rs. 22,000,000.
(d) The premium paid on redemption of the long-term loan was Rs. 2,000,000, which has been
written off to the statement of profit or loss.
(e) The proposed dividend for 2014 had been paid during the year.
(f) Interest received during the year was Rs. 450,000. Interest charged in the statement of profit
or loss for the year was Rs. 6,400,000. Accrued interest of Rs. 440,000 is included in
payables at 31 December 2014 (nil at 31 December 2015).
(g) The government stock is a long term investment.

Required
Prepare a cash flow statement for the year ended 31 December 2015, together with notes as
required by IAS 7.

3.3 FALLEN
Fallen has prepared the following rough draft accounts for the year ended 31 December 2015.
Statement of profit or loss
Rs. in ‘000
Revenue 11,563
Cost of sales (5,502)
———
Gross profit 6,061
Distribution costs (402)
Administration expenses (882)
Interest payable (152)
———
Operating profit before tax 4,625
Taxation (35%) including deferred tax (1,531)
———
Profit after tax 3,094
Dividends (700)
———
Retained profit 2,394
———

© Emile Woolf International 20 The Institute of Chartered Accountants of Pakistan


Questions

Statements of financial position


31 December
2015 2014
Rs. in ‘000 Rs. in ‘000
Leasehold premises (net) 6,600 5,700
Plant, machinery and equipment (net) 5,040 3,780
Investments at cost 2,406 2,208
Inventories 2,880 1,986
Receivables 2,586 1,992
Bank – 576
——— ———
19,512 16,242
——— ———

31 December
2015 2014
Rs. in ‘000 Rs. in ‘000
Share capital 2,280 1,800
Share premium 2,112 1,800
Profit and loss account 9,108 6,714
Deferred taxation 202 138
Long-term loan (10%) 1,240 1,800
Provision for deferred repairs 1,202 1,016
Payables 1,026 702
Overdraft 222 –
Taxation
Corporation tax 1,730 2,038
Proposed dividends 390 234
——— ———
19,512 16,242
——— ———
The following data is relevant.
(1) The 10% long-term loan were redeemed at par.
(2) Plant and equipment with a written down value of Rs. 276,000 was sold for Rs. 168,000. New
plant was purchased for Rs. 2,500,000.
(3) Leasehold premises costing Rs. 1,300,000 were acquired during the year.
(4) The investments are highly liquid securities held for the short term.

Required
Prepare the cash flow statement and supporting notes in accordance with IAS 7 for Fallen Inc for
2015.

© Emile Woolf International 21 The Institute of Chartered Accountants of Pakistan


Financial accounting and reporting II

3.4 BIN QASIM MOTORS LIMITED


The summarised financial statements of Bin Qasim Motors Limited for the year to 30 September
2015, together with a comparative balance sheet, are:

Statement of profit or loss Rs. 000


Sales revenue 7,482
Cost of sales (4,284)
Gross profit 3,198
Operating expenses (1,479)
Interest payable (260)
Investment income 120
Profit before tax 1,579
Income tax (520)
Profit for the period 1,059

Statement of financial position as at 30 September


2015 2014
Rs. in ‘000 Rs. in ‘000
Assets
Non-current assets
Property, plant and equipment 2,344 1,908
Investment 690 nil
3,034 1,908
Current assets
Inventory 1,046 785
Trade accounts receivable 935 824
Short term treasury bills 120 50
Bank nil 122
2,101 1,781
Total assets 5,135 3,689

Total equity and liabilities


Equity:
Share capital 1,400 1,000
Reserves:
Share premium 460 60
Retained earnings
At beginning of the year 192 147
Net profit for period 1,059 65
Dividends (180) (20)
At end of the year 1,071 192
2,931 1,252
Revaluation surplus 90 40

© Emile Woolf International 22 The Institute of Chartered Accountants of Pakistan


Questions

Statement of financial position as at 30 September


2015 2014
Rs. in ‘000 Rs. in ‘000
Non-current liabilities
Deferred tax 439 400
Deferred income 275 200
10% Convertible loan stock nil 400
714 1,000
Current liabilities
Trade accounts payable 644 760
Accrued interest 40 25
Provision for negligence claim nil 120
Provision for income tax 480 367
Deferred income 100 125
Overdraft 136 nil
1,400 1,397
Total equity and liabilities 5,135 3,689
The following information is relevant
(i) Non-current assets
Property, plant and equipment is analysed as follows: Rs in ‘000

30 September 2015 30 September 2014


Cost/ Cost/
Valuation Depreciation NBV Valuation Depreciation NBV
Land and buildings 2,000 760 1,240 1,800 680 1,120
Plant 1,568 464 1,104 1,220 432 788
3,568 1,224 2,344 3,020 1,112 1,908

On 1 October 2014 Bin Qasim Motors Limited recorded an increase in the value of its land of
Rs. 150,000.
During the year an item of plant that had cost Rs. 500,000 and had accumulated depreciation
of Rs. 244,000 was sold at a loss (included in cost of sales) of Rs. 86,000 on its carrying
value.
(ii) Deferred income
Bin Qasim Motors Limited sells servicing contracts on certain types of machinery. Payments
are received in advance for a service which Bin Qasim Motors Limited must provide over a
number of following years. Income that relates to these contracts is deferred and recognised
in P&L as the period of service passes.
A credit of Rs. 125,000 for the current year’s recognition of deferred income has been
included revenue in this period.
(iii) Share capital and loan stocks
The increase in the share capital during the year was due to the following events:
(1) On 1 January 2015 there was a bonus issue (out of the revaluation surplus) of one
bonus share for every 10 shares held.
(2) On 1 April 2015 the 10% convertible loan stock holders exercised their right to convert
to ordinary shares. The terms of conversion were 25 ordinary shares of Rs. 1 each for
each Rs. 100 of 10% convertible loan stock.

© Emile Woolf International 23 The Institute of Chartered Accountants of Pakistan


Financial accounting and reporting II

(3) The remaining increase in the ordinary shares was due to a stock market placement of
shares for cash on 12 August 2015.
(iv) Provision for negligence claim
In June 2015 Bin Qasim Motors Limited made an out of court settlement of a negligence claim
brought about by a former employee. The dispute had been in progress for two years and Bin
Qasim Motors Limited had made provisions for the potential liability in each of the two
previous years. The unprovided amount of the claim at the time of settlement was Rs. 30,000
and this was charged to operating expenses.

Required
Prepare a statement of cash flows for Bin Qasim Motors Limited for the year to 30 September 2015
in accordance with IAS 7 Statement of Cash Flows.

3.5 ITTEHAD MANUFACTURING LTD


The financial statements of Ittehad Manufacturing Ltd for the year to 30 September 2015, together
with the comparative statement of financial position (balance sheet) for the year to 30 September
2014 are shown below:

Rs. in million
Sales revenue 3,820
Cost of sales (note 1) (2,620)
Gross profit for period 1,200
Operating expenses (note 1) (280)

920
Interest – Loan note (30)
Profit before tax 890
Taxation (270)
Net profit for the period 620

Statement of financial position as at 30 September:


2015 2016
Rs. in million Rs. in million
Non-current assets
Property, plant and equipment 1,890 1,830
Intangible assets (note 2) 670 300
2,560 2,130
Current assets
Inventory 1,420 940
Accounts receivable 990 680
Cash 70 nil
2,480 1,620
Total assets 5,040 3,750

© Emile Woolf International 24 The Institute of Chartered Accountants of Pakistan


Questions

Equity and liabilities


Ordinary shares of Rs. 1 each 750 500
Reserves
Share premium 350 100
Revaluation 190 nil
Retained earnings 1,860 1,600
3,150 2,200
Non-current liabilities (note 3) 610 240
Current liabilities (note 4) 1,280 1,310
Total equity and liabilities 5,040 3,750

Extract from statement of changes in equity


2015 2014
Rs. in million
Retained earnings – brought forward 1,600 1,000
Profit for the year 620 800
Dividends (320) (200)
Bonus issue (50)
Transfer from revaluation surplus 10 –
Retained earnings – carried forward 1,860 1,600
Notes to the financial statements:
(1) Cost of sales includes depreciation of property, plant and equipment of Rs. 320 million and a
loss on the sale of plant of Rs. 50 million.

(2) Intangible non-current assets:


2015 2014
Rs. in million
Deferred development expenditure 470 100
Goodwill 200 200
670 300

(3) Non-current liabilities:


10% loan note 300 100
Deferred tax 310 140
610 240

(4) Current liabilities:


Accounts payable 875 730
Bank overdraft nil 115
Accrued loan interest 15 5
Deferred income 260 300
Taxation 130 160
1,280 1,310

© Emile Woolf International 25 The Institute of Chartered Accountants of Pakistan


Financial accounting and reporting II

The following additional information is relevant:


(i) Intangible non-current assets:
The company successfully completed the development of a new product during the current
year, capitalising a further Rs. 500 million before amortisation charges for the period.
(ii) Property, plant and equipment/revaluation surplus:
‰ The company revalued its buildings by Rs. 200 million on 1 October 2014. The surplus
was credited to a revaluation surplus.
‰ New plant was acquired during the year at a cost of Rs. 250.
‰ Rs. 10 million has been transferred from the revaluation surplus to retained earnings as
a year-end adjustment in respect of the additional depreciation created by the
revaluation.
‰ The remaining movement on property, plant and equipment was due to the disposal of
obsolete plant.
(iii) Share issues:
On 1 October 2014 a bonus issue of 1 new share for every 10 held was made from retained
earnings. Ittehad Manufacturing Ltd made a further issue of ordinary shares for cash during
the year.

Required
(a) A statement of cash flows for Ittehad Manufacturing Ltd for the year to 30 September 2015
prepared in accordance with IAS 7 Statement of Cash Flows
(b) Comment briefly on the financial position of Ittehad Manufacturing Ltd as portrayed by the
information in your statement of cash flows.

3.6 WASEEM INDUSTRIES LIMITED


The following statements of financial position relate to Waseem Industries Limited for the years ended
December 31:
2015 2014
Rs. in million Rs. in million
ASSETS
Non-current assets
Fixed assets
Property, plant and equipment 242 182
Capital work-in-progress 20 18
262 200
Long term investments 75 100
Long term deposits 13 13
Total non-current assets 350 313

Current assets
Stocks-in-trade 55 48
Trade debts 51 38
Advances, prepayments and other
receivables 37 40
Cash and bank balances 11 20
Total current assets 154 146
TOTAL ASSETS 504 459

© Emile Woolf International 26 The Institute of Chartered Accountants of Pakistan


Questions

2015 2014
Rs. in million Rs. in million
EQUITY AND LIABILITIES
Shareholders' equity
Share capital 150 125
Share premium 55 80
Unappropriated profit 85 50
290 255
Non-current liabilities
Long term finances - Secured 94 118
Deferred liability - Gratuity (unfunded) 16 12
110 130
Current liabilities
Current portion of long term finances 25 22
Short term finances 13 6
Trade and other payables 66 46
104 74
TOTAL EQUITY AND LIABILITIES 504 459

Other relevant information is as follows:


(i) An interim bonus issue of one for five ordinary shares was made during the year out of share
premium. The company also approved final cash dividend of 10% (2014: 8%), in its annual
general meeting.
(ii) During the year, the company provided Rs. 17 million (2014: Rs. 13 million) on account of
depreciation. The details relating to disposal of property, plant and equipment are as follows:

Carrying amount Sale proceeds


Rs. m Rs. m
Plant and machinery 20 22
Vehicles 3 4

(iii) Advances, prepayments and other receivables include advance tax of Rs. 10 million (2014:
Rs. 7 million).
(iv) In 2015, the company paid Rs. 6 million on account of gratuity.
(v) Accrued mark-up on long term finances amounting to Rs. 7 million (2014: Rs. 9 million) is
included in trade and other payables. Financial charges included in the profit and loss account
are Rs. 16 million (2014 : Rs. 14 million).
(vi) Income tax expense for the year 2015 amounted to Rs. 19 million (2014: Rs. 13 million).

Required
Prepare a cash flow statement in accordance with the requirements of IAS 7 Cash Flow Statement”
using the indirect method.

© Emile Woolf International 27 The Institute of Chartered Accountants of Pakistan


Financial accounting and reporting II

3.7 JALIB INDUSTRIES LIMITED


Jalib Industries Limited is a listed company. The relevant information contained in the financial
statements for the year ended December 31, 2015 is as follows:
Statement of Financial Position
2015 2014
Rupees in million
Non-current assets
Property, plant and equipment 129.40 100.60
Capital work in progress 22.50 37.00
151.90 137.60
Current assets
Stock in trade 531.80 451.00
Trade debts 28.50 24.70
Advances and other receivables 37.40 42.00
Cash and bank 12.00 3.00
609.70 520.70
761.60 658.30
Equity
Issued, subscribed and paid-up capital 396.00 300.00
Share premium 45.00 12.00
Unappropriated profit 142.60 163.00
583.60 475.00
Non-current liabilities
Deferred liabilities 40.80 27.50
Long term loans 80.00 100.00
120.80 127.50
Current liabilities
Current portion of long term loans 18.00 20.00
Creditors, accrued and other liabilities 36.20 34.40
Dividend payable 3.00 1.40
57.20 55.80
761.60 658.30
Statement of profit or loss 2015
Rupees in million
Sales 2,535.00
Cost of goods sold (1,774.50)
Gross profit 760.50
Operating expenses (554.00)
Financial charges (10.50)
Loss on sale of fixed assets (4.60)
(569.10)
Profit before tax 191.40
Tax expense - Current (104.60)
- Deferred (2.20)
(106.80)
Profit after tax 84.60

© Emile Woolf International 28 The Institute of Chartered Accountants of Pakistan


Questions

The following supporting information is available:


(i) During the year, an amount of Rs. 42 million was transferred from capital work in progress
to property, plant and equipment.
(ii) The company sold property, plant and equipment having book value of Rs. 15 million for Rs.
10.4 million.
(iii) Depreciation for the year amounted to Rs. 27.7 million.
(iv) Trade debts written off during the year amounted to Rs. 1 million. It is the policy of the
company to maintain the provision for doubtful debts at 5% of trade debts.
(v) Advances and other receivables include advance tax of Rs. 3.6 million (2014: Rs. 2.2 million).
(vi) Deferred liabilities include deferred tax and provision for gratuity. There was no deferred
tax liability at the beginning of the year. Provision for gratuity made during the year amounted
to Rs. 15.5 million.
(vii) Creditors, accrued and other liabilities include accrued financial charges amounting to Rs. 5
million (2014: Rs. 6 million).
(viii) On January 15, 2016, the company declared final dividend for the year ended December 31,
2015 comprising 7.5% (2014: 25%) cash dividend and 12.5% (2014:10%) bonus shares, for
its ordinary shareholders.

Required
Prepare a statement of cash flow for the year ended December 31, 2015 in accordance with the
requirements of International Accounting Standards. Show all necessary workings.

3.8 APOLLO INDUSTRY LIMITED


Following are the relevant extracts from the financial statements of Apollo Industry Limited, a listed
company, for the year ended December 31, 2015.
Statement of financial position as at December 31, 2015

2015 2014
Rs. 000 Rs. 000
Issued, subscribed and paid up capital 25,000 20,000
Unappropriated profit 20,900 22,000
45,900 42,000
Surplus on revaluation of property, plant & equipment 7,000 8,000
Non-current liabilities
Staff gratuity 1,400 1,190
Deferred tax liability- net 590 -
1,990 1,190
Trade and other payables 4,200 6,250
59,090 57,440

Property, plant and equipment 35,000 25,500


Capital work in progress 5,500 10,000
Intangible assets 1,100 1,140
41,600 36,640

© Emile Woolf International 29 The Institute of Chartered Accountants of Pakistan


Financial accounting and reporting II

2015 2014
Rs. 000 Rs. 000
Deferred tax asset- net - 350
Long term deposits and prepayments 400 300
42,000 37,290
Current Assets
Tax refundable 950 800
Other current assets 15,700 12,125
Cash and bank balances 440 7,225
17,090 20,150
59,090 57,440

Statement of comprehensive Income for the year ended December 31, 2015

2015
Rs. 000
Sales 146,700
Cost of sales (127,500)
Gross profit 19,200

Operating expenses (15,000)


Financial charges (500)
Other income 2,800
(12,700)
Profit before tax 6,500
Tax expense - current (4,660)
- deferred (940)
Tax for the year (5,600)
Profit after tax 900

Other relevant information is as under:


(i) During the year, the company has issued 10% bonus shares.
(ii) Depreciation and amortization for the year amounted to Rs. 7 million.
(iii) WDV of assets disposed off during the year amounted to Rs. 1.2 million. (The assets had not
been revalued)
(iv) Other income includes interest earned on short term placements, amounting to Rs. 1 million.
The remaining amount represents gain on disposal of property, plant and equipment.
(v) Gratuity of Rs. 0.3 million was paid to outgoing employees.
(vi) Intangible assets worth Rs. 50 thousand were acquired during the year.
Required
Prepare the Statement of Cash Flows for the year ended December 31, 2015 in accordance with
the requirements of IAS - 7 (Statement of Cash Flows) using ‘indirect method’.

© Emile Woolf International 30 The Institute of Chartered Accountants of Pakistan


Questions

3.9 MARVEL ENGINEERING LIMITED


Following are the extracts from the draft financial statements of Marvel Engineering Limited
(MEL), a listed company, for the year ended 30 June 2015:
Statement of Financial Position

Rs. in million
2015 2014 2015 2014
Non current assets Share capital and
reserves
Property, plant and 633 410 Share capital 494 440
equipment (Rs. 10 each)
Long term investments 130 100 Share premium 8 -
763 510 Retained earnings 133 110
Current assets 635 550
Stock-in-trade 97 68 Non current
liabilities
Trade debts 133 57 Long term loans 330 110
Other receivables 100 120 Gratuity payable 55 50
Cash at bank 31 39 Deferred taxation 15 21
361 284 400 181
Current liabilities
Trade and other 73 56
payables
Tax payable - net 12 5
Dividend payable 4 2
89 63
1,124 794 1,124 794

Statement of profit or loss


2015
Rs. in million
Revenue 654
Cost of sales (458)
Gross profit 196
Operating expenses (68)
Financial charges (75)
Other income 35
(108)
Profit before tax 88
Income tax expense (21)
Profit after tax 67

© Emile Woolf International 31 The Institute of Chartered Accountants of Pakistan


Financial accounting and reporting II

Additional information:
(i) During the year, the company recognised a provision for impairment in respect of one of its
plant, amounting to Rs. 11 million. Total depreciation for the year amounted to Rs. 50 million.
(ii) It is the policy of the company to maintain a provision for doubtful debts at 5% of trade
debts. During the year, trade debts amounting to Rs. 6 million (2014: Rs. 2 million) were
written off.
(iii) Trade and other payables include accrued financial charges amounting to Rs. 7 million
(2014: Rs. 3 million).
(iv) On 15 July 2015, MEL’s board of directors proposed a final dividend of 10% for the year
ended 30 June 2015 (2014: 5% cash dividend and 5% bonus declared on 20 July 2014).
(v) Other income comprises of the following:

Rs. m
Dividend income 30
Gain on sale of vehicles (carrying value of Rs. 5 million) 2
Gain on sale of investments (carrying value of Rs. 10 million) 3
35

(vi) Gratuity paid during the year amounted to Rs. 6 million.

Required
Prepare the statement of cash flows for Marvel Engineering Limited for the year ended 30 June
2015.

© Emile Woolf International 32 The Institute of Chartered Accountants of Pakistan


Questions

CHAPTER 4 – CONSOLIDATED ACCOUNTS: STATEMENTS OF FINANCIAL


POSITION – BASIC APPROACH

4.1 HALL
Statements of financial position at 31 December 2015
Hall Stand
Rs. 000 Rs. 000
Assets
Non-current assets
Property, plant and equipment 35,000 20,000
Investment in Stand 12,000 –

Current assets 16,000 14,000


——— ———
63,000 34,000
——— ———
Equity and liabilities
Capital and reserves
Share capital 10,000 4,000
Retained earnings 13,000 12,000
——— ———
23,000 16,000
Non-current liabilities
8% Debenture loans 20,000 9,000

Current liabilities 20,000 9,000


——— ———
63,000 34,000
——— ———

On 1 January 2013 Hall acquired 75% of Stand for Rs. 12,000,000. At that date the balance on
Stand’s retained earnings was Rs. 8,000,000.

Required
Prepare the consolidated statement of financial position of Hall as at 31 December 2015.

4.2 HASSLE
Statements of financial position at 31 December 2015
Hassle Strife
Rs. Rs.
Investment in Strife 60,000 –
Sundry assets 247,500 226,600
———– —–——
307,500 226,600
———– ——–—
Share capital 120,000 50,000
Retained earnings 87,500 70,000
Liabilities 100,000 106,600
———– ——–—
307,500 226,600
———– ——–—

Hassle bought 80% of Strife when the balance on Strife’s retained profit was Rs. 50,000.

Required
Prepare the consolidated statement of financial position at 31 December 2015.

© Emile Woolf International 33 The Institute of Chartered Accountants of Pakistan


Financial accounting and reporting II

4.3 HYMN
The following are the summarised statements of financial position of a group of companies as at 31
December 2015.

Hymn Psalm
Rs. Rs.
Assets
Non-current assets
Property, plant and equipment 105,000 65,000
Investment 85,000
Current assets 220,000 55,000
———– ———–
410,000 120,000
———– ———–
Equity and liabilities
Equity
Share capital 100,000 50,000
Retained earnings 155,000 49,000
———– ———–
255,000 99,000
Current liabilities 155,000 21,000
———– ———–
410,000 120,000
———– ———–
Hymn purchased 80% of Psalm’s shares on 1 January 2015 when there was a credit balance on
that company’s retained earnings of Rs. 20,000.

Required
Prepare the Hymn group consolidated statement of financial position as at 31 December 2015.

4.4 HANG
On 31 December 2012, Hang acquired 60% of Swing for Rs. 140,000. At that date Swing had a
retained earnings balance of Rs. 50,000 and a share premium account balance of Rs. 49,000.
The following statements of financial position have been prepared as at 31 December 2015.
Hang Swing
Rs. Rs.
Assets
Non-current assets
Property, plant and equipment 240,000 180,000
Investment in Swing 140,000
Current assets 250,000 196,000
———– ———–
630,000 376,000
———– ———–

© Emile Woolf International 34 The Institute of Chartered Accountants of Pakistan


Questions

Equity and liabilities


Equity
Share capital 200,000 90,000
Share premium 25,000 49,000
Retained earnings 180,000 80,000
———– ———–
405,000 219,000
Current liabilities 225,000 157,000
———– ———–
630,000 376,000
———– ———–
Required
Prepare the consolidated statement of financial position of Hang and its subsidiary as at 31
December 2015.

4.5 HASH
Statements of financial position at 31 December 2015
Hash Stash
Rs. 000 Rs. 000
Investment in Stash (80%) 100,000 –
Sundry assets 207,500 226,600
———– —–——
307,500 226,600
———– ——–—

Share capital 120,000 50,000


Retained earnings 87,500 70,000
Liabilities 100,000 106,600
———– ——–—
307,500 226,600
———– ——–—

Hash purchased the shares in Stash on 30th September 2015.


Stash’s retained profit for the year ended 31st December 2015 was Rs. 24,000,000.

Required
Prepare the consolidated statement of financial position at 31 December 2015.

© Emile Woolf International 35 The Institute of Chartered Accountants of Pakistan


Financial accounting and reporting II

CHAPTER 5 – CONSOLIDATED ACCOUNTS: STATEMENTS OF FINANCIAL


POSITION – COMPLICATIONS

5.1 HAIL
The following are the draft statements of financial position of Hail and its subsidiary Snow as at
31 December 2015.
Hail Snow
Rs. 000 Rs. 000
Assets
Non-current assets
Property, plant and equipment 161,000 85,000
Investments 68,000
Current assets
Cash 7,700 25,200
Trade receivables 92,500 45,800
Snow current account 15,000 -
Inventory 56,200 36,200
———– ——–—
400,400 192,200
———– ——–—
Equity and liabilities
Shareholders’ equity
Share capital 100,000 50,000
Retained earnings 185,400 41,200
Share premium - 5,000
Capital reserve - 20,000
———– ——–—
285,400 116,200
Current liabilities 115,000 68,000
Hail current account - 8,000
———– ——–—
400,400 192,200
———– ——–—
Notes
(1) Snow has 50,000 shares in issues. Hail acquired 45,000 of these on 1 January 2012 for a
cost of Rs. 65,000,000 when the balances on Snow’s reserves were

Rs. 000
Share premium account 5,000
Capital reserve –
Retained earnings 10,000
(2) Hail declared a dividend of Rs. 3,000,000 before the year end and Snow declared one of Rs.
2,000,000. These transactions have not been accounted for.
(3) The current account difference is due to cash in transit.

Required
Prepare the consolidated statement of financial position as at 31 December 2015 of Hail.

© Emile Woolf International 36 The Institute of Chartered Accountants of Pakistan


Questions

5.2 HAIRY
The summarised statements of financial position of Hairy and Spider as at 31 December 2015 were
as follows.

Hairy Spider
Rs. 000 Rs. 000
Assets
Non-current assets
Property, plant and equipment 120,000 60,000
Investments 55,000 –
Current assets
Cash 11,000 4,000
Investments – 3,000
Trade receivables 72,600 19,100
Current account – Hairy – 3,200
Inventory 17,000 11,000
———– ———–
275,600 100,300
———– ———–
Equity and liabilities
Share capital 100,000 60,000
Share premium 20,000 –
Capital reserve 23,000 16,000
Retained earnings 91,900 7,300
Trade payables 38,000 17,000
Current account – Spider 2,700 –
———– ———–
275,600 100,300
———– ———–

The following information is relevant.


(1) On 31 December 2012, Hairy acquired 48,000 shares in Spider for Rs. 55,000,000 cash.
Spider has 60,000 shares in total.
(2) The inventory of Hairy includes Rs. 4,000,000 goods from Spider invoiced to Hairy at cost
plus 25%.
(3) The difference on the current account balances is due to cash in transit.
(4) The balance on Spider’s retained earnings was Rs. 2,300,000 at the date of acquisition. There
has been no movement in the balance on Spider’s capital reserve since the date of
acquisition.

Required
Prepare the consolidated statement of financial position of Hairy and its subsidiary Spider as at 31
December 2015.

© Emile Woolf International 37 The Institute of Chartered Accountants of Pakistan


Financial accounting and reporting II

5.3 HARD
On 31 December 2011, Hard acquired 60% of the ordinary share capital of Soft for Rs. 110 million.
At that date Soft had a retained earnings balance of Rs. 50 million and a share premium account
balance of Rs. 10 million.
The following statements of financial position have been prepared as at 31 December 2015.

Hard Soft
Rs. 000 Rs. 000
Assets
Non-current assets
Property, plant and equipment 225,000 175,000
Investments in Soft 110,000

Current assets 271,000 157,000


———– ———–
606,000 332,000
———– ———–
Equity and liabilities
Capital and reserves
Share capital 100,000 100,000
Share premium 15,000 10,000
Retained earnings 260,000 80,000
———– ———–
375,000 190,000
Current liabilities 231,000 142,000
———– ———–
606,000 332,000
———– ———–

During the year to 31 December 2015 Hard sold a tangible asset to Soft for Rs. 50 million. The asset
was originally purchased in the year to 31 December 2012 at a cost of Rs. 100 million and had a
useful economic life of five years.
Soft’s depreciation policy is 25% per annum based on cost. Both companies charge a full year’s
depreciation in the year of acquisition and none in the year of disposal.

Required
Prepare the consolidated statement of financial position of Hard and its subsidiary as at 31
December 2015.

© Emile Woolf International 38 The Institute of Chartered Accountants of Pakistan


Questions

5.4 HALE
On 1 July 2012 Hale acquired 128,000 of Sowen’s 160,000 shares. The following statements of
financial position have been prepared as at 31 December 2015.
Hale Sowen
Rs. 000 Rs. 000

Property, plant and equipment 152,000 129,600


Investment in Sowen 203,000 –
Inventory at cost 112,000 74,400
Receivables 104,000 84,000
Bank balance 41,000 8,000
———– ———–
612,000 296,000
 

Hale Sowen
Rs. 000 Rs. 000

Share capital 100,000 160,000


Retained earnings 460,000 112,000
Payables 52,000 24,000
———– ———–
612,000 296,000
 
The following information is available.
(1) At 1 July 2012 Sowen had a debit balance of Rs. 11 million on retained earnings.
(2) Property, plant and equipment of Sowen included land at a cost of Rs. 72 million. This land
had a fair value of Rs. 100,000 at the date of acquisition.
(3) The inventory of Sowen includes goods purchased from Hale for Rs. 16 million. Hale invoiced
those goods at cost plus 25%.

Required
Prepare the consolidated statement of financial position of Hale as at 31 December 2015.

5.5 HELLO
On 1 January 2012, Hello acquired 60% of the ordinary share capital of Solong for Rs. 110,000. At
that date Solong had a retained earnings balance of Rs. 60,000.
The following statements of financial position have been prepared as at 31 December 2015.
Hello Solong
Rs. Rs.
Assets
Non-current assets
Property, plant and equipment 225,000 175,000
Investments in Solong 110,000

Current assets 271,000 157,000


———– ———–
606,000 332,000
———– ———–

© Emile Woolf International 39 The Institute of Chartered Accountants of Pakistan


Financial accounting and reporting II

Equity and liabilities


Capital and reserves
Share capital 100,000 100,000
Retained earnings 275,000 90,000
———– ———–
375,000 190,000
Current liabilities 231,000 142,000
———– ———–
606,000 332,000
———– ———–

The fair value of Solong’s net assets at the date of acquisition was determined to be Rs. 170,000.
The difference between the book value and the fair value of the new assets at the date of acquisition
was due to an item of plant which had a useful life of 10 years from the date of acquisition.

Required
Prepare the consolidated statement of financial position of Hello and its subsidiary as at 31
December 2015.

5.6 HASAN LIMITED


On 1 April 2014, Hasan Limited acquired 90% of the equity shares in Shakeel Limited. On the same
day Hasan Limited accepted a 10% loan note from Shakeel Limited for Rs. 200,000 which was
repayable at Rs. 40,000 per annum (on 31 March each year) over the next five years. Shakeel
Limited’s retained earnings at the date of acquisition were Rs. 2,200,000.

Statements of financial position as at 31 March 2015


Hasan Shakeel
Limited Limited
Rs. 000 Rs. 000
Non-current assets
Property, plant and equipment 2,120 1,990
Intangible – software – 1,800
Investments – equity in Shakeel Limited 4,110 –
Investments – 10% loan note Shakeel 200 –
Limited
Investments – others 65 210
6,495 4,000
Current assets
Inventories 719 560
Trade receivables 524 328
Shakeel Limited current account 75 –
Cash 20
1,338 888
Total assets 7,833 4,888

Equity and liabilities:


Capital and reserves
Equity shares of Rs. 1 each 2,000 1,500
Share premium 2,000 500
Retained earnings 2,900 1,955
6,900 3,955

© Emile Woolf International 40 The Institute of Chartered Accountants of Pakistan


Questions

Hasan Shakeel
Limited Limited
Rs. 000 Rs. 000
Non-current liabilities
10% Loan note from Hasan Limited – 160
Government grant 230 40
230 200
Current liabilities
Trade payables 475 472
Hasan Limited current account – 60
Income taxes payable 228 174
Operating overdraft – 27
703 733
Total equity and liabilities 7,833 4,888
The following information is relevant:
(i) Included in Shakeel Limited’s property at the date of acquisition was a leasehold property
recorded at its depreciated historical cost of Rs. 400,000. The leasehold had been sub-let for
its remaining life of only four years at an annual rental of Rs. 80,000 payable in advance on 1
April each year. The directors of Hasan Limited are of the opinion that the fair value of this
leasehold is best reflected by the present value of its future cash flows. An appropriate cost of
capital for the group is 10% per annum.
The present value of a Rs. 1 annuity received at the end of each year where interest rates are
10% can be taken as:
3 year annuity Rs. 2.50
4 year annuity Rs. 3.20
(ii) The software of Shakeel Limited represents the depreciated cost of the development of an
integrated business accounting package. It was completed at a capitalised cost of Rs.
2,400,000 and went on sale on 1 April 2013. Shakeel Limited’s directors are depreciating the
software on a straight-line basis over an eight-year life (i.e. Rs. 300,000 per annum).
However, the directors of Hasan Limited are of the opinion that a five-year life would be more
appropriate as sales of business software rarely exceed this period.
(iii) The inventory of Hasan Limited on 31 March 2015 contains goods at a transfer price of Rs.
25,000 that were supplied by Shakeel Limited who had marked them up with a profit of 25%
on cost. Unrealised profits are adjusted for against the profit of the company that made them.
(iv) On 31 March 2015 Shakeel Limited remitted to Hasan Limited a cash payment of Rs. 55,000.
This was not received by Hasan Limited until early April. It was made up of an annual
repayment of the 10% loan note of Rs. 40,000 (the interest had already been paid) and Rs.
15,000 of the current account balance.
(v) The accounting policy of Hasan Limited for non-controlling interests (NCI) in a subsidiary is to
value NCI at a proportionate share of the net assets.
(v) An impairment test at 31 March 2015 on the consolidated goodwill concluded that it should be
written down by Rs. 120,000. No other assets were impaired.

Required
Prepare the consolidated statement of financial position of Hasan Limited as at
31 March 2015.

© Emile Woolf International 41 The Institute of Chartered Accountants of Pakistan


Financial accounting and reporting II

CHAPTER 6 – CONSOLIDATED ACCOUNTS: STATEMENTS OF COMPREHENSIVE


INCOME

6.1 HARRY
The following are the statements of profit or loss for the year ended 31 December 2015 of Harry and
its subsidiary Sally.

Harry Sally
Rs. 000 Rs. 000
Revenue 1,120 390
Cost of sales (610) (220)
Gross profit 510 170
Distribution costs (50) (40)
Administration costs (55) (45)
Operating profit 405 85
Investment income 20 4
Finance costs (18) (4)
Profit before tax 407 85
Income tax expense (140) (25)

Profit for the year 267 60

Rs. 000 Rs. 000


Retained profit brought forward 100 45
Profit for year 267 60
Dividends paid and proposed (50) (20)
Retained profit carried forward 317 85

The following information is relevant.


(1) Harry acquired 75% of Sally six years ago when Sally’s retained earnings were Rs. 9,000.
(2) Harry made sales to Sally totalling Rs. 100,000 in the year. At the year end the statement of
financial position of Sally included inventory purchased from Harry. Harry had taken a profit of
Rs. 3,000 on this inventory.
(3) Harry’s investment income includes Rs. 15,000 being its share of Sally’s dividends.

Required
Prepare a consolidated statement of profit or loss and a working showing the movement on
consolidated retained profit for the year ended 31 December 2015.

© Emile Woolf International 42 The Institute of Chartered Accountants of Pakistan


Questions

6.2 HORNY
Statements of profit or loss for the year ended 31 December 2015.

Horny Smooth
Rs. 000 Rs. 000
Revenue 304,900 195,300
Cost of sales (144,200) (98,550)
Gross profit 160,700 96,750
Operating costs (76,450) (52,100)
Operating profit 84,250 44,650
Investment income 10,500 2,600
Profit before tax 94,750 47,250
Income tax expense(42,900) (16,500)
Profit for the year 51,850 30,750

Statement of changes in equity (extracts) for the year ended 31 December 2015.
Horny Smooth
Rs. 000 Rs. 000
Retained earnings brought forward 80,200 31,000
Profit for the year 51,850 30,750
Proposed ordinary dividend (20,000) -
112,050 61,750
The following information is also available.
(1) Horny acquired 75% of the share capital of Smooth on 31 August 2015.
(2) Negative goodwill of Rs. 3.8 million arose on the acquisition.
(3) Profits of both companies are deemed to accrue evenly over the year except for the
investment income of Smooth all of which was received in November 2015.
(4) Horny has bought goods from Smooth throughout the year at Rs. 2 million per month. At the
year-end Horny does not hold any inventory purchased from Smooth.

Required
Prepare the consolidated statement of profit or loss and a working showing the movement on
consolidated retained profit for the year ended 31 December 2015.

6.3 HERON
Statements of financial position as at 30 June 2015
Heron Stork
Assets Rs. 000 Rs. 000
Non-current assets
Property, plant and equipment 31,000 15,000
Investment in Stork ( 1,000 ordinary shares) 1,000
32,000 15,000
Current assets 23,000 11,000
55,000 26,000

© Emile Woolf International 43 The Institute of Chartered Accountants of Pakistan


Financial accounting and reporting II

Shareholders’ equity and liabilities


Share capital (Rs. 0001 ordinary shares) 10,000 1,500
Share premium 5,000 –
Retained earnings 20,000 18,500
35,000 20,000
Non-current liabilities 15,000 –
Current liabilities 5,000 6,000
55,000 26,000

Heron acquired its shares in Stork when the balance on the retained earnings was Rs. 000nil.
Statements of profit or loss for the year ended 30 June 2015

Heron Stork
Rs. 000 Rs. 000
Revenue 30,000 25,000
Cost of sales (9,000) (10,000)

Gross profit 21,000 15,000


Distribution costs (3,000) (1,200)
Administrative expenses (1,000) (2,800)
Finance costs (2,000) –

Profit before tax 15,000 11,000


Income tax expense (3,000) (3,000)

Profit for the period 12,000 8,000


 

Statement of changes in equity for the year ended 30 June 2015 (extract)

Retained earnings brought forward 8,000 10,500


Profit for the financial year 12,000 8,000
——— ———
Retained earnings carried forward 20,000 18,500
 
Required
Prepare Heron’s consolidated statement of profit or loss, consolidated statement of financial position
and a working showing the movement on consolidated retained profit for Heron for the year ended
30 June 2015.

© Emile Woolf International 44 The Institute of Chartered Accountants of Pakistan


Questions

6.4 HANKS
Statements of financial position as at 31 December 2015
Hanks Streep Scott
Rs. 000 Rs. 000 Rs. 000
Assets
Non-current assets
Property, plant and equipment 32,000 25,000 20,000
Investments 33,500 – –
———– ——— ———
65,500 25,000 20,000
Current assets
Cash at bank and in hand 9,500 2,000 4,000
Trade receivables 20,000 8,000 17,000
Inventory 30,000 18,000 18,000
———– ——— ———
125,000 53,000 59,000
———– ——— ———
Equity and liabilities
Share capital 40,000 10,000 15,000
Share premium account 6,500 – –
Retained earnings 55,000 37,000 27,000
———– ——— ———
101,500 47,000 42,000
Current liabilities 23,500 6,000 17,000
———– ——— ———
125,000 53,000 59,000
  
Statements of profit or loss for the year ended 31 December 2015
Hanks Streep Scott
Rs. 000 Rs. 000 Rs. 000
Revenue 125,000 117,000 82,000
Cost of sales (65,000) (64,000) (42,000)
———– ———– ———
Gross profit 60,000 53,000 40,000
Distribution costs (21,000) (14,000) (16,000)
Administrative expenses (14,000) (8,000) (7,000)
———– ———– ———
Profit before taxation 25,000 31,000 17,000
Income tax expense (10,000) (9,000) (5,000)
———– ———– ———
Profit after tax 15,000 22,000 12,000
  

© Emile Woolf International 45 The Institute of Chartered Accountants of Pakistan


Financial accounting and reporting II

Statement of changes in equity (extract) for the year ending


31 December 2015
Hanks Streep Scott
Rs. 000 Rs. 000 Rs. 000
Retained earnings brought forward 40,000 15,000 15,000
Retained profit for the financial year 15,000 22,000 12,000
Dividends – – –
———– ———– ———
Retained earnings carried forward 55,000 37,000 27,000
  
You are given the following additional information
(1) Hanks owns 80% of Streep’s shares. These were purchased in 2012 for Rs. 20.5 million cash,
when the balance on Streep’s retained earnings stood at Rs. 7million.
(2) In 2010 Hanks purchased 60% of the shares of Scott by the issue of shares with a nominal
value of Rs. 6.5 million. These shares were issued at a premium of Rs. 6.5 million. At that
date the retained earnings of Scott stood at Rs. 3 million and the fair value of the net assets of
Scott was Rs. 24 million. It was agreed that any undervaluation of the net assets should be
attributed to land. This land was still held at 31 December 2015.
(3) Included in the inventory of Scott and Streep at 31 December 2015 are goods purchased from
Hanks for Rs. 5.2 million and Rs. 3.9 million respectively. Hanks aims to earn a profit of 30%
on cost. Total sales from Hanks to Scott and to Streep were Rs. 8 million and Rs. 6 million
respectively.
(4) Hanks and Streep each proposed a dividend before the year end of Rs. 2 million and Rs. 2.5
million respectively. No accounting entries have yet been made for these.
(5) Hanks has carried out annual impairment tests on goodwill in accordance with IFRS 3 and
IAS 36. The estimated recoverable amount of goodwill at 31 December 2012 was Rs. 5
million and at 31 December 2015 was Rs. 4.5 million.

Required
Prepare the consolidated statement of profit or loss and consolidated statement of changes in equity
for the year ended 31 December 2015 and the consolidated statement of financial position at that
date.

© Emile Woolf International 46 The Institute of Chartered Accountants of Pakistan


Questions

CHAPTER 7 – IAS 16: PROPERTY, PLANT AND EQUIPMENT

7.1 ROONEY
(a) Rooney has recently finished building a new item of plant for its own use. The item is a press
for use in the manufacture of industrial diamonds. Rooney commenced construction of the
asset on 1st April 2013 and completed it on 1st April 2015.
1st January 2013, Rooney took out a loan to finance the construction of the asset. Interest is
charged on the loan at the rate of 5% per annum. The annual interest must be paid in four
equal instalments at the end of each quarter. Rooney capitalises interest on manufactured
assets in accordance with the rules in IAS 23 Borrowing costs.
The costs (excluding finance costs) of manufacturing the asset were Rs. 28 million.
Required
State the IAS 23 rules on the capitalisation of borrowing costs, calculate the cost of the asset
on initial recognition and explain the amount of borrowing cost capitalised.
(b) The press comprises two significant parts, the hydraulic system and the ‘frame’. The hydraulic
system has a three year life and the ‘frame’ has an eight year life. Rooney depreciates plant
on a straight line basis. The cost of the hydraulic system is 30% of the total cost of
manufacture.
Rooney uses the IAS 16 revaluation model in accounting for diamond presses and revalues
these assets on an annual basis.
Revaluation surpluses or deficits are apportioned between the hydraulic system and the
‘frame’ on the basis of their year end book values before the revaluation.
Required
Explain the IAS 16 rules on accounting for significant parts of property, plant and equipment
and show the accounting treatment of the diamond press in the financial statements for the
financial years ending:
(i) 31st March 2016 (assume that the press has a fair value of Rs. 21 million)
(ii) 31st March 2017 (assume that the press has a fair value of Rs. 19.6 million).

7.2 EHTISHAM
The following information relates to the financial statements of Ehtisham for the year to 31 March
2015.
The head office of Ehtisham was acquired on 1 April 2012 for Rs. 1million. Ehtisham intend to
occupy the building for 25 years. On 31 March 2014 it was revalued to Rs. 1.15 million. On 31 March
2015, a surplus of vacant commercial property in the area had led to a fall in property prices and the
fair value was now only Rs. 0.8 million.

Required
Explain the correct accounting treatment for the above (with calculations if appropriate).

© Emile Woolf International 47 The Institute of Chartered Accountants of Pakistan


Financial accounting and reporting II

7.3 CARLY
The following is an extract from the financial statements of Carly on 31 December 2014.
Property, plant and equipment
Land and Plant and
buildings equipment Computers Total
Rs. Rs. Rs. Rs.
Cost
On 31 December 2014 1,500,000 340,500 617,800 2,458,300

Accumulated depreciation
On 31 December 2014 600,000 125,900 505,800 1,231,700

Carrying amount
On 31 December 2014 900,000 214,600 112,000 1,226,600

Accounting policies
Depreciation
Depreciation is provided at the following rates.
On land and buildings 2% per annum straight line on buildings only
On plant and equipment 25% reducing balance
On computers 33.33% per annum straight line
During 2015 the following transactions took place.
(1) On 31 December the land and buildings were revalued to Rs. 1,750,000. Of this amount, Rs.
650,000 related to the land (which had originally cost Rs. 500,000). The remaining useful life
of the buildings was assessed as 40 years.
(2) A machine which had cost Rs. 80,000 and had accumulated depreciation of Rs. 57,000 at the
start of the year was sold for Rs. 25,000 in the first week of the year.
(3) A new machine was purchased on 31 March 2015. The following costs were incurred:

Rs.
Purchase price, before discount, inclusive of reclaimable sales tax of Rs.3,000 20,000
Discount 1,000
Delivery costs 500
Installation costs 750
Interest on loan taken out to finance the purchase 300

(4) On 1 January it was decided to change the method of providing depreciation on computer
equipment from the existing method to 40% reducing balance.

Required
Produce the analysis of property, plant and equipment as it would appear in the financial statements
of Carly for the year ended 31 December 2015.

© Emile Woolf International 48 The Institute of Chartered Accountants of Pakistan


Questions

7.4 ADJUSTMENTS LIMITED


Adjustments Limited has carried out a review of its non-current assets.
(a) A lathe was purchased on 1 January 2009 for Rs. 150,000. The plant had an estimated useful
life of twelve years, residual value of nil. Depreciation is charged on the straight line basis. On
1 January 2015, when the asset’s net book value is Rs. 75,000, the directors decide that the
asset’s total useful life is only ten years.
(b) A grinder was purchased on 1 January 2012 for Rs. 100,000. The plant had an estimated
useful life of ten years and a residual value of nil. Depreciation is charged on the straight line
basis. On 1 January 2015, when the asset’s net book value is Rs. 70,000, the directors decide
that it would be more appropriate to depreciate this asset using the sum of digits approach.
The remaining useful life is unchanged.
(c) The company purchased a fifty year lease some years ago for Rs. 1,000,000. This was being
depreciated over its life on a straight line basis. On 1 January 2015, when the net book value
is Rs. 480,000 and twenty-four years of the lease are remaining, the asset is revalued to Rs.
1,500,000. This revised value is being incorporated into the accounts.

Required
Explain the effects of these changes on the depreciation for the year to 31 December 2015.

7.5 FAM
Fam had the following tangible fixed assets at 31 December 2014.

Cost Depreciation NBV


Rs. 000 Rs. 000 Rs. 000
Land 500 – 500
Buildings 400 80 320
Plant and machinery 1,613 458 1,155
Fixtures and fittings 390 140 250
Assets under construction 91 – 91
——— —— ———
2,994 678 2,316
  
In the year ended 31 December 2015 the following transactions occur.
(1) Further costs of Rs. 53,000 are incurred on buildings being constructed by the company. A
building costing Rs. 100,000 is completed during the year.
(2) A deposit of Rs. 20,000 is paid for a new computer system which is undelivered at the year
end.
(3) Additions to plant are Rs. 154,000.
(4) Additions to fixtures, excluding the deposit on the new computer system, are Rs. 40,000.
(5) The following assets are sold.

Cost Depreciation Proceeds


brought forward
Rs. 000 Rs. 000 Rs. 000
Plant 277 195 86
Fixtures 41 31 2

© Emile Woolf International 49 The Institute of Chartered Accountants of Pakistan


Financial accounting and reporting II

(6) Land and buildings were revalued at 1 January 2015 to Rs. 1,500,000, of which land is worth
Rs. 900,000. The revaluation was performed by Messrs Jackson & Co, Chartered Surveyors,
on the basis of existing use value on the open market.
(7) The useful economic life of the buildings is unchanged. The buildings were purchased ten
years before the revaluation.
(8) Depreciation is provided on all assets in use at the year end at the following rates.
Buildings 2% per annum straight line
Plant 20% per annum straight line
Fixtures 25% per annum reducing balance

Required
Show the disclosure under IAS 16 in relation to fixed assets in the notes to the published accounts
for the year ended 31 December 2015.

7.6 HUMAYUN CHEMICALS LIMITED


(a) On July 1, 2013, Humayun Chemicals Limited acquired a machine at a cost of Rs. 10
million. The useful life of the machine and its salvage value was estimated at 5 years and Rs.
3.0 million respectively. The cost of machine is being depreciated under the straight line
method.
Based on the practice followed by similar type of companies, the company has determined
that the remaining useful economic life of the machine is six years. It has also been
established that the residual value at the end of the useful life will be equal to 10% of the cost
of machine.

Required
Compute the depreciation expenses and other adjustments (if any) required to be made in
the financial statements of the company for the year ended June 30, 2015 under each of the
following assumptions:
(i) the review of useful life and residual value was carried out on June 30, 2015;
(ii) the review of useful life and residual value was carried out on June 30, 2014 but in the
financial statements for the year then ended the depreciation expense was erroneously
recorded on the previous basis.
(b) Discuss the requirements of International Accounting Standard(s) in respect of
estimation and revision of useful life of an item of property, plant and equipment.

7.7 FARADAY PHARMACEUTICAL LIMITED


Faraday Pharmaceutical Limited (FPL) acquired a building for Rs. 200 million on July 1, 2011. The
following information relating to the building is available:
(i) It is being depreciated on the straight line basis, over 20 years.
(ii) FPL uses the revaluation model for subsequent measurement of its property, plant and
equipment and accounts for revaluations on the net replacement value method. The details of
revaluation carried out by the independent valuers during the past years are as follows:

Revaluation date Fair value Rupees in million

July 1, 2012 230


July 1, 2013 170
July 1, 2014 180
(iii) FPL transfers the maximum possible amount from the revaluation surplus to retained
earnings on an annual basis.

© Emile Woolf International 50 The Institute of Chartered Accountants of Pakistan


Questions

(iv) There is no change in the useful life of the building.

Required
Prepare the journal entries to record the above transactions from the date of acquisition of the
building to the year ended June 30, 2015.
(Ignore deferred tax)

7.8 SCIENTIFIC PHARMA LIMITED


Scientific Pharma Limited (SPL) is a manufacturer of pharmaceutical products. In January 2015,
one of its plants suffered a major break down. It was repaired at a cost of Rs. 1.5 million but the
production capacity was reduced significantly. The plant was ready for production on June 30, 2015.
At that time the company’s engineers advised that the plant could be used at a reduced level for 3
years only. The factory was estimated to have a recoverable amount of Rs. 19,277,000 at June 30, 2015

Other related information is as under:


(i) The plant was imported at FOB price of US$ 800,000. The payment was made at the time of
shipment on July 1, 2005 at Rs. 52 per US$. Other charges including installation cost
amounted to Rs. 7 million. Installation of the plant was completed on December 31, 2005 and
commercial production commenced from April 1, 2006.
(ii) The company uses straight line method of deprecation. Depreciation is charged from the
month the asset is available for use upto the month prior to disposal. At the time of purchase,
the estimated useful life of the plant was estimated at 15 years whereas the salvage value
was estimated at Rs. 2.0 million.
(iii) Based on the report of a professional independent valuer, the plant was revalued on July 1,
2010 at Rs. 45 million. There was however, no change in estimated useful life of the plant.
(iv) The factory remained closed from April 1, to June 30, 2012 due to law and order situation.
(v) The salvage value has not changed since it was first estimated at the time of purchase.

Required
Prepare accounting entries for the year ended June 30, 2015. Give all the necessary calculations.
(Ignore taxation)

© Emile Woolf International 51 The Institute of Chartered Accountants of Pakistan


Financial accounting and reporting II

CHAPTER 8 – IAS 36: IMPAIRMENT OF ASSETS

8.1 ABA LIMITED


Aba Limited conducts its activities from two properties, a head office in the city centre and a property
in the countryside where staff training is conducted. Both properties were acquired on 1 April 2013
and had estimated lives of 25 years with no residual value. The company has a policy of carrying its
land and buildings at current values. However, until recently property prices had not changed for
some years. On 1 October 2015 the properties were revalued by a firm of surveyors. Details of this
and the original costs are:

Land Buildings
Rs. Rs.
Head office – cost 1 April 2013 500,000 1,200,000
– revalued 1 October 2015 700,000 1,350,000
Training premises – cost 1 April 2013 300,000 900,000
– revalued 1 October 2015 350,000 600,000

The fall in the value of the training premises is due mainly to damage done by the use of heavy
equipment during training. The surveyors have also reported that the expected life of the training
property in its current use will only be a further 10 years from the date of valuation. The estimated
life of the head office remained unaltered.
Note: Aba Limited treats its land and its buildings as separate assets. Depreciation is based on the
straight-line method from the date of purchase or subsequent revaluation.

Required
Prepare extracts of the financial statements of Aba Limited in respect of the above properties for the
year to 31 March 2016.

8.2 HUSSAIN ASSOCIATES LTD


The assistant financial controller of the Hussain Associates Ltd group has identified the matters
below which she believes may indicate impairment of one or more assets:
(a) Hussain Associates Ltd owns and operates an item of plant that cost Rs.640,000 and had
accumulated depreciation of Rs.400,000 at 1 October 2015. It is being depreciated at 12½%
on cost.
On 1 April 2016 (exactly half way through the year) the plant was damaged when a factory
vehicle collided into it. Due to the unavailability of replacement parts, it is not possible to repair
the plant, but it still operates, albeit at a reduced capacity. It is also expected that as a result
of the damage the remaining life of the plant from the date of the damage will be only two
years.
Based on its reduced capacity, the estimated present value of the plant in use is Rs.150,000.
The plant has a current disposal value of Rs.20,000 (which will be nil in two years’ time), but
Hussain Associates Ltd has been offered a trade-in value of Rs.180,000 against a
replacement machine which has a cost of Rs.1 million (there would be no disposal costs for
the replaced plant). Hussain Associates Ltd is reluctant to replace the plant as it is worried
about the long-term demand for the product produced by the plant. The trade-in value is only
available if the plant is replaced.

Required
Prepare extracts from the statement of financial position and statement of profit or loss of
Hussain Associates Ltd in respect of the plant for the year ended 30 September 2016. Your
answer should explain how you arrived at your figures.

© Emile Woolf International 52 The Institute of Chartered Accountants of Pakistan


Questions

(b) On 1 April 2015 Hussain Associates Ltd acquired 100% of the share capital of Sparkle
Limited, whose only activity is the extraction and sale of spa water. Sparkle Limited had been
profitable since its acquisition, but bad publicity resulting from several consumers becoming ill
due to a contamination of the spa water supply in April 2016 has led to unexpected losses in
the last six months. The carrying amounts of Sparkle Limited’s assets at 30 September 2016
are:

Rs.000
Brand (Sparkle Spring – see below) 7,000
Land containing spa 12,000
Purifying and bottling plant 8,000
Inventories 5,000
32,000

The source of the contamination was found and it has now ceased.
The company originally sold the bottled water under the brand name of ‘Sparkle Spring’, but
because of the contamination it has re-branded its bottled water as ‘Refresh’. After a large
advertising campaign, sales are now starting to recover and are approaching previous levels.
The value of the brand in the balance sheet is the depreciated amount of the original brand
name of ‘Sparkle Spring’.
The directors have acknowledged that Rs.1.5 million will have to be spent in the first three
months of the next accounting period to upgrade the purifying and bottling plant.
Inventories contain some old ‘Sparkle Spring’ bottled water at a cost of Rs.2 million; the
remaining inventories are labelled with the new brand ‘Refresh’. Samples of all the bottled
water have been tested by the health authority and have been passed as fit to sell. The old
bottled water will have to be relabelled at a cost of Rs.250,000, but is then expected to be sold
at the normal selling price of (normal) cost plus 50%.
Based on the estimated future cash flows, the directors have estimated that the value in use
of Sparkle Limited at 30 September 2016, calculated according to the guidance in IAS 36, is
Rs.20 million. There is no reliable estimate of the fair value less costs to sell of Sparkle
Limited.

Required
Calculate the amounts at which the assets of Sparkle Limited should appear in the
consolidated statement of financial position of Hussain Associates Ltd at 30 September 2016.
Your answer should explain how you arrived at your figures.

8.3 IMPS
A division of IMPS has the following non-current assets, which are stated at their carrying values
at 31 December Year 4:

Rs.m Rs.m

Goodwill 70

Property, plant and equipment:


Land and buildings 320
Plant and machinery 110
430
500

© Emile Woolf International 53 The Institute of Chartered Accountants of Pakistan


Financial accounting and reporting II

Because these assets are used to produce a specific product, it is possible to identify the cash
flows arising from their use. The management of IMPS believes that the value of these assets
may have become impaired, because a major competitor has developed a superior version of the
same product and, as a result, sales are expected to fall.
The following additional information is relevant:
Forecast cash inflows arising from the use of the assets are as follows:

Year ended 31 December:


Rs.m
Year 5 185
Year 6 160
Year 7 130

(i) The directors are of the opinion that the market would expect a pre-tax return of 12% on an
investment in an entity that manufactures a product of this type.
(ii) The land and buildings are carried at valuation. The surplus relating to the revaluation of
the land and buildings that remains in the revaluation reserve at 31 December Year 4 is
Rs.65 million. All other non-current assets are carried at historical cost.
(iii) The goodwill does not have a market value. It is estimated that the land and buildings
could be sold for Rs.270 million and the plant and machinery could be sold for Rs.50
million, net of direct selling costs.
Required
(a) Calculate the impairment loss that will be recognised in the accounts of IMPS.
(b) Explain how this loss will be treated in the financial statements for the year ended 31
December Year 4.

© Emile Woolf International 54 The Institute of Chartered Accountants of Pakistan


Questions

CHAPTER 9 – IAS 38: INTANGIBLE ASSETS

9.1 FAZAL
The following information relates to the financial statements of Fazal for the year to 31 March 2015.
The IT division has begun a training course for all managers in a new programming language at a
cost of Rs. 200,000. The consultants running the training course have quantified the present value of
the training benefits over the next two years to be Rs. 400,000. The project cost has been included
in the statement of financial position as a current asset. The accounting policy note identifies that the
costs will be written off over the next two years to match the benefits.

Required
Explain the correct accounting treatment for the above (with calculations if appropriate).

9.2 HENRY
During 2015 Henry has the following research and development projects in progress.
Project A was completed at the end of 2014. Development expenditure brought forward at the
beginning of 2015 was Rs. 412,500 on this project. Savings in production costs arising from this
project are first expected to arise in 2015. In 2015 savings are expected to be Rs. 100,000, followed
by savings of Rs. 300,000 in 2016 and Rs. 200,000 in 2017.
Project B commenced on 1 April 2015. Costs incurred during the year were Rs. 56,000. In addition
to these costs a machine was purchased on 1 April 2015 for Rs. 30,000 for use on the project. This
machine has a useful life of five years. At the end of 2015 there were still some uncertainties
surrounding the completion of the project.
Project C had been started in 2014. In 2014 the costs relating to this project of Rs. 36,700 had been
written off, as at the end of 2014 there were still some uncertainties surrounding the completion of
the project. Those uncertainties have now been resolved and a further Rs. 45,000 costs incurred
during the year.

Required
Show how the above would appear in the financial statements (including notes to the financial
statements) of Henry as of 31 December 2015.

9.3 TOBY
Toby entered into the following transactions during the year ended 31 December 2015. The directors
of Toby wish to capitalise all assets wherever possible.
(1) On 1 January Toby acquired the net assets of George for Rs. 105,000. The assets acquired
had the following book and fair values.

Book value Fair value


Rs. Rs.
Goodwill 5,000 5,000
Patents 15,000 20,000
Non-current assets 40,000 50,000
Other sundry net assets 30,000 25,000
––––––– ––––––––
90,000 100,000
 
The patent expires at the end of 2022. The goodwill arising from the above had a recoverable
value at the end of 2015 of Rs. 7,000.

© Emile Woolf International 55 The Institute of Chartered Accountants of Pakistan


Financial accounting and reporting II

(2) On 1 April Toby acquired a brand from a competitor for Rs. 50,000. The directors of Toby
have assessed the useful life of the brand as five years.
(3) During the year Toby spent Rs. 40,000 on developing a new brand name. The development
was completed on 30 June. The useful life of this brand has been assessed as eight years.
(4) The directors of Toby believe that there is total goodwill of Rs. 2 million within Toby and that
this has an indefinite useful life.

Required
Prepare the note to the financial statements for intangible assets as at 31 December 2015.

9.4 BROOKLYN
Brooklyn is a bio-technology company performing research for pharmaceutical companies. The
finance director has contacted your financial consulting company to arrange a meeting to discuss
issues relevant to the preparation of the financial statements for the year to 30th June 2015. Your
initial telephone conversation has provided the necessary background information.
1 On 1st August 2014 Brooklyn began investigating a new bio-process. On 1st September 2015,
the new process was widely supported by the scientific community and the feasibility project
was approved. A grant was then obtained relating to future work. Several pharmaceutical
companies have expressed an interest in buying the ‘know how’ when the project completes
in June 2016. The nominal ledger account set up for the project shows that the expenditure
incurred between 1st August 2014 and 30th June 2015 was Rs. 300,000 per month.
2 In August 2015, an employee lodged a legal claim against the company for damage to his
health as a result of working for the company for the two years through to 31st March 2014
when he had to retire due to ill health. He has argued that his health deteriorated as a result of
the stress from his position in the organisation. Brooklyn has denied the claim and has
appointed an employment lawyer to assist with contesting the case. The lawyer has advised
that there is a 25% chance that the claim will be rejected, 50% chance that the damages will
be Rs. 600,000 and 25% chance of Rs. 1 million. The company has an insurance policy that
will pay 10% of any damages to the company. The lawyer has said that the case could take
until 30th June 2018 to resolve. The present value of the estimated damages discounted at 8%
is Rs. 476,280 and Rs. 793,800 respectively.
3 Brooklyn owns several buildings, which include an administrative office in the centre of
London. The company has revalued these on a regular basis every five years and the next
valuation is due on 30th June 2017. Property prices have increased since the last review and
particularly for the London premises. The cost of engaging a professionally qualified valuer is
very expensive and so to reduce costs the finance director is proposing that the property
manager, who is a professionally qualified valuer, should value the London property and that
the increase in value should be included in the financial statements. The finance director is of
the opinion that the property prices may fall next year.

Required
Prepare notes for your meeting with the finance director which explain and justify the accounting
treatment of these issues, preparing calculations where appropriate and identifying matters on which
your require further information.

© Emile Woolf International 56 The Institute of Chartered Accountants of Pakistan


Questions

9.5 ZOUQ INC


Zouq Inc. is a multinational company. As part of its vision to expand its business in South Asia, it
purchased a 90% share of a locally incorporated company, Momin Limited. Following are the brief
details of the acquisition:

Date of acquisition January 1, 2014


Total paid up capital of Momin Limited (Rs. 10 each) 500,000,000

Purchase price per share Rs. 30


Net assets of Momin Limited (as per 2013 audited financial statements) 650,000,000

Fair value of net assets (other than intangible assets) of Momin Limited 1,100,000,000

Momin Limited has an established line of products under the brand name of “Badar”. On behalf
of Zouq Inc., a firm of specialists has valued the brand name at Rs. 100 million with an estimated
useful life of 10 years at January 1, 2014. It is expected that the benefits will be spread equally over
the brand’s useful life.
An impairment test of goodwill and brand was carried out on December 31, 2014 which
indicated an impairment of Rs. 50 million in the value of goodwill.
An impairment test carried out on December 31, 2015 indicated a decrease of Rs. 13.5 million in
the carrying value of the brand.

Required:
(a) What are the requirements of International Accounting Standards relating to amortization of
intangible assets having finite life?
(b) Prepare the ledger accounts for goodwill and the brand, showing initial recognition and all
subsequent adjustments.

9.6 STAR-BRIGHT PHARMACEUTICAL LIMITED


Star-Bright Pharmaceutical Limited (SPL), a listed company, purchased a brand on January 1,
2010 at a cost of Rs. 382 million. It has incurred a substantial amount on further development of
the brand, in subsequent years.
It is the policy of SPL to amortise the development expenditures which meet the recognition
criteria as given in IAS-38 ‘Intangible Assets’, over a period of ten years. The amortization
commences when the development expenditures first meet the recognition criteria. However, it
was discovered during the year 2015 that the development expenditure incurred after acquisition
had erroneously been written-off to the profit and loss account, details of which are as follows:

Year ended Rs. m


December 31, 2012 24
December 31, 2013 54
December 31, 2014 38
December 31, 2015 43

The draft financial statements (before correction of error) show that retained earnings as at
December 31, 2015 was Rs. 1,950 million (2014: Rs. 1,785 million).

Required
In accordance with the requirements of International Financial Reporting Standards, prepare
relevant extracts of the Statement of Financial Position along with the note on intangible assets
after incorporating the required corrections.
(Ignore tax)

© Emile Woolf International 57 The Institute of Chartered Accountants of Pakistan


Financial accounting and reporting II

9.7 RAISIN INTERNATIONAL


(a) Discuss the criteria that should be used while recognizing intangible assets arising from
research and development work.
(b) Raisin International (RI) is planning to expand its line of products. The related information for
the year ended 31 December 2015 is as follows:
(i) Research and development of a new product commenced on 1 January 2015. On 1
October 2015, the recognition criteria for capitalization of an internally generated
intangible asset were met. It is estimated that the product would have a useful life
of 7 years. Details of expenditures incurred are as follows:

Rs. m
Research work 4.50
Development work 9.00
Training of production staff 0.50
Cost of trial run 0.80
Total costs 14.80

(ii) The right to manufacture a well-established product under a patent for a period of five
years was purchased on 1 March 2015 for Rs. 17 million. The patent has an expected
remaining useful life of 10 years. RI has the option to renew the patent for a further
period of five years for a sum of Rs. 12 million.
(iii) RI has acquired a brand at a cost of Rs. 2 million. The cost was incurred in the month of
June 2015. The life of the brand is expected to be 10 years. Currently, there is no
active market for this brand. However, RI is planning to launch an aggressive marketing
campaign in February 2016.
(iv) In September 2014, RI developed a new production process and capitalised it as an
intangible asset at Rs. 7 million. The new process is expected to have an indefinite
useful life. During 2015, RI incurred further development expenditure of Rs. 3 million
on the new process which meets the recognition criteria for capitalization of an
intangible asset.

Required
In the light of International Financial Reporting Standards, explain how each of the above transaction
should be accounted for in the financial statements of Raisin International for the year ended 31
December 2015.

© Emile Woolf International 58 The Institute of Chartered Accountants of Pakistan


Questions

CHAPTER 10 – IFRS 16: LEASES

10.1 X LTD
X Ltd is considering acquiring a machine. It has two options; cash purchase at a cost of
Rs.11,420,000 or a lease.
The terms of the lease are as follows:
(i) The lease period is for four years from 1 January 2016 with an annual rental of
Rs.4,000,000 payable on 31 December each year.
(ii) The lessee is required to pay all repairs, maintenance and other incidental costs.
(iii) The interest rate implicit in the lease is 15% p.a.

Note:
Estimated useful economic life span of the machine is four years.

Required
(a) Prepare a schedule of the allocation of the finance charges in the books of X Limited for
the entire lease period.
(b) Prepare an extract of the Statement of Financial Position of X Limited as on 31 December
2016.

10.2 PROGRESS LTD


Progress Ltd. acquired a machine from Fine Rentals Ltd. on January 3, 2016 under a lease
agreement extending over three years.
The agreement required them to make an initial deposit of Rs.1,280,000 to be followed by three
annual payments of Rs.800,000 on 31 December each year starting from 2016.
The cash price of the machinery was Rs.3,200,000 and Fine Rentals Ltd. added 12% interest
which was duly communicated to Progress Ltd.
The annuity method is used to allocate interest.

Required
(a) Compute the interest element and the capital portion of the annual repayments; and
(b) Show the journal entries that will record the transaction resulting from the lease
agreement.

10.3 MIRACLE TEXTILE LIMITED


On 1 July 2014, Miracle Textile Limited (MTL) acquired a machine on lease, from a bank.
Details of the lease are as follows:
(i) Cost of machine is Rs.20 million.
(ii) The lease term and useful life is 4 years and 10 years respectively.
(iii) Instalment of Rs.5.80 million is to be paid annually in advance on 1 July.
(iv) The interest rate implicit in the lease is 15.725879%.
(v) At the end of lease term, MTL has an option to purchase the machine on payment of
Rs.2 million. The fair value of the machine at the end of lease term is expected to be
Rs.3 million.
MTL depreciates the machine on the straight line method to a nil residual value.

© Emile Woolf International 59 The Institute of Chartered Accountants of Pakistan


Financial accounting and reporting II

Required
Prepare relevant extracts of the statement of financial position and related notes to the financial
statements for the year ended 30 June 2016 along with comparative figures. Ignore taxation.

10.4 ACACIA LTD


On 1 April 2015 Acacia Ltd entered into the following lease agreement.
Plant with a fair value of Rs.275,000 was leased under an agreement which requires Acacia Ltd
to make annual payments of Rs.78,250 on 1 April each year, commencing on 1 April 2015, for
four years. After the four years Acacia Ltd has the option to continue to lease the plant at a
nominal rent for a further three years and is likely to do so as the asset has an estimated useful
life of six years. The present value of the lease payments is Rs.272,850. Acacia Ltd is
responsible for insuring and maintaining the plant during the period of the lease.
Acacia Ltd allocates finance charges on an actuarial basis. The interest rate implicit in the lease
is 10%.
Required
Prepare all relevant extracts from Acacia Ltd's financial statements for the year ended 31 March
2016.

10.5 SHOAIB LEASING LIMITED


Shoaib Leasing Limited (the lessor) has entered into a three year agreement with Sarfaraz
Limited (the lessee) to lease a machine with an expected useful life of 4 years. The cost of
machine is Rs. 2,100,000.
The following information relating to lease transaction is available:
(i) Date of commencement of lease is July 1, 2016.
(ii) The lease contains a purchase bargain option at Rs.100,000. At the end of the lease term,
the value of the machine will be Rs.300,000.
(iii) Lease instalments of Rs.860,000 are payable annually, in arrears, on June 30.
(iv) The implicit interest rate is 12.9972%.

Required
(a) Prepare the journal entries for the years ending June 30, 2017, 2018 and 2019 in the
books of lessor. Ignore tax.
(b) Produce extracts from the statement of financial position including relevant notes as at
June 30, 2017 to show how the transactions carried out in 2017 would be reflected in
the financial statements of the lessor.
(Disclosure of accounting policy is not required.)

10.6 AKBAR LTD.


Akbar Ltd. (AL) prepares financial statements on 31 March each year. On
1 April Year 4, AL sold a machine to another company, Shahwez Ltd. (SL), for Rs.850,000 and
then leased it back under a ten year arrangement. AL had purchased the machine exactly ten
years previously for Rs.500,000 and had charged total depreciation of Rs.60,000 on the machine
up to the date of disposal. Assume that the transfer of machine by the seller-lessee satisfies the
requirements of IFRS 15.
Details of the sale and leaseback arrangement are as follows:
‰ Consideration received from SL Rs.850,000
‰ Fair value at date of disposal Rs.550,000

© Emile Woolf International 60 The Institute of Chartered Accountants of Pakistan


Questions

‰ Lease rentals (payable at the end of each year) is Rs.100,000 and interest rate implicit in
the lease is 10% p.a

Required
How AL should reflect in its books of accounts:
a) Right-of-use retained by AL
b) Gain / loss on rights transferred

10.7 ALI LIMITED


Ali Limited entered into a sale and leaseback arrangement with a bank on
1 April 2015. The arrangement involved the sale at fair value of plant and machinery to the bank
for Rs.1,440,000.
This amount has been credited to Ali Limited’s operating income. The carrying amount of the
plant and machinery was Rs.840,000 and its remaining useful life was five years at 1 April 2015.
No depreciation has been charged in respect of this plant and machinery for the year ended 31
March 2016.
Under the terms of the lease, Ali Limited is to pay five annual payments at
31 March each year, of Rs.360,000 (in arrears). The first payment has been made and has been
debited to operating costs. The interest rate implicit in the lease is 8%. The transfer of asset does
not satisfy the requirements of IFRS 15.

Required
Explain how the above transaction should be accounted for, with all relevant calculations, in the
financial statements for the year ended 31 March 2016.

10.8 MOAZZAM TEXTILE MILLS LIMITED


Moazzam Textile Mills Limited (MTML) is facing severe financial difficulties. To improve the cash
flows, the management has decided to sell and lease back three power generators of the
company under three different sale and lease back arrangements which were signed on August
15, 2016. At the same time, MTML enters into a contract with the buyer-lessor for the right to use
the generators for 5 years, with annual payments of Rs.1,000,000 each for Generator A and
Generator B and Rs.1,500,000 for Generator C, payable at the end of each year. The interest
rate implicit in the lease is 4.5%, The related information as on
August 15, 2016 is given below:

Carrying Amount of
Cost Fair Value Value in Use
Value Financing

Rs.000 Rs.000 Rs.000 Rs.000 Rs.000

Generator A 10,000 7,500 6,000 6,500 6,000

Generator B 12,000 6,000 5,000 5,000 6,000

Generator C 10,000 7,000 10,000 12,000 10,000

Required
Prepare the accounting entries that should be recorded by the company on August 15, 2016 in
respect of the above transactions.
Note: Cost of making sale is negligible. Ignore tax and deferred tax implications, if any.

© Emile Woolf International 61 The Institute of Chartered Accountants of Pakistan


Financial accounting and reporting II

CHAPTER 11 – IAS 37: PROVISIONS CONTINGENT LIABILITIES AND


CONTINGENT ASSETS AND IAS 10: EVENTS AFTER THE REPORTING PERIOD
11.1 BADAR
The following information relates to the financial statements of Badar for the year to 31 March 2015.
The mining division of Badar has a 3 year operating licence from an overseas government. This
allows it to mine and extract copper from a particular site. When the licence began on 1 April 2014,
Badar started to build on the site. The cost of the construction was Rs. 500,000.
The overseas country has no particular environmental decommissioning laws. In its past financial
statements Badar has given information about the company’s environmental policy and has provided
examples to demonstrate that it is a responsible company that believes in restoring mining sites at
the end of the extraction period. The cost of removing the construction at the end of the three years
is estimated to be Rs. 100,000.
The cost of the site currently shown in the trial balance is Rs. 500,000. The company has a cost of
borrowing of 10%.

Required
Explain the correct accounting treatment for the above (with calculations if appropriate).

11.2 GEORGINA
Georgina Company is preparing its financial statements for the year ended 30 September 2015. The
following matters are all outstanding at the year end.
(1) Georgina is facing litigation for damages from a customer for the supply of faulty goods on 1
September 2015. The claim, which is for Rs. 500,000, was received on 15 October 2015.
Georgina’s legal advisors consider that Georgina is liable and that it is likely that this claim will
succeed. On 25 October 2015 Georgina sent a counter-claim to its suppliers for Rs. 400,000.
Georgina’s legal advisors are unsure whether or not this claim will succeed.
(2) Georgina’s sales director, who was dismissed on 15 September, has lodged a claim for Rs.
100,000 for unfair dismissal. Georgina’s legal advisors believe that there is no case to answer
and therefore think it is unlikely that this claim will succeed.
(3) Although Georgina has no legal obligation to do so, it has habitually operated a policy of
allowing customers to return goods within 28 days, even where those goods are not faulty.
Georgina estimates that such returns usually amount to 1% of sales. Sales in September
2015 were Rs. 400,000. By the end of October 2015, prior to the drafting of the financial
statements, goods sold in September for Rs. 3,500 had been returned.
(4) On 15 September 2015 Georgina announced in the press that it is to close one of its divisions
in January 2016. A detailed closure plan is in place and the costs of closure are reliably
estimated at Rs. 300,000, including Rs. 50,000 for staff relocation.

Required
State, with reasons, how the above should be treated in Georgina’s financial statements for the year
ended 30 September 2015.

11.3 EARLEY INC


Earley Inc is finalising its accounts for the year ended 31 December 2014. The following events have
arisen since the year end and the financial director has asked you to comment on the final accounts.

(a) At 31 December 2014 trade receivables included a figure of Rs. 250,000 in respect of
Nedengy Inc. On 8 March 2015, when the current debt was Rs. 200,000, Nedengy Inc went
into receivership. Recent correspondence with the receiver indicates that no dividend will be
paid to unsecured creditors.

© Emile Woolf International 62 The Institute of Chartered Accountants of Pakistan


Questions

(b) On 15 March 2015 Earley Inc sold its former head office building, Whitley Wood, for Rs. 2.7
million. At the year end the building was unoccupied and carried at a value of Rs. 3.1 million.

(c) Inventories at the year-end included Rs. 650,000 of a new electric tricycle, the Opasney. In
January 2015 the European Union declared the tricycle to be unsafe and prohibited it from
sale. An alternative market, in Bongolia, is being investigated, although the current price is
expected to be cost less 30%.

(d) Stingy Inc, a subsidiary in Outer Sonning, was nationalised in February 2015. The Outer
Sonning authorities have refused to pay any compensation. The net assets of Stingy Inc have
been valued at Rs. 200,000 at the year end.

(e) Freak floods caused Rs. 150,000 damage to the Southcote branch of Earley Inc in January
2015. The branch was fully insured.

(f) On 1 April 2015 Earley Inc announced a 1 for 1 rights issue aiming to raise Rs. 15 million.

Required
Explain how you would respond to the matters listed above.

11.4 ACCOUNTING TREATMENT


You have been asked to advise on the appropriate accounting treatment for the following situations
arising in the books of various companies. The year end in each case can be taken as 31 December
2015 and you should assume that the amounts involved are material in each case.

(a) At the year end there was a debit balance in the books of a company for Rs. 15,000,
representing an estimate of the amount receivable from an insurance company for an
accident claim. In February 2016, before the directors had agreed the final draft of the
published accounts, correspondence with lawyers indicated that Rs. 18,600 might be payable
on certain conditions.

(b) A company has an item of equipment which cost Rs. 400,000 in 2012 and was expected to
last for ten years. At the beginning of the 2015 financial year the book value was Rs. 280,000.
It is now thought that the company will soon cease to make the product for which the
equipment was specifically purchased. Its recoverable amount is only Rs. 80,000 at 31
December 2015.

(c) On 30 November a company entered into a legal action defending a claim for supplying faulty
machinery. The company’s solicitors advise that there is a 20% probability that the claim will
succeed. The amount of the claim is Rs. 500,000.

(d) An item has been produced at a manufacturing cost of Rs. 1,800 against a customer’s order
at an agreed price of Rs. 2,300. The item was in inventory at the year-end awaiting delivery
instructions. In January 2016 the customer was declared bankrupt and the most reasonable
course of action seems to be to make a modification to the unit, costing approximately Rs.
300, which is expected to make it marketable with other customers at a price of about Rs.
1,900.

(e) At 31 December a company has a total potential liability of Rs. 1,000,400 for warranty work on
contracts. Past experience shows that 10% of these costs are likely to be incurred, that 30%
may be incurred but that the remaining 60% is highly unlikely to be incurred.

Required
For each of the above situations outline the accounting treatment you would recommend and give
the reasoning of principles involved. The accounting treatment should refer to entries in the books
and/or the year-end financial statements as appropriate.

© Emile Woolf International 63 The Institute of Chartered Accountants of Pakistan


Financial accounting and reporting II

11.5 J-MART LIMITED


(a) Explain the terms “adjusting events” and “non-adjusting events” and give three examples of
each.
(b) J-Mart Limited, a chain of departmental stores has distributed its operations into four
Divisions i.e. Food, Furniture, Clothing and Household Appliances. The following information
has been extracted from the records:
(i) The company allows the dissatisfied customers to return the goods within 30 days. It is
estimated that 5% of the sales made in June 2015 will be refunded in July 2015.
(ii) On June 2, 2015, three employees were seriously injured as a result of a fire at the
company’s warehouse. They have lodged claims seeking damages of Rs. 2.0 million
from the company. The company’s lawyers have advised that it is probable that the
court may award compensation of Rs. 400,000.
(iii) Under a new legislation, the company is required to fit smoke detectors at all the stores
by December 31, 2015. The company has not yet installed the smoke detectors.
(iv) On June 20, 2015, the board of directors decided to close down the Household
Appliances Division. However, the decision was made public after June 30, 2015.
(v) The company has a large warehouse in Lahore which was acquired under a three-
year rent agreement signed on April 1, 2014. The agreement is non- cancellable
and the company cannot sub-let the warehouse. However, due to operational
difficulties, the company shifted the warehouse to a new location.
(vi) A 15% cash dividend was declared on July 5, 2015.

Required
Describe how each of the above issue should be dealt with in the financial statements for the year
ended June 30, 2010. Support your point of view in the light of relevant International
Accounting Standards.

11.6 AKBER CHEMICALS LIMITED


Akber Chemicals Limited is engaged in the business of manufacture and sale of different type
of chemicals. The following transactions have not yet been incorporated in the financial statements
for the year ended June 30, 2015:
(a) On June 15, 2015, one of its tankers carrying chemicals fell into a canal, thus polluting the
water. The company has never faced such a situation before. The company has neither any
legal obligation to clean the canal nor does it have any published environmental policy. In a
meeting held on July 26, 2015 the Board of Directors decided to clean the canal, which is
estimated to cost Rs. 5.5 million.
(b) During the second week of July 2015, a significant decline in the demand for company’s
products was observed which also led to a decrease in net realizable value of finished goods.
It was estimated that goods costing Rs. 25 million as at June 30, 2015 would only fetch
Rs. 23 million.
(c) On June 21, 2015, a customer lodged a claim of Rs. 2 million with the company as a
consignment dispatched on June 1, 2015 was not according to the agreed specifications. The
company’s inspection team found that this defect arose because of inferior quality of raw
materials supplied by the vendor. On June 28, 2015, the company lodged a claim for
damages of Rs. 5.0 million, with its vendor, which include reimbursement of the cost of raw
materials. The company anticipates that it will have to pay compensation to its customer and
would be able to recover 50% of the amount claimed from the vendor.

Required
Discuss how Akber Chemicals (Pvt.) Limited would deal with the above situations in its financial
statements for the year ended June 30, 2015. Explain your point of view with reference to the
guidance contained in the International Financial Reporting Standards.

© Emile Woolf International 64 The Institute of Chartered Accountants of Pakistan


Questions

11.7 QALLAT INDUSTRIES LIMITED


The following information pertains to Qallat Industries Limited (QIL) for its financial year ended
June 30, 2015:
(i) QIL sells all its products on one-year warranty which covers all types of defects. Previous
history indicates that 2% of the products contain major defects whereas 10% have minor
defects. It is estimated that if major defects were detected in all the products sold, repair
cost of Rs. 150 million would result. If minor defects were detected in all products sold,
repair cost of Rs. 70 million would result. Total sales for the year are amounted to Rs. 830
million.
(ii) QIL has two large warehouses, A and B. These were acquired under non-cancellable
lease agreements. Details are as follows:

Warehouse A Warehouse B
Effective date of agreement July 1, 2010 January 1, 2013
Lease period 10 years 8 years
Rental amount per month Rs. 450,000 Rs. 300,000
On account of serious operating difficulties, QIL vacated both the warehouses on January 1,
2015 and moved to a warehouse situated close to its factory. On the same day QIL sub-
let Warehouse A at Rs. 250,000 per month for the remaining lease period. Warehouse B
was sub-let on March 1, 2015 for Rs. 350,000 per month for the remaining lease period.
(iii) On July 18, 2015, QIL was sued by an employee claiming damages for Rs. 6 million on
account of an injury caused to him due to alleged violation of safety regulations on the part of
the company, while he was working on the machine on June 15, 2015. Before filing the suit,
he contacted the management on June 29, 2015 and asked for compensation of Rs. 4 million
which was turned down by the management. The lawyer of the company anticipates that the
court may award compensation ranging between Rs. 1.5 million to Rs. 3 million. However, in
his view the most probable amount is Rs. 2 million.
(iv) On November 1, 2014 a new law was introduced requiring all factories to install specialised
safety equipment within four months. The Equipment costing Rs. 5.0 million was ordered on
December 15, 2014 against 100% advance payment but the supplier delayed installation to
July 31, 2015. On August 5, 2015 the company received a notice from the authorities levying
a penalty of Rs. 0.4 million i.e. Rs. 0.1 million for each month during which the violation
continued. QIL has lodged a claim for recovery of the penalty from the supplier of the
equipment.
Required
Describe how each of the above issues should be dealt with in the financial statements for the year
ended June 30, 2015. Support your answer in the light of relevant International Accounting
Standards and quantify the effect where possible.

11.8 SKYLINE LIMITED


The following information pertains to Skyline Limited (SL) for the financial year ended December 31,
2015:
(i) A customer who owed Rs. 1 million was declared bankrupt after his warehouse was destroyed
by fire on February 10, 2016. It is expected that the customer would be able to recover 50% of
the loss from the insurance company.
(ii) An employee of SL forged the signatures of directors and made cash withdrawals of Rs. 7.5
million from the bank. Of these, Rs. 1.5 million were withdrawn before December 31, 2015.
Investigations revealed that an employee of the bank was also involved and therefore, under
a settlement arrangement, the bank paid 60% of the amount to SL on January 27, 2016.
(iii) SL has filed a claim against one of its vendors for supplying defective goods. SL’s legal
consultant is confident that damages of Rs. 1 million would be paid to SL. The supplier has
already reimbursed the actual cost of the defective goods.

© Emile Woolf International 65 The Institute of Chartered Accountants of Pakistan


Financial accounting and reporting II

(iv) A suit for infringement of patents, seeking damages of Rs. 2 million, was filed by a third party.
SL’s legal consultant is of the opinion that an unfavourable outcome is most likely. On the
basis of past experience he has advised that there is 60% probability that the amount of
damages would be Rs. 1 million and 40% likelihood that the amount would be Rs. 1.5 million.
Required
Advise SL about the amount of provision that should be incorporated and the disclosures that are
required to be made in the financial statements for the year ended December 31, 2015.

11.9 WALNUT LIMITED


Walnut Limited (WL) is engaged in the business of import and distribution of electronic appliances.
The following events took place subsequent to the reporting period i.e. 31 December 2015:
(i) On 15 January 2016, one of WL’s competitors announced launching of an upgraded version
of DVD players. WL’s inventories include a large stock of existing version of DVD players
which are valued at Rs. 15 million. Because of the introduction of the upgraded version, the
net realizable value of the existing version in WL’s inventory at 31 December 2015 has
reduced to Rs. 12.5 million.
(ii) On 20 December 2015, the board of directors decided to close down the division which
imports and sells mobile sets. This decision was made public on 29 December 2015.
However, the business was actually closed on 29 February 2016.
Net costs incurred in connection with the closure of this division were as follows:
Rs. m
Redundancy costs 1.50
Staff training 0.15
Operating loss from 1 July 2015 to closure of division 0.80
Less: Profit on sale of remaining mobile sets (0.50)
1.95

(iii) On 16 January 2016, LED TV sets valuing Rs. 3 million were stolen from a warehouse.
These sets were included in WL’s inventory as at 31 December 2015.
(iv) WL owns 9,000 shares of a listed company whose price as on 31 December 2015 was Rs. 22
per share. During February 2016, the share price declined significantly after the government
announced a new legislation which would adversely affect the company’s operations. No
provision in this regard has been made in the draft financial statements.
(v) On 31 January 2016, a customer announced voluntary liquidation. On 31 December 2015, this
customer owed Rs. 1.5 million.
(vi) On 15 February 2016, WL announced final dividend for the year ended 31 December 2015
comprising 20% cash dividend and 10% bonus shares, for its ordinary shareholders.

Required
Describe how each of the above transactions should be accounted for in the financial statements
of Walnut Limited for the year ended 31 December 2015. Support your answer in the light of
relevant International Financial Reporting Standards.

11.10 ATTOCK TECHNOLOGIES LIMITED


Attock Technologies Limited (ATL) manufactures five hi-tech products, each on a different plant. It is
in the process of preparing its financial statements for the year ended June 30, 2015. As the CFO of
the company, the following matters are under your consideration:
(i) Inventory carried at Rs. 25 million on June 30, 2015 was sold for Rs. 15 million after it had
been damaged in a flood, in July 2015.

© Emile Woolf International 66 The Institute of Chartered Accountants of Pakistan


Questions

(ii) On July 5, 2015 one of ATL’s corporate customers declared bankruptcy. The liquidator
announced on August 25, 2015 that 20% of the debt would be paid on liquidation.
(iii) A new product introduced by a competitor on August 1, 2015 had caused a significant
decline in the market demand of one of ATL’s major products. As a result, ATL is considering
a reduction in price and a cut in production.
(iv) On August 18, 2015 the government announced a retrospective increase in the tax rate
applicable to the company.
(v) The directors of ATL declared a dividend of Rs. 3 per share on August 28, 2015.

Required
State how the above events should be treated in ATL’s financial statements for the year ended
June 30, 2015. You may assume that all the above events are material to the company.

© Emile Woolf International 67 The Institute of Chartered Accountants of Pakistan


Financial accounting and reporting II

CHAPTER 12 – IAS 8: ACCOUNTING POLICIES, CHANGES IN ACCOUNTING


ESTIMATES AND ERRORS

12.1 WONDER LIMITED


Wonder Limited (WL) is engaged in the manufacturing and sale of textile machinery. Following are
the draft extracts of the statement of financial position and the statement of profit or loss for the year
ended 30 June 2015:
Statement of Financial Position

2015 2014
Rs. m Rs. m
Property, plant and equipment 189 130
Retained earnings 166 108
Deferred tax liability 45 27

Statement of profit or loss

2015 2014
Rs. m Rs. m
Profit before taxation 90 120
Taxation 32 42
Profit after taxation 58 78

Following additional information has not been taken into account in the preparation of the
above financial statements:
(i) Cost of repairs amounting to Rs. 20 million was erroneously debited to the machinery
account on 1 October 2013. The estimated useful life of the machine is 10 years.
(ii) On 1 July 2014, WL reviewed the estimated useful life of its plant and revised it from 5
years to 8 years. The plant was purchased on 1 July 2013 at a cost of Rs. 70 million.
Depreciation is provided under the straight line method. Applicable tax rate is 30%.

Required
Prepare relevant extracts (including comparative figures) for the year ended 30 June 2015 related to
the following:
(a) Statement of financial position
(b) Statement of profit or loss
(c) Statement of changes in equity
(d) Correction of error note

12.2 DUNCAN
Duncan Company has previously written off any expenditure on borrowing costs in the period in
which it was incurred.
The company has appointed new auditors this year. They have expressed the view that the previous
recognition of borrowing costs in the statement of profit or loss was in error. The company has
decided to correct the error retrospectively in accordance with IAS 8.
The financial statements for 2014 and the 2015 draft financial statements, both reflecting the old
policy, show the following.

© Emile Woolf International 68 The Institute of Chartered Accountants of Pakistan


Questions

Statement of changes in equity (extract)

2014 2015
Retained earnings Retained
earnings
Rs. 000 Rs. 000
Opening balance 22,500 23,950
Profit after tax for the period 3,200 4,712
Dividends paid (1,750) (2,500)
––––––– –––––––
Closing balance 23,950 26,162
 
Borrowing costs written off were Rs. 500,000 in 2014 and Rs. 600,000 in 2015.
The directors have calculated that borrowing costs, net of depreciation which should have been
included in property, plant and equipment had the correct policy been applied, are as follows.

Rs. 000
At 30 December 2013 400
At 31 December 2014 450
At 31 December 2015 180

Had the correct policy been in force depreciation of Rs. 450,000 would have been charged in 2014
and Rs. 870,000 in 2015.

Required
Show how the change in accounting policy must be reflected in the statement of changes in equity
for the year ended 31 December 2015. Work to the nearest Rs. 000.

12.3 MOHANI MANUFACTURING LIMITED


Mohani Manufacturing Limited is engaged in manufacturing of spare parts for motor car
assemblers. The audited financial statements for the year ended December 31, 2014 disclosed that
the profit and retained earnings were Rs. 21 million and Rs. 89 million respectively. The draft
financial statements for the year show a profit of Rs. 15 million. However, following adjustments are
required to be made:
(i) The management of the company has decided to change the method for valuation of raw
materials from FIFO to weighted average. The value of inventory under each method is as
follows:

FIFO Weighted Average


Rs. m Rs. m
December 31, 2013 37.0 35.5
December 31, 2014 42.3 44.5
December 31, 2015 58.4 54.4
(ii) In 2014, the company purchased a plant for Rs. 100 million. Depreciation on plant was
recorded at Rs. 25 million instead of Rs. 10 million. This error was discovered after the
publication of financial statements for the year ended December 31, 2014. The error is
considered to be material.

Required
Produce an extract showing the movement in retained earnings, as would appear in the
statement of changes in equity for the year ended December 31, 2015.

© Emile Woolf International 69 The Institute of Chartered Accountants of Pakistan


Financial accounting and reporting II

CHAPTER 13 – IAS 12: INCOME TAXES

13.1 FRANCESCA
On 30 June 2014 Francesca Company had a credit balance on its deferred tax account of Rs.
1,340,600 all in respect of the difference between depreciation and capital allowances.
During the year ended 30 June 2015 the following transactions took place.
(1) Rs. 45 million was charged against profit in respect of depreciation. The tax computation
showed capital allowances of Rs. 50 million.
(2) Interest receivable of Rs. 50,000 was reflected in profit for the period. However, only Rs.
45,000 of interest was actually received during the year. Interest is not taxed until it is
received.
(3) Interest payable of Rs. 32,000 was treated as an expense for the period. However, only Rs.
28,000 of interest was actually paid during the year. Interest is not an allowable expense for
tax purposes until it is paid.
(4) During the year Francesca incurred development costs of Rs. 500,600, which it has
capitalised. Development costs are an allowable expense for tax purposes in the period in
which they are paid.
(5) Land and buildings with a net book value of Rs. 4,900,500 were revalued to Rs. 6 million.
The tax rate is 30%. Francesca has a right of offset between its deferred tax liabilities and its
deferred tax assets.

Required
Calculate the deferred tax liability on 30 June 2015. Show where the increase or decrease in the
liability in the year would be charged or credited.

13.2 SHEP (I)


Shep was incorporated on 1 January 2015. In the year ended 31 December 2015 the company
made a profit before taxation of Rs. 121,000
During the period Shep made the following capital additions.
Rs.

Plant 48,000
Motor vehicles 12,000
During the period:
Accounting depreciation 11,000
Tax depreciation 15,000
Tax is chargeable at a rate of 30%.

Required
(a) Calculate the corporate income tax liability for the year ended 31st December 2015.
(b) Calculate the deferred tax balance that is required in the statement of financial position as at
31st December 2015.
(c) Prepare a note showing the movement on the deferred tax account and thus calculate the
deferred tax charge for the year ended 31st December 2015
(d) Prepare the statement of profit or loss note which shows the compilation of the tax expense
for the year ended 31st December 2015.

© Emile Woolf International 70 The Institute of Chartered Accountants of Pakistan


Questions

13.3 SHEP (II)


Continuing from the previous year. The following information is relevant for the year ended 31st
December 2016.
(a) Capital transactions
Rs.
Depreciation charged 14,000
Tax allowances 16,000

(b) Interest payable


On 1st April 2016 the company issued Rs. 25,000 of 8% convertible loan stock. Interest is paid
in arrears on 30th September and 30th March. Assume that tax relief on interest expense is
only given when the interest is paid.
(c) Interest receivable
On 1st April Shep purchased debentures having a nominal value of Rs. 4,000. Interest at 15%
pa is receivable on 30th September and 30th March. Assume that interest income is not taxed
until the cash is actually received.
(d) Provision for warranty
In preparing the financial statements for the year to 31st December 2016, Shep has
recognised a provision for warranty payments in the amount of Rs. 1,200. This has been
correctly recognised in accordance with IAS 37 and the amount has been expensed. Assume
that tax relief on the warranty cost is only given when the expense is paid.
(e) Fine
During the period Shep has paid a fine of Rs. 6,000. The fine is not tax deductible.
(f) Further information
The accounting profit before tax for the year was Rs. 125,000.

Tax is chargeable at a rate of 30%.

Required
(a) Calculate the corporate income tax liability for the year ended 31st December 2016.
(b) Calculate the deferred tax balance that is required in the statement of financial position as at
31st December 2016.
(c) Prepare a note showing the movement on the deferred tax account and thus calculate the
deferred tax charge for the year ended 31st December 2016
(d) Prepare the statement of profit or loss note which shows the compilation of the tax expense
for the year ended 31st December 2016.
(e) Prepare a note to reconcile the product of the accounting profit and the tax rate to the tax
expense for year ended 31st December 2016.

13.4 SHEP (III)


Continuing from the previous year. The following information is relevant for the year ended 31st
December 2017.
(a) Interest payable/Interest receivable
Shep still has Rs. 25,000 of 8% convertible loan stack in issue and still retains its holding in
the debentures purchased in 2004.

© Emile Woolf International 71 The Institute of Chartered Accountants of Pakistan


Financial accounting and reporting II

(b) Provision for warranty


During the year Shep had paid out Rs. 500 in warranty claims and provided for a further Rs.
2,000.
(d) Development costs
During 2017 Shep has capitalised development expenditure of Rs. 17,800 in accordance with
the provisions of IAS 38. Assume that tax relief on this expenditure is taken in full in the period
in which it is incurred.
(e) Further information
Rs.

Profit before taxation 175,000


Depreciation charged 18,500
Tax allowable depreciation 24,700
(f) Entertainment
Shep paid for a large office party during 2017 to celebrate a successful first two years of the
business. This cost Rs. 20,000. Assume that this expenditure is not tax deductible.

Tax is chargeable at a rate of 30%.

Required
(a) Calculate the corporate income tax liability for the year ended 31st December 2017.
(b) Calculate the deferred tax balance that is required in the statement of financial position as at
31st December 2017.
(c) Prepare a note showing the movement on the deferred tax account and thus calculate the
deferred tax charge for the year ended 31st December 2017
(d) Prepare the statement of profit or loss note which shows the compilation of the tax expense
for the year ended 31st December 2017.
(e) Prepare a note to reconcile the product of the accounting profit and the tax rate to the tax
expense for year ended 31st December 2017.

13.5 SHEP (IV)


Using the information provided in “Shep III” and assume that Shep is subject to a higher tax rate of
34% in 2017.

Required
(a) Calculate the corporate income tax liability for the year ended 31st December 2017.
(b) Calculate the deferred tax balance that is required in the statement of financial position as at
31st December 2017.
(c) Prepare a note showing the movement on the deferred tax account and thus calculate the
deferred tax charge for the year ended 31st December 2017
(d) Prepare the statement of profit or loss note which shows the compilation of the tax expense
for the year ended 31st December 2017.
(e) Prepare a note to reconcile the product of the accounting profit and the tax rate to the tax
expense for year ended 31st December 2017.

© Emile Woolf International 72 The Institute of Chartered Accountants of Pakistan


Questions

13.6 WAQAR LIMITED


Waqar Limited has provided you the following information for determining its tax and deferred tax
expense for the year 2014 and 2015:
(i) During the year ended December 31, 2015, the company’s accounting profit before tax
amounted to Rs. 40 million (2014: Rs. 30 million). The profit includes capital gains amounting
to Rs. 10 million (2014: Rs. 8 million) which are exempt from tax.
(ii) The accounting written down values of the fixed assets, as at December 31, 2013 were as
follows:
Accumulated Written
Cost Depreciation down value
Rs. m Rs. m Rs. m
Machinery 200 25 175
Furniture and fittings 50 10 40
No additions or disposals of fixed assets were made in the years 2014 and 2015.
(iii) Machinery was acquired on January 1, 2013 and is being depreciated on straight- line
basis over its estimated useful life of 8 years. The tax base of machinery as at December 31,
2013 was Rs. 90 million.
(iv) Furniture and fittings are also depreciated on the straight line basis at the rate of 10% per
annum. The tax base of furniture and fittings as at December 31, 2013 was Rs. 40.5 million.
(v) Normal rate of tax depreciation on both types of assets is 10% on written down value.
(vi) The tax rates for 2013, 2014 and 2015 were 35%, 35% and 30% respectively.

Required
For each year:
(a) Calculate the corporate income tax liability for the year.
(b) Calculate the deferred tax balance that is required in the statement of financial position as at
the year end.
(c) Prepare a note showing the movement on the deferred tax account and thus calculate the
deferred tax charge for the year.
(d) Prepare the statement of profit or loss note which shows the compilation of the tax expense.
(e) Prepare a note to reconcile the product of the accounting profit and the tax rate to the tax
expense.

13.7 SHAKIR INDUSTRIES


Given below is the statement of profit or loss of Shakir Industries for the year ended December
31, 2015:
2015
Rs. m
Sales 143.00
Cost of goods sold (96.60)
Gross profit 46.40
Operating expenses (28.70)
Operating profit Other income 17.70
Profit before interest and tax 3.40
Financial charges 21.10
Profit before tax (5.30)
15.80

© Emile Woolf International 73 The Institute of Chartered Accountants of Pakistan


Financial accounting and reporting II

Following information is available:


(i) Operating expenses include an amount of Rs. 0.7 million paid as penalty to SECP on non-
compliance of certain requirements of the Companies Act, 2017.
(ii) During the year, the company made a provision of Rs. 2.4 million for gratuity. The actual
payment on account of gratuity to outgoing members was Rs. 1.6 million.
(iii) Lease payments made during the year amounted to Rs. 0.65 million which include
financial charges of Rs. 0.15 million. As at December 31, 2015, obligations against assets
subject to finance lease stood at Rs. 1.2 million. The movement in assets held under
finance lease is as follows:

Rs. m
Opening balance – 01/01/2015 2.50
Depreciation for the year (0.7)
Closing balance – 31/12/2015 1.80
(iv) The details of owned fixed assets are as follows:

Accounting Tax
Rs. m Rs. m
Opening balance – 01/01/2015 12.50 10.20
Purchased during the year 5.3 5.3
Depreciation for the year (1.1) (1.65)
Closing balance – 31/12/2015 16.70 13.85
(v) Capital work-in-progress as on December 31, 2015 include financial charges of Rs. 2.3
million which have been capitalised in accordance with IAS-23 “Borrowing Costs”. However,
the entire financial charges are admissible, under the Income Tax Ordinance, 2001.
(vi) Deferred tax liability and provision for gratuity as at January 1, 2015 was Rs. 0.55 million
and Rs. 0.7 million respectively.
(vii) Applicable income tax rate is 35%.

Required
Based on the available information, compute the current and deferred tax expenses for the year
ended December 31, 2015.

13.8 MARS LIMITED


Mars Limited (ML) is engaged in the manufacturing of chemicals. On July 1, 2014 it obtained a
motor vehicle on lease from a bank. Details of the lease agreement are as follows:
(i) Cost of motor vehicle is Rs. 1,600,000.
(ii) Instalments of Rs. 480,000 are to be paid annually in advance.
(iii) The lease term and useful life is 4 years and 5 years respectively.
(iv) The interest rate implicit in the lease is 13.701%.
ML follows a policy of depreciating the motor vehicles over their useful life, on the straight-line
method. However, the tax department allows only the lease payments as a deduction from taxable
profits.
The tax rate applicable to the company is 30%. ML’s accounting profit before tax for the year
ended June 30, 2015 is Rs. 4,900,000.
There are no temporary differences other than those evident from the information provided
above.

© Emile Woolf International 74 The Institute of Chartered Accountants of Pakistan


Questions

Required
(a) Prepare journal entries in the books of Mars Limited for the year ended June 30, 2015 to
record the above transactions including tax and deferred tax.
(b) Prepare a note to the financial statements related to disclosure of finance lease liability, in
accordance with the requirements of IFRS.
(Ignore comparative figures.)

13.9 BILAL ENGINEERING LIMITED


Bilal Engineering Limited earned profit before tax amounting to Rs. 50 million during the year
ended December 31, 2015. The accountant of the company has submitted draft accounts to the
Finance Manager along with the following information which he believes could be useful in
determining the amount of taxation:
(i) Accounting deprecation for the year is Rs. 10 million which includes Rs. 1 million charged
on the difference between cost and revalued amount.
(ii) A motor vehicle costing Rs. 1 million was taken on lease in 2014. Related clauses of the
lease agreement are as under:

‰ Annual instalment of Rs. 0.3 million is payable annually in advance.


‰ The lease term and useful life is 4 years and 5 years respectively.

‰ The interest rate implicit in the lease is 13.701% per annum.


‰ Accounting depreciation on the leased vehicle is included in the depreciation
referred to in para (i) above.
(iii) Tax depreciation on the assets owned by the company is Rs. 7 million.
(iv) Research and development expenses of Rs. 15 million were incurred in 2013 and are being
amortised over a period of 15 years. For tax purposes research and development expenses
are allowed to be written off in 10 years. However, 10% of these expenses were not verifiable
and have not been claimed.
(v) Expenses amounting to Rs. 0.25 million were disallowed in 2012. Out of these Rs. 0.15
million were allowed in appeal, during the current year. The company had initially expected
that the full amount would be allowed but has decided not to file a further appeal.
(vi) The applicable tax rate is 35%.

Required
(a) Prepare journal entries in respect of taxation, for the year ended December 31, 2015.
(b) Prepare a reconciliation to explain the relationship between tax expense and accounting profit
as is required to be disclosed under IAS 12 Income Taxes.

13.10 GALAXY INTERNATIONAL


The following information relates to Galaxy International (GI), a listed company, which was
incorporated on January 1, 2014.
(i) The (loss) / profit before taxation for the years ended December 31, 2014 and 2015 amounted
to (Rs. 1.75 million) and Rs. 23.5 million respectively.
(ii) The details of accounting and tax depreciation on fixed assets is as follows:
2015 2014
Rs. m Rs. m
Accounting depreciation 15 15
Tax depreciation 6 45

© Emile Woolf International 75 The Institute of Chartered Accountants of Pakistan


Financial accounting and reporting II

(iii) In 2014, GI accrued certain expenses amounting to Rs. 2 million which were disallowed by
the tax authorities. However, these expenses are expected to be allowed on the basis of
payment in 2015.
(iv) GI earned interest on Special Investment Bonds amounting to Rs. 1.0 million and Rs. 1.25
million in the years 2014 and 2015 respectively. This income is exempt from tax.
(v) GI operates an unfunded gratuity scheme. The provision during the years 2014 and 2015
amounted to Rs. 1.7 million and Rs. 2.2 million respectively. No payment has so far been
made on account of gratuity.
(vi) The applicable tax rate is 35%.

Required
Prepare a note on taxation for inclusion in the company’s financial statements for the year ended
December 31, 2015 giving appropriate disclosures relating to current and deferred tax expenses
including a reconciliation to explain the relationship between tax expense and accounting profit.

13.11 APRICOT LIMITED


The following information relates to Apricot Limited (AL), a listed company, for the financial year
ended 31 December 2015:
(i) The profit before tax for the year amounted to Rs. 60 million (2014: Rs. 45 million).
(ii) The accounting and tax written down value of fixed assets as on 31 December 2014 was Rs.
95 million and Rs. 90 million respectively. Accounting depreciation for the year is Rs. 10
million (2014: Rs. 9 million) whereas tax depreciation for the year is Rs. 8 million (2014: Rs. 7
million).
(iii) During the year, AL sold a machine for Rs. 3 million and recognised a profit of Rs. 0.5 million.
The tax written down value of the machine as on 31 December 2014 was Rs. 2 million.
There were no other additions/disposals of fixed assets in 2014 and 2015.
(iv) AL earned capital gain of Rs. 6 million (2014:Nil) on sale of shares of a listed company. This
income is exempt from tax.
(v) Bad debt expenses recognised during the year was Rs. 5 million (2014: Rs. 7 million).
(vi) Bad debts written off during the year amounted to Rs. 3 million (2014: Rs. 4 million).
(vii) Deferred tax liability and provision for bad debts as on 31 December 2011 was Rs.
18.90 million and Rs. 9 million respectively.
(viii) The company’s assessed brought forward losses up to 31 December 2011 amounted to
Rs. 19.25 million.
(ix) Applicable tax rate is 35%.

Required
Prepare a note on taxation for inclusion in AL’s financial statements for the year ended 31
December 2015 giving appropriate disclosures relating to current and deferred tax expenses
including comparative figures for 2014 and a reconciliation to explain the relationship between 2015
tax expense and 2015 accounting profit.

© Emile Woolf International 76 The Institute of Chartered Accountants of Pakistan


Questions

CHAPTER 14: IAS 33: EARNINGS PER SHARE

14.1 AIRCON LTD


Mr Hamad, currently owns 20 million shares in Aircon Ltd. He recently received the published
financial statements of Aircon Ltd for the year ended 31 March 2016. Mr Hamad is not sure how
the performance of the company during the year will affect the market value of the entity’s shares
but he is aware that the earnings per share statistics are often used by analysts in assessing the
performance of listed companies.
Extracts from these published financial statements and other relevant information are given
below.
Statement of profit or loss for the period ended 31 March 2016

2016 2015
Rs.’m Rs.’m
Revenue 18,000 15,300
Cost of sales (11,340) (9,180)
Gross profit 6,660 6,120
Operating expenses (3,420) (3,240)
Operating profit 3,240 2,880
Interest payable (540) (576)
Profit before tax 2,700 2,304
Taxation (846) (720)
Profit after tax 1,854 1,584

Statement of financial position as at 31 March 2016


2016 2015
Rs.’m Rs.’m Rs.’m Rs.’m
Intangible assets 5,400 -
Tangible assets 7,200 6,660
 
12,600 6,660
Current Assets
Inventory 2,340 1,800
Receivables 2,700 2,160
Cash in bank 180 5,220 162 4,122
   
17,820 10,782
 
Capital and Reserves
Share Capital 2,700 900
Share Premium 4,860 900
Retained Earnings 1,620 1,206
 
9,180 3,006

© Emile Woolf International 77 The Institute of Chartered Accountants of Pakistan


Financial accounting and reporting II

2016 2015
Rs.’m Rs.’m Rs.’m Rs.’m
Current Liabilities
Trade Payables 3,060 2,160
Taxation 900 756
Bank Overdraft 1,080 1,260
 
5,040 4,176
 
14,220 7,182
15% Loan Note 3,600 3,600
 
17,820 10,782
 

The following information is also relevant:


(i) The share capital of the company comprises Rs.1 equity shares only.
(ii) On 1 October 2015, the company made a rights issue to existing shareholders of two new
shares for every one share held at a price of Rs.5.94 per share and paid issue cost of
Rs.180,000.
(iii) The market price of shares immediately before the rights issue was Rs.6.30 per share.
(iv) No other changes took place in the equity capital of Aircon Ltd in the year ended 31 March
2016.

Required
(a) Compute EPS for the year and the comparative figures that will be included in the
published financial statements of Aircon Ltd for the year ended 31 March 2016.
(b) Using the extracts you have been provided with, write a report to Mr Hamad identifying the
key factors which led to the change in the EPS of Aircon Ltd since the year ended 31
March 2016.
(c) Comment on the relevance of the EPS statistics to shareholders.

© Emile Woolf International 78 The Institute of Chartered Accountants of Pakistan


Questions

CHAPTER 15 – IAS 23: BORROING COST

15.1 SPIN INDUSTRIES LIMITED


On September 1, 2014, Spin Industries Limited (SIL) started construction of its new office
building and completed it on May 31, 2015. The payments made to the contractor were as follows:

Date of Payment Rupees


September 1, 2014 10,000,000
December 1, 2014 15,000,000
February 1, 2015 12,000,000
June 1, 2015 9,000,000
In addition to the above payments, SIL paid a fee of Rs. 8 million on September 1, 2014 for obtaining
a permit allowing the construction of the building.

The project was financed through the following sources:


(i) On August 1, 2014 a medium term loan of Rs. 25 million was obtained specifically for the
construction of the building. The loan carried mark up of 12% per annum payable semi-
annually. A commitment fee @ 0.5% of the amount of loan was charged by the bank.
Surplus funds were invested in savings account @ 8% per annum. On February 1, 2015 SIL
paid the six monthly interest plus Rs. 5 million towards the principal.
(ii) Existing running finance facilities of SIL
‰ Running finance facility of Rs. 28 million from Bank A carrying mark up of 13%
payable annually. The average outstanding balance during the period of construction
was Rs. 25 million.
‰ Running finance facility of Rs. 25 million from Bank B. The mark up accrued during
the period of construction was Rs. 3 million and the average running finance
balance during that period was Rs. 20 million.

Required
Calculate the amount of borrowing costs to be capitalised on June 30, 2015 in accordance with the
requirements of International Accounting Standards. (Borrowing cost calculations should be
based on number of months).

15.2 GRANITE CORPORATION


On 1 March 2014, Granite Corporation (GC) started the construction of a new plant to meet the
growing demand for its products. The new plant was completed at a cost of Rs. 100 million on 31
May 2015.

GC financed the cost of the project from the following sources:


(i) On 1 March 2014, a 7-year loan of Rs. 70 million was obtained specifically for the construction
of the plant. The loan carried mark-up @ 13% per annum payable semi-annually. An
arrangement fee @ 1% of the loan amount was paid to the bank.
Two instalments, each comprising of repayment of principal of Rs. 5 million with interest, were
paid on 31 August 2014 and 28 February 2015.
(ii) GC also has a running finance facility of Rs. 100 million carrying mark-up @ 14% per
annum. Average utilization of this facility, prior to commencement of construction was Rs. 10
million. Any additional amount required for the project was provided through this facility.
(iii) Surplus funds were used to reduce the running finance utilization or invested in savings
account @ 8% per annum.

© Emile Woolf International 79 The Institute of Chartered Accountants of Pakistan


Financial accounting and reporting II

Payments made to the contractor were as follows:

Payment date Rs. m


01 March 2014 25
31 January 2015 65
30 September 2015 10

The construction work was suspended from 1 February 2015 to 28 February 2015. The suspension
was caused due to delay in shipment of essential components for the installation of the plant.

Required
Calculate the amount of borrowing costs that may be capitalised during the years ended 30 June
2014 and 2015 in accordance with the requirements of International Financial Reporting Standards.

15.3 IMRAN LIMITED


On January 1, 2015, Imran Limited started the construction of its new factory. The
construction period is approximately 15 months and the cost is estimated at Rs. 80 million.
The work has been divided into 5 phases and payment to contractor shall be made on
completion of each phase.
In the year the company had the following sources of finance available.
(i) Rights i s s u e o f shares amounting to Rs. 15 million on January 1, 2015. The company
usually pays a dividend of 10% each year.
(ii) Bank loan of Rs. 32 million carrying a mark-up of 13% was raised on March 1, 2015. (This
loan was outstanding for 306 days in the year).
(iii) On August 1, 2015, Rs. 10 million were borrowed from the bank. Interest thereon, is payable
at the rate of 11%. (This loan was outstanding for 153 days in the year).
Investment income on temporary investment of the borrowings amounted to Rs. 0.5 million.
The details of bills submitted by the contractor, during the year are as follows:

Particulars Date of payment Rupees

On completion of 1st phase March 1, 2015 20,000,000

On completion of 2nd phase April 1, 2015 18,000,000

On completion of 3rd phase October 1, 2015 16,000,000

On completion of 4th phase Payment not yet made 17,000,000

On June 1, 2015, the Building Control Authority issued instructions for stoppage of work on account
of certain discrepancies in the completion plan. The company filed a petition in the Court and the
matter was decided in the company’s favour on July 31, 2015. Work recommenced after a delay
of 61 days.
The following periods may be relevant:

Period Days
March 1 to December 31 306
April 1 to December 31 275
August 1 to December 31 153
October 1 to December 31 92

© Emile Woolf International 80 The Institute of Chartered Accountants of Pakistan


Questions

Required
a) Assuming that the loans were taken specifically for the project, calculate the amount of
borrowing costs that s h o u l d be capitalised i n t h e p e r i o d e n d i n g December 31, 2015
in accordance with the requirements of IAS 23 Borrowing Costs.
b) Assuming that the loans constituted general finance, calculate the amount of borrowing
costs that s h o u l d be capitalised i n t h e p e r i o d e n d i n g December 31, 2015 in
accordance with the requirements of IAS 23 Borrowing Costs.

15.4 QURESHI STEEL LIMITED


On July 1, 2014, Qureshi Steel Limited (QSL) signed an agreement with Pak Construction Limited
for construction of a factory building at a cost of Rs. 100 million. It was agreed that the factory
would be ready for use from January 1, 2016. The terms of payments were agreed as under:
(i) 10% advance payment would be made on signing of the agreement. The advance paid would
be adjusted at 10% of the quarterly progress bills.
(ii) 5% retention money would also be deducted from the progress bills. Retention money will be
refunded one year after completion of the factory building.
(iii) Progress bills will be raised on last day of each quarter and settled on 15th of the next month.
The under mentioned progress bills were received and settled by QSL as per the agreement:

Invoice date Amount (Rs. )

September 30, 2014 30 million


December 31, 2014 20 million

March 31, 2015 10 million

June 30, 2015 15 million

On April 30, 2015 an invoice of Rs. 1.5 million was raised by the contractor for damages sustained at
the site, on account of rains. After negotiations, QSL finally agreed to make additional payment of
Rs. 1.0 million to compensate the contractor. The amount was paid on May 15, 2015. It is expected
that 75% of the payment would be recovered from the insurance company.

The cost of the project has been financed through the following sources:
(i) Issue of right shares amounting to Rs. 15 million, on September 1, 2014. The company has
been following a policy of paying dividend of 20% for the past many years.
(ii) Bank loan of Rs. 25 million obtained on December 1, 2014. The loan carries a markup of 13%
per annum. The principal is repayable in 5 half yearly equal instalments of Rs. 5 million each
along with the interest, commencing from May 31, 2015. Loan processing charges of Rs. 0.5
million were deducted by the bank at the time of disbursement of loan. Surplus funds, when
available, were invested in short term deposits at 8% per annum.
(iii) Cash withdrawals from the existing running finance facility provided by a bank. Average
running finance balance for the year was Rs. 60 million. Markup charged by the bank for the
year was Rs. 9 million.

Required
Compute cost of capital work in progress for the factory building as of June 30, 2015 in
accordance with the requirements of relevant IFRSs.
(Borrowing costs calculations should be based on number of months)

© Emile Woolf International 81 The Institute of Chartered Accountants of Pakistan


Financial accounting and reporting II

CHAPTER 16 – ETHICAL ISSUES IN FINANCIAL REPORTING

16.1 ETHICAL ISSUES


Waheed is a chartered accountant, recently employed by AA plc as deputy to the finance director,
Arif (also a chartered accountant). AA plc is listed on the Lahore stock exchange.
On Waheed’s first day on the job he met with Arif who said ‘Look, keep it to yourself but I’m having a
second interview next week for a new job. The first thing that I need you to do is to review the
financial statements before the auditors arrive. I qualified a few years ago and am not up to date on
all of the little technicalities in IFRS. You should now these better than me and you’ll know more
about what the auditors might focus on. We must do our best to present the financial statements in
the most favourable light as the bonus paid to employees (including me) depends on profit being
more than 10% bigger than last year’s and remember that you qualify for this too. Keep this in mind
when you carry out the review as we do not really want to find anything. Do well at this and I might
put in a good word for you when I leave as I’m sure you’ll be a great replacement for me.”

Required
Explain the ethical issues inherent in the above conversation and what Waheed should do about
them.

16.2 SINDH INDUSTRIES LTD


Jafar has recently been appointed as financial controller to Sindh Industries Ltd. Until a month ago,
Sindh Industries had a finance director, who resigned suddenly, due to ill health. Since Jafar joined
the company, he has learned that his resignation was related to stress caused by a series of
disagreements with the managing director about the performance of the business.. The directors
have not yet appointed a replacement.
It is now March 2016 and you have been asked to finalise the financial statements for the year
ended 31 December 2015. The draft statement of profit or loss extract and statement of financial
position are shown below:
Draft statement of profit or loss for the year ended 31 December 2015

Rs. 000
Profit before tax 2,500

Draft statement of financial position at 31 December 2015

Rs. 000
Property, plant and equipment 12,000
Current assets 3,500
Total assets 15,500

Share capital 2,000


Retained earnings 6,000
Equity 8,000
Non-current liabilities 5,000
Current liabilities 2,500
Total equity and liabilities 15,500

© Emile Woolf International 82 The Institute of Chartered Accountants of Pakistan


Questions

During the year ended 31 December 2015 Sindh Industries entered into the following transactions.
(1) Just before the year end Sindh Industries signed a contract to deliver consultancy services for
a period of 2 years at a fee of Rs. 500,000 per annum. The full amount of this fee has been
paid in advance and is non-refundable.
(2) Sindh Industries has constructed a new factory. The construction has been financed from the
pool of existing borrowings. Land at a cost of Rs. 1.8 million was acquired on 1 February 2015
and construction began on 1 June 2015. Construction was completed on 30 September 2015
at an additional cost of Rs. 2.7 million. Although the factory was usable from that date, full
production did not commence until 1 December 2015. Throughout the year the company’s
average borrowings were as follows:

Annual
interest
Amount rate
Rs. %
Bank overdraft 1,000,000 9.75
Bank loan 1,750,000 10
Debenture 2,500,000 8

An amount of Rs. 450,000 has been included in property, plant and equipment in respect of
borrowing costs relating to the construction of the factory. The useful life of the factory has
been estimated at 20 years. No depreciation has been charged for the year. The reason for
this is that the factory has only been in use for one month and that the depreciation charge
would be immaterial.
(3) A blast furnace with a carrying amount at 1 January 2015 of Rs. 3.5 million has been
depreciated in the draft financial statements on the basis of a remaining life of 20 years. In
December 2015 the directors carried out a review of the useful lives of various significant
items of plant and machinery, including the blast furnace and came to the conclusion that the
useful life of the furnace was 20 years at 31 December 2015. The reasoning behind this
judgement was that the lining of the furnace had been replaced in the last week of December
20X6 at a cost of Rs. 1.4 million. Provided that the lining is replaced every five years, the life
of the furnace can be extended accordingly. You have found a report, commissioned by the
previous finance director and prepared by a firm of asset valuation specialists, which
assesses the remaining useful life of the main structure of the furnace at 1 January 2015 at 15
years and the lining of the furnace at 5 years. You have also found evidence that the
managing director has seen this report.
Jafar has had a conversation with the managing director who told him, “We need to make the
figures look as good as possible so I hope you’re not going to start being difficult. The
consultancy fee is non-refundable so there’s no reason why we can’t include it in full. I think
we should look at our depreciation policies. We’re writing off our assets over far too short a
period. As you know, we’re planning to go for a stock market listing in the near future and
being prudent and playing safe won’t help us do that. It won’t help your future with this
company either.”

Required
(a) Explain the required IFRS accounting treatment of these issues, preparing relevant
calculations where appropriate.
(b) Prepare a revised draft of the statement of profit or loss extract for the year ended
31 December 2015 and the statement of financial position at that date.
(c) Discuss the ethical issues arising from your review of the draft financial statements and the
actions that you should consider.

© Emile Woolf International 83 The Institute of Chartered Accountants of Pakistan


Financial accounting and reporting II

© Emile Woolf International 84 The Institute of Chartered Accountants of Pakistan


Certificate in Accounting and Finance
Financial accounting and reporting II

SECTION
B
Answers

© Emile Woolf International 85 The Institute of Chartered Accountants of Pakistan


Financial accounting and reporting II

CHAPTER 2 – IAS 1: PRESENTATION OF FINANCIAL STATEMENTS

2.1 LARRY
Statement of profit or loss
For the year ended 31 December 2015
Rs. in
million
Revenue 3,304
Cost of sales (2,542 + 118 – 127) (2,533)
Gross profit 771
Other income 20
Distribution costs (175)
Administrative expenses (342)
Profit before tax 274
Income tax expense (75)
Profit for the period 199
Statement of financial position
As at 31 December 2015
Assets Rs. in
million
Non-current assets
Property, plant and equipment (2,830 – 918) 1,912
Intangible assets (26 – 5) 21
1,933
Current assets
Inventories 127
Trade and other receivables 189
Cash (89 +2) 91
407
Total assets 2,340
Equity and liabilities
Equity
Share capital 400
Retained earnings (1,562 + 199) 1,761
2,161
Non-current liabilities
Long-term borrowings (18 x 2/3) 12
Current liabilities
Trade and other payables 86
Current portion of long-term borrowing (18 ÷ 3) 6
Current tax payable 75
167
Total equity and liabilities 2,340

© Emile Woolf International 86 The Institute of Chartered Accountants of Pakistan


Answers

2.2 MINGORA IMPORTS LIMITED


Statement of profit or loss for the year ended 31 December 2015
Rs. in
million
Revenue 1,740
Change in inventories of finished goods and work-in-progress (W3) 40
Staff costs (W3) (620)
Depreciation and other amortisation expense (W3) (42)
Other expenses (W3) (359)
Profit before tax 759
Income tax expense (120)
Profit for the period 639
Statement of financial position as at 31 December 2015
Assets Rs. in
million
Non-current assets
Property, plant and equipment (W1) 368
Intangible assets (W2) 40
408
Current assets
Inventories (180 + 140) 320
Trade and other receivables (420 x 95%) 399
Cash 440
1,159
Total assets 1,567
Equity and liabilities
Equity
Share capital 600
Other reserves 120
Retaind earnings 635
1,355
Current liabilities
Trade and other payables 92
Current tax payable 120
212
Total equity and liabilities 1,567
Statement of changes in equity for the year ended 31 December 2015
Amounts in Rs. million
Share Revaluati Retained
capital on earnings Total
reserve
Balance at 31 December 2014 620 121 721
Dividends paid (125) (125)
Net revaluation surplus in the year (360 – (300 – 120 - 120
60))
Profit after tax for the period - - 639 639
Balance at 31 December 2015 620 120 635 1,355

© Emile Woolf International 87 The Institute of Chartered Accountants of Pakistan


Financial accounting and reporting II

Workings
(1) Property, plant and equipment
Rs. in
million
Cost brought forward
Leasehold 300
Computers 50
Revaluation 60
Cost carried forward 410
Accumulated depreciation brought forward (60 + 20) 80
Revaluation (60)
Charge for the year
Leasehold (360 ÷ 30) 12
Computers (50 ÷ 5) 10
Accumulated depreciation carried forward 42
Carrying amount carried forward 368
(2) Intangible assets
Rs. in
million
Cost 60
Amortisation (60 ÷ 3) (20)
Carried forward 40
(3) Allocation of costs
Amounts in Rs. million
Change in Depreciati Other
Staff costs
inventories on etc expenses
Work-in-progress (140 – 125) (15)
Staff costs 260
Finished goods (180 – 155) (25)
Consultancy fees 44
Directors’ salaries 360
Doubtful receivables (420 u 5%) 21
Sundry 294
Amortisation of patent (W2) 20
Depreciation (12 + 10) (W1) 22
(40) 620 42 359

© Emile Woolf International 88 The Institute of Chartered Accountants of Pakistan


Answers

2.3 BARRY
Barry
Statement of profit or loss
For the year ended 31st August 2015
Rs. in
million
Revenue 30,000
Cost of sales (W1) (19,650)
Gross profit 10,350
Distribution costs (W1) (1,370)
Administrative expenses (W1) (1,930)
Profit from operations 7,050
Finance costs (350)
Profit before tax 6,700
Tax (W2) (2,500)
Profit after tax 4,200
Barry
Statement of financial position
As at 31st August 2015
Rs. in
million
ASSETS
Non-current assets
Property, plant and equipment 39,600
Current assets
Inventory 4,600
Trade and other receivables (7,400 + 200) 7,600
Cash and cash equivalents 700
12,900
Total assets 52,500
EQUITY AND LIABILITIES
Capital and reserves
Equity shares 21,000
Share premium 2,000
Accumulated profits (W3) 11,800
Total equity 34,800
Revaluation reserve (W4) 4,700
Non current liabilities
Borrowings 5,200
Current liabilities
Trade and other payables 5,300
Taxation (2,100 + 400) 2,500
7,800
Total equity and liabilities 52,500

© Emile Woolf International 89 The Institute of Chartered Accountants of Pakistan


Financial accounting and reporting II

Reconciliation of opening and closing property, plant and equipment


Rs. in ‘000
Assets
Plant & Fixtures
under
Land Buildings machine & Total
construct
ry fittings
ion
Cost/ Valuation
At 1 Sept 2014 10,000 9,000 20,100 10,000 400 49,500
Additions - - - - 50 50
Reclassification - - 450 - (450) -
Revaluation 1,000 1,000 - - - 2,000
At 31 Aug 2015 11,000 10,000 20,550 10,000 - 51,550
Depreciation
At 1 Sept 2014 - 3,000 4,000 3,700 - 10,700
Revaluation - (3,000) - - - (3,000)
Charge for year - 1,000 2,550 700 - 4,250
At 31 Aug 2015 - 1,000 6,550 4,400 - 11,950
Net book value
At 31 Aug 2015 11,000 9,000 14,000 5,600 - 39,600
At 1 Sept 2014 10,000 6,000 16,100 6,300 400 38,800
Workings
1 Allocation of expenses Rs.in ‘000
Cost of Admin Distrib
sales
Raw materials consumed 9,500
Manufacturing overheads 5,000
Increase in inventories (1,400)
Staff costs (70%/20%/10%) 3,290 940 470
Distribution costs 900
Depreciation
Building (50%/50%) 500 500
Plant and machinery 2,550
Fixtures and fittings (30%/70%) 210 490
19,650 1,930 1,370
2 Tax charge
Rs. in ‘000
Current year 2,100
Under provision from previous year 400
2,500

© Emile Woolf International 90 The Institute of Chartered Accountants of Pakistan


Answers

3 Accumulated profits carried forward


Rs. in ‘000
Accumulated profits carried forward per question 14,000
Less tax charge
- Current year estimate 2,100
- Under-provision in previous year 400
(2,500)
Add transfer of excess depreciation on revalued building 300
11,800
4 Revaluation reserve carried forward
Revaluation reserve per question 5,000
Add transfer of excess depreciation on revalued building (300)
4,700

2.4 OSCAR INC


(a) Statement of profit or loss
For the year ended 31 March 2015
Rs. in ‘000

Sales 2,010
Operating costs (140 + 960 – 150 + 420 + 210 + 16) (1,596)
————
Operating profit before interest 414
Income from investments 75
————
Profit before taxation 489
Income tax (49)
————
440
————
Statement of financial position
As at 31 March 2015
Assets
Non-current assets
Tangible assets 530
Investments 560
————
1,090
Current assets
Inventory 150
Receivables 470
————
620
————
1,710
————

© Emile Woolf International 91 The Institute of Chartered Accountants of Pakistan


Financial accounting and reporting II

Equity and liabilities


Capital and reserves
Share capital 600
Retained earnings 500
————
1,100
Current liabilities 414
Provisions for liabilities and charges 196
————
1,710
————
Workings

(1) Income tax


Rs. in ‘000

Income tax (current year) 74


Over provision for tax in the previous year (25)
——
49
——
(2) Tangible assets – plant and machinery
Rs. in ‘000

Cost at 1 April 2014 and 31 March 2015 750


——
Accumulated depreciation
At 31 March 2014 188
Provided during the year (27 + 5) 32
——
At 31 March 2015 220
——

Net book value at 31 March 2015 530


——

(3) Current liabilities


Rs. in ‘000

Trade payables 260


Mainstream corporation tax 74
Bank overdraft 80
——
414
——

© Emile Woolf International 92 The Institute of Chartered Accountants of Pakistan


Answers

(4) Provisions for liabilities and charges


Rs. in ‘000
At 1 April 2014 180
Provided in the year 16
——
At 31 March 2015 196
——

(5) Retained earnings


Rs. in ‘000
Retained earnings 440
Opening retained earnings 180
Dividends (120)
———
Closing retained earnings 500
———

2.5 CLIFTON PHARMA LIMITED


(a) Clifton Pharma Limited
Statement of profit or loss for the year ended 30 September 2015
Rs. in ‘000
Revenue 338,300
Cost of sales: see working (1) (180,000)
Gross profit 158,300
Operating expenses: see working (2) (36,600)
Investment income 2,000
Finance costs: Loan notes – see working (3) (3,000)
Finance lease – see working (2) (1,700)
(4,700)
Profit before tax 119,000
Income tax expense: see working (4) (21,000)
Profit for the period 98,000
(b) Clifton Pharma Limited
Statement of financial position as at 30 September 2015
Non-current assets
Property, plant and equipment: see working (5) 358,000
Investments 92,400
450,400
Current assets
Inventory 23,700
Trade receivables 76,400
Bank 12,100
112,200
Total assets 562,600

© Emile Woolf International 93 The Institute of Chartered Accountants of Pakistan


Financial accounting and reporting II

Equity and liabilities


Capital and reserves
Share capital 280,000
Share premium 20,000
Retained earnings: see working (6) 117,300
417,300
Revaluation surplus 20,000
Non-current liabilities
3% loan notes: see working (3) 51,500
Deferred tax: see working (4) 23,000
Finance lease obligation: see working (2) 11,700
86,200
Current liabilities
Trade payables 14,100
Accrued lease finance costs: see working (2) 1,700
Finance lease obligation: see working (2) 5,300
Income tax payable 18,000
39,100
Total equity and liabilities 562,600
Workings
(1) Cost of sales Rs. in ‘000
As given in the trial balance 134,000
Depreciation of plant and equipment: 20% u (197,000 – 30,000
47,000)
Depreciation of leased vehicles: 24,000/4 years 6,000
Amortisation of leasehold property: 250,000/25 years 10,000
180,000

(2) Vehicle rentals and finance lease. Operating expenses


Rental costs given in the trial balance 8,600
Relating to finance lease (7,000)
Balance: relating to operating lease – operating expense 1,600
Other operating expenses (trial balance in question) 35,000
Total operating expenses 36,600
Finance lease
Fair value of leased assets 24,000
Less: First rental payment, paid in advance (7,000)
1 October 2014
Remaining obligation, 1 October 2014 17,000
Interest at 10% to 30 September 2015 (current liability) 1,700
Lease payment due 1 October 2015 7,000
Capital repayment due (= balance, current liability) (5,300)
Remaining lease obligation = non-current liability 11,700

© Emile Woolf International 94 The Institute of Chartered Accountants of Pakistan


Answers

(3) Loan notes


The effective interest rate is 6%. Actual interest paid was Rs.1,500,000 (in trial
balance); therefore the balancing Rs.1,500,000 should be added to the loan notes
obligation, to make the total loan notes liability Rs.50 million + Rs.1,500,000 = Rs.51.5
million.
(4) Taxation
Deferred tax liability b/f 20,000
Deferred tax: credit in the statement of profit or loss 2,000
Deferred tax liability c/f (92,000 u 25%) 23,000
Tax expense
Income tax on profits for the year 18,000
Deferred tax movement 3,000
Tax charge in the statement of profit or loss 21,000
(5) Non-current assets and depreciation
Leasehold property Rs. in ‘000
Carrying value in the trial balance (250,000 – 40,000) 210,000
Amortisation charge for the year to 30 September 2015 (10,000)
200,000
Re-valued amount 220,000
Transfer to revaluation reserve 20,000
The annual depreciation charges for plant and equipment and the leased vehicles are
shown in workings (1) Rs. in ’000
Cost or Accumulated Carrying
valuation depreciation amount
Leasehold property 220,000 0 220,000
Plant and equipment 197,000 77,000 120,000
(non-leased)
Leased vehicles 24,000 6,000 18,000
441,000 83,000 358,000
(6) Retained profits
At 1 October 2014 (trial balance) 19,300
Profit for the year 98,000
Retained profits at 30 September 2015 117,300

2.6 SARHAD SUGAR LIMITED


(a) Sarhad Sugar Limited –
Statement of profit or loss for the year ended 30 September 2015
Revenue (300,000 – 2,500) 297,500
Cost of sales (w (i)) (225,400)
Gross profit 72,100
Distribution costs (14,500)
Administrative expenses (22,200 – 400 + 100 see note below) (21,900)
Finance costs (1,000)
Profit before tax 34,700
(Income tax expense (11,400 + (6,000 – 5,800 deferred tax)) (11,600)
Profit for the year 23,100

© Emile Woolf International 95 The Institute of Chartered Accountants of Pakistan


Financial accounting and reporting II

(b) Sarhad Sugar Limited


Statement of financial position as at 30 September 2015

Assets
Non-current assets (w (ii))
Property, plant and equipment (43,000 + 38,400) 81,400
Development costs 14,800
96,200
Current assets
Inventory 20,000
Trade receivables 43,100
63,100
Total assets 159,300
Equity and liabilities:
Equity
Share capital 70,000
Retained earnings (w (iii)) 41,600
117,100
Revaluation reserve (w (iii)) 5,500
Non-current liabilities
Deferred tax 6,000
Current liabilities
Trade payables (23,800 – 400 + 100 – re legal action) 23,500
Bank overdraft 1,300
Current tax payable 11,400
36,200
Total equity and liabilities 159,300

Note: As it is considered that the outcome of the legal action against Sarhad Sugar Limited is
unlikely to succeed (only a 20% chance) it is inappropriate to provide for any damages. The
potential damages are an example of a contingent liability which should be disclosed (at Rs.2
million) as a note to the financial statements. The unrecoverable legal costs are a liability (the
start of the legal action is a past event) and should be provided for in full.
Workings (figures in brackets in Rs.000)
(i) Cost of sales: Rs. in ‘000
Per trial balance 204,000
Depreciation (w (iii)) – leasehold property 2,500
– plant and equipment 9,600
Loss on disposal of plant (4,000 – 2,500) 1,500
Amortisation of development costs (w (iii)) 4,000
Research and development expensed (1,400 + 2,400 (w (iii)) 3,800
––––––––
225,400


© Emile Woolf International 96 The Institute of Chartered Accountants of Pakistan


Answers

(ii) Non-current assets: Rs. in ‘000


Leasehold property
Valuation at 1 October 2014 50,000
Depreciation for year (20 year life) (2,500)
––––––––
Carrying amount at date of revaluation 47,500
Valuation at 30 September 2015 (43,000)
––––––––
Revaluation deficit 4,500


Plant and equipment per trial balance (76,600 – 24,600) 52,000


Disposal (8,000 – 4,000) (4,000)
––––––––
48,000
Depreciation for year (20%) (9,600)
––––––––
Carrying amount at 30 September 2015 38,400

Capitalised/deferred development costs
Carrying amount at 1 October 2014 (20,000 – 6,000) 14,000
Amortised for year (20,000 x 20%) (4,000)
Capitalised during year (800 x 6 months) 4,800
––––––––
Carrying amount at 30 September 2015 14,800

Note: development costs can only be treated as an asset from the point where they
meet the recognition criteria in IAS 38 Intangible assets. Thus development costs from
1 April to 30 September 2015 of Rs.4·8 million (800 x 6 months) can be capitalised.
These will not be amortised as the project is still in development.
The research costs of Rs.1·4 million plus three months’ development costs of Rs.2·4
million (800 x 3 months) (i.e. those incurred before 1 April 2015) are treated as an
expense.
(iii) Movements on reserves
Revaluation Retained
surplus earnings
Rs. in ‘000
Balances at 1 October 2014 10,000 24,500
Dividend (6,000)
Comprehensive income 23,100
Revaluation loss (4,500)
Balances at 30 September 2015 5,500 41,600

© Emile Woolf International 97 The Institute of Chartered Accountants of Pakistan


Financial accounting and reporting II

2.7 BSZ LIMITED


Statement of financial position as at June 30, 2017
Note Rs. in
million
ASSETS
Fixed Assets
Property, plant & equipment 1 576
Intangible assets 2 8
584

Long term advances – considered good 4


Current assets
Stocks in trade 90
Accounts receivable 3 57
Advances, deposits, prepayments and other receivables 4 45
Cash at banks 5 29
221
809
EQUITY AND LIABILITIES
Share capital and reserves
Authorized share capital
50,000,000 shares of Rs. 10 each 500

Issued, subscribed and paid up capital


40,000,000 shares of Rs. 10 each 400
Unappropriated profit 65
465
Surplus on revaluation of fixed assets 120
Non-current liabilities
Deferred taxation 40

Current liabilities
Short term loan 85
Account and other payables 6 82
Provision for taxation 17
184
809

© Emile Woolf International 98 The Institute of Chartered Accountants of Pakistan


Answers

Rs. in
Notes million
1. Property, plant and equipment
Operating assets 556
Capital work in progress – building 20
576

1.1 Operating assets Rs. in million


Freehold
Cost/revalued amount land Building Machines Fixtures Total
As of July 01 2016 375.0 130.0 100.0 19.0 624.0
Additions - - - 8.0 8.0
Disposals - - (15.0) - (15.0)
As at June 30 2017 375.0 130.0 85.0 27.0 617.0

Freehold
land Building Machines Fixtures Total
Accumulated depreciation
As of July 01 2016 - 19.5 22.5 5.9 47.9
For the year - 6.5 18.1
8
(105 × 85) + 10% × 15 × /12) 9.5
3
(105 × 19) + 10% × 8 × /12) 2.1
Disposals - - (5.0) - (5.0)
As at June 30 2017 - 26.0 27.0 8.0 61.0
Carrying amount 375.0 104.0 58.0 19.0 556.0
Depreciation rate - 5% 10% 10%

1.2 Revaluation
During the year 2013, the first revaluation of freehold land was carried out. The valuation
was carried out under market value basis by an independent valuer, Mr. Dee, Chartered
Civil Engineer of M/s SSS Consultants (Pvt.) Ltd., Islamabad. It resulted in a surplus of Rs.
120 million over book values which was credited to surplus on revaluation of fixed assets.
Had there been no revaluation, the value of freehold land would be Rs. 255 million.
1.3 Disposal of machine
Rs. in
million
Proceeds 13.0
Cost 15.0
Accumulated depreciation (5.0)
Carrying amount (10.0)
Profit on disposal 3.0

© Emile Woolf International 99 The Institute of Chartered Accountants of Pakistan


Financial accounting and reporting II

Note 2017
Rs. in
million
2. Intangible Assets
Cost of computer software/license 10.0
Accumulated Amortization as of July 1, 2016 1.0
Amortization for the year 1.0
Accumulated Amortization as of June 30, 2016 2.0
Carrying value as at June 30, 2017 8.0
Amortization rate 10%

3. Accounts Receivable
Considered good
- Secured 30
- Unsecured 27
57
Considered doubtful 3
60
Less: Provision for bad debts 3.1 3
57

3.1 Provision for bad debts


Balance as at July 1, 2016 3.4
Provision made during the year 1.0
Amount written off during the year (1.4)
Balance as at June 30, 2017 (Rs. 30 million x 10%) 3.0

4 Advances, Deposits, Prepayments and Other Receivables


Advances
- suppliers - considered good 12
- staffs 6
18
Deposits 11
Prepayments 4
Sales tax receivable 12
45
5 Cash at banks
Cash at banks - current accounts 7
saving accounts 5.1 22
29

© Emile Woolf International 100 The Institute of Chartered Accountants of Pakistan


Answers

5.1: It carries interest / mark up ranging from 3% to 7% per annum.


Note 2017
Rs. in
million
6 Accounts and other payables
Accounts payable 75
Accrued liabilities 7
82

2.8 YASIR INDUSTRIES LIMITED


Statement of Financial Position as at June 30, 2017

Rs. in
Assets million

Non-current assets
Property, plant and equipment (W2)
351.00
Intangible assets (20 – 12)
8.00
359.00
Current assets
Inventories (W6)
64.50
Trade receivables (W5)
39.00
103.50
462.50
Equity and Liabilities
Equity
Issued, subscribed and paid up capital
120.00
Retained earnings (W4)
87.10
207.10
Revaluation surplus 41.25
Non-current liabilities
Redeemable preference shares
40.00
Debentures
80.00
Deferred taxation (W 10)
9.00
129.00
Current liabilities
Trade payables
30.40
Accrued expenses (W3)
25.00
Taxation
16.50
Bank overdraft
13.25
85.15
Total equity and liabilities 462.50

© Emile Woolf International 101 The Institute of Chartered Accountants of Pakistan


Financial accounting and reporting II

Statement of profit or loss for the year ended June 30, 2017
Rs. in
million
Sales revenue (W5) 445.40
Cost of sales (W7) (250.72)
Gross profit 194.68
Distribution costs (W8) (20.05)
Administrative expenses (W8) (40.38)
Financial charges (W9) (9.10)
125.15
Loss due to fraud (30.00)
Profit before tax 95.15
Income tax expense (W10) (19.50)
Profit for the year 75.65
Workings
(W1) Leasehold property

Annual depreciation before the revaluation (230 ÷ 40 years) = Rs. 5.75 million per annum.
Depreciation this year has been charged incorrectly on cost (whereas it should have been
on the revalued amount).
This year’s charge must be added back
Dr Cr
Accumulated depreciation 5.75
Cost of sales (50%) 2.88
Administrative expenses (30%) 1.72
Distribution costs (20%) 1.15

Rs. in
million
Carrying amount at the 30 June (as per trial balance)(230.00 – 40.25) 189.75
Add back depreciation incorrectly charged (see above) 5.75
Carrying amount of property at the start of the year 195.5

Rs. in
Revaluation surplus million
Revalued amount of leasehold property 238.00
Less: WDV of leasehold property at revaluation 195.50
Revaluation surplus arising in the year 42.50
Transfer to retained earnings in respect of incremental depreciation (Rs. 7
million – Rs. 5.75 million) (1.25)
41.25

© Emile Woolf International 102 The Institute of Chartered Accountants of Pakistan


Answers

Depreciation of revalued property

Number of years depreciation by the year end: (40.25 ÷ 5.75) = 7 years.


Therefore, remaining useful life as at the year-end = 33 years
Revaluation was at the start of the year
Remaining useful life at the start of the year = 34 years
Depreciation charge based on the revalued amount (238/34 years) = Rs. 7 million
Dr Cr
Cost of sales (50%) 3.5
Administrative expenses (30%) 2.1
Distribution costs (20%) 1.4
Accumulated depreciation 7.00
(W2) Property, plant and equipment
Rs. in
million
Leasehold property (Rs. 238m – 7) 231
Machines (Rs. 168.6 – Rs. 48.6m) 120
351
(W3) Accrued Expenses
As per trial balance 15.00
6
Accrued interest on debentures (Rs. 80m × 12% × /12) 4.80
Dividend on preference shares (Rs. 40m × 10%) 4.00
23.80
(W4) Retained earnings
Balance as per trial balance 10.20
Profit for the year 75.65
Transfer from revalution surplus 1.25
87.10
(W5) Sales and receivables
Sales. Rec.
Rs. in Rs. m
million
Given in the trial balance 478.40 66.00
Deduct revenue incorrectly recognised (sale or return) (27.00) (27.00)
Cost of sales 451.40 39.00
(W6) Closing inventory
Rs. in
million
Given in the question 42.00
100
Add back inventory held by customer on sale or return ( /120 u 27) 22.50
Cost of sales 64.50

© Emile Woolf International 103 The Institute of Chartered Accountants of Pakistan


Financial accounting and reporting II

(W7) Cost of sales


Rs. in
million
Opening inventory as of July 1, 2016 38.90
Purchases 175.70
Direct labour 61.00
Manufacturing overheads excluding incremental depreciation 39.00
Less: Closing inventory (64.50)
Deduct depreciation incorrectly charged on cost (2.88)
Add depreciation charged on revalued amount 3.50
Cost of sales 250.10
(W8) Administrative expenses and distribution costs
Admin. DIst.
Rs. in Rs. m
million
Given in the trial balance 40.00 19.80
Deduct depreciation incorrectly charged on cost (1.72) (1.15)
Add depreciation charged on revalued amount 2.10 1.40
Cost of sales 40.38 20.05
(W9) Financial charges
Rs. in
million
Balance as per trial balance 0.30
Accrued interest on debentures (Rs. 80m × 12% × 6/12) 4.80
Preference dividend for the year (Rs. 40m × 10%) 4.00
9.10
(W10) Taxation
Deferred taxation Rs. in
million
Balance b/f 6.00
Charge for the year (balancing figure) 3.00
Balance c/f (30% u Rs. 30 million temporary difference) 9.00
Tax expense
Current tax 16.50
Deferred tax (see above) 3.00
19.50

© Emile Woolf International 104 The Institute of Chartered Accountants of Pakistan


Answers

2.9 SHAHEEN LIMITED


Shaheen Limited
Statement of financial position
As of June 30, 2017
Assets Rs. in ‘000
Non-current assets
Property, plant and equipment (86,000  12,000  4,500) 69,500
Intangible assets (6,000  600) 5,400
74,900
Current assets
Stock in trade 30,000
Trade receivables (37,800  10,000) 27,800
Other receivables and prepayments (14,000 + 6,000) 20,000
Cash and bank balances 4,725
82,525
157,425
Equity and liabilities
Share capital and reserves
issued, subscribed and paid up capital 60,000
Unappropriated profit 35,372
95,372
Non-current liabilities
Long term borrowings (31,525  6,000) 25,525
Deferred taxation (5,000  1,470) 3,530
29,055
Current liabilities
Trade payables 12,000
Current portion of long term borrowings 6,000
Provision for litigation 5,000
Provision for taxation (2,000 + 9,988  2,000) 9,998
32,998
157,425
Shaheen Limited
Statement of profit or loss and other comprehensive income
As of June 30, 2017 Rs. in ‘000
Sales revenue 200,000
Cost of sales (W2) (104,708)
Gross profit 95,292

© Emile Woolf International 105 The Institute of Chartered Accountants of Pakistan


Financial accounting and reporting II

Selling and distribution expenses (W2) (36,275)


Administrative expenses (W2) (30,450)
(66,725)
Financial charges (5,000)
Profit before taxation 23,567
Taxation (W3) (6,528)
Profit after taxation 17,039
Other comprehensive income – net of tax -
Total comprehensive income 17,039

Shaheen Limited
Statement of changes in equity 2017
As of June 30, 2017 Rs.000
Issued,
Retained
subscribed & paid
earnings
up capital
Balance July 1, 2016 60,000 32,000*
Correction of prior year error (10,000 u 20/120) (1,667)
Balance July 1, 2016 (restated) 60,000 30,333
Comprehensive income for the year 17,039
Dividend for the year ended June 30, 2016
(60,000*0.20) (12,000)
Balance June 30, 2017 60,000 35,372
*Retained earnings as at 01-07-11 = 20,000+ (20% of 60,000)=32,000
Workings
W1 Depreciation for the year
On building (36,000/20) 1,800
On plant and equipment (30,000  3,000)/10 2,700
Total 4,500
Selling and
Cost of Administrative
distribution
W2 Costs sales costs
costs
Opening inventory 23,000
Costs as per Trial balance 100,000 35,000 30,000
Closing inventory (30,000)
Depreciation (75%, 15%, and 10% of Rs.
4,500) 3,375 675 450
Adjustment for goods sent on sale or return,
erroneously booked as sales last year now
returned during the year. (10,000/1.2) 8,333
Amortization of export license (6,000/5*0.5) 600
104,708 36,275 30,450

© Emile Woolf International 106 The Institute of Chartered Accountants of Pakistan


Answers

W3:Taxation
Profit before tax 23,567
Disallowances and add backs 5,000
Taxable income 28,567
Current For the year (28,567*0.35) 9,998
For prior years (7,000  5,000) (2,000)
Deferred For the year (5,000  800)*0.35 (1,470)
6,528

2.10 MOONLIGHT PAKISTAN LIMITED


(a) Moonlight Pakistan Limited
Statement of Financial Position
As at December 31, 2017
Rs. in
million
ASSETS
Non-current assets
Property, plant and equipment (W2) 3,472
Current assets
Stocks in trade 758
Trade receivables 702
Cash and bank 354
1,814
5,286
EQUITY
Issued, subscribed and paid-up capital (W3) 1,750
Share premium (420 x 2/12) 70
Retained earnings (W3) 876
2,696
Surplus on revaluation of fixed assets 240
LIABILITIES
Non-current liabilities
Long term loan 1,600
Deferred tax (22 + 80 x 35%) 50
Provision for gratuity 23
1,673
Current liabilities
Creditor and other liabilities (544 + 96) 640
Income tax payable 37
677
5,286

© Emile Woolf International 107 The Institute of Chartered Accountants of Pakistan


Financial accounting and reporting II

(b) Moonlight Pakistan Limited


Statement of profit or loss
For the year ended December 31, 2017
Rs. in
million
Sales 3,608
Cost of sales (W1) (2,149)
Gross profit 1,459
Selling expenses (W1) 252
Administrative expenses (W1) 270
522
937
Financial charges (210 + 1,600 x 12% x 6/12) 306
Profit before taxation 631
Taxation (37 + 80 x 35%) 65
Profit after taxation 566

W1: Cost of sales/selling expenses/admin expenses


Cost of Selling Admin.
sales expenses expenses
Rs. in million
As per trial balance 1,784 220 250
Depreciation – building (60% : 25% : 15%) (W2) 69 29 17
Depreciation – plant 287 - -
Provision for gratuity (23-8) x 60%:20%:20% 9 3 3
2,149 252 270

W2: Property, plant and equipment


Land Building Plant Total
Rs. in million

Cost as at January 1, 2017 600 2,000 2,104 4,704


Accumulated depreciation - (400) (670) (1,070)

Revaluation (1,840 - (2,000 - 400 )) - 240 - 240


Current year depreciation - (287) (402)
(1,840/16) (115)
600 1,725 1,147 3,472

© Emile Woolf International 108 The Institute of Chartered Accountants of Pakistan


Answers

W3: Share Capital/Retained Earnings


Retained
Share capital
earnings
Rs. in million
As per trial balance 1,200 510
Bonus issue (1200 ÷ 6) 200 (200)
Right issue (420 x 10/12) 350 -
Profit for the year - 566
1,750 876

2.11 FIGS PAKISTAN LIMITED


Figs Pakistan Limited
Statement of profit or loss and other comprehensive income
For the year ended 31 December 2017
2017
Rs. in
Note million
Sales 1 44,758
Cost of sales 2 (26,203)
Gross profit 18,555
Distribution costs 3 (6,431)
Administrative expenses 4 (752)
Other operating expenses 5 (399)
Other operating income 6 30
Profit from operations 11,003
Finance costs 7 (166)
Profit before tax 10,837
Taxation 8 (2,532)
Profit after tax 8,305
Other comprehensive income -
Total comprehensive income for the year 8,305

Figs Pakistan Limited


Notes to the financial statements
For the year ended 31 December 2017
1 Sales
Manufactured goods
Gross sales 56,528
Sales tax (10,201)
46,327
Imported goods
Gross sales 1,078
Sales tax (53)
1,025
Sales discounts (2,594)
44,758

© Emile Woolf International 109 The Institute of Chartered Accountants of Pakistan


Financial accounting and reporting II

2 Cost of sales
Raw material consumed (1,751 + 22,603 - 2,125) 22,229
Stores and spares consumed 180
Salaries, wages and benefits (2,367 × 55%) 2.1 1,302
Utilities (734 × 85%) 624
Depreciation and amortizations (1.287 × 70%) 901
Stationery and office expenses (230 × 25%) 58
Repairs and maintenance (315 × 85%) 268
25,562
Opening work in process 73
Closing work in process (125)
25,510
Opening finished goods (manufactured) 1,210
Closing finished goods (manufactured) (1,153)
25,567
Finished goods (imported)
Opening stock 44
Purchases 658
702
Closing stock (66)
636
26,203

2.1 Salaries, wages and benefits include Rs. 30 million (54 × 55%) and Rs. 24 million (44 × 55%)
in respect of defined contribution plan and defined benefit plan respectively.

Rs. in
3 Distribution costs million
Advertisement and sales promotion 4,040
Outward freight and handling 1,279
Salaries, wages and benefits (2,367 × 30%) 3.2 710
Utilities (734 × 5%) 37
Depreciation and amortization (1,287 × 20%) 257
Stationery and office expenses (230 × 40%) 92
Repairs and maintenance (315 × 5%) 16
6,431

3.1 Salaries, wages and benefits include Rs. 16 million (54 × 30%) and Rs. 13 million (44×30%) in
respect of defined contribution plan and defined benefit plan respectively.

© Emile Woolf International 110 The Institute of Chartered Accountants of Pakistan


Answers

Rs. in
4 Administrative expenses million
Salaries, wages and benefits (2,367 × 15%) 4.1 355
Utilities (734 × 10%) 73
Depreciation and amortization (1,287 × 10%) 129
Stationery and office expenses (230 × 35%) 80
Repairs and maintenance (315 × 10%) 31
Legal and professional charges 71
Auditor's remuneration 4.2 13
752

4.1 Salaries, wages and benefits include Rs. 8 million (54 × 15%) and Rs. 7 million (44×15%) in
respect of defined contribution plan and defined benefit plan respectively.

Rs. in
4.2 Auditor's remuneration million
Audit fees 8
Taxation services 4
Out of pocket expenses 1
13
5 Other operating expenses
Donation 5.1 34
Worker's Profit Participation Fund 257
Worker Welfare Fund 98
Loss on disposal of property, plant and equipment 10
399

5.1 Donations
Donations include Rs. 5 million given to Dates Cancer Foundation (DCF). One of the
company’s directors, Mr. Peanut is a trustee of DCH.
Donations other than that mentioned above were not made to any donee in which a director or
his spouse had any interest at any time during the year.
Rs. in
6 Other operating income million
Income from financial assets
Dividend income 12
Return on savings account 2
Income from non-financial assets
Scrap sales 16
30
7 Finance costs
Finance charges on short term borrowings 133
Exchange loss 22
Finance charges on lease 11
166

© Emile Woolf International 111 The Institute of Chartered Accountants of Pakistan


Financial accounting and reporting II

Rs. in
million
8 Taxation
Current - for the year 1,440
Deferred (3,120 × 35%) 1,092
2,532

© Emile Woolf International 112 The Institute of Chartered Accountants of Pakistan


Answers

CHAPTER 3 – IAS 7: STATEMENTS OF CASH FLOWS

3.1 KLEA
Statement of cash flows for the year ended 31st March 2015
Rs. in ‘000
Cash flows from operating activities
Profit before taxation 1,606
Adjustments for:
Depreciation (W4) 800
Finance income (50)
Interest expense 320
––––––
2,676
Increase in trade receivables (400)
Increase in inventories (1,200)
Increase in trade payables 334
––––––
Cash generated from operations 1,410
Interest paid (320)
Income taxes paid (W1) (630)
––––––
Net cash from operating activities 460
Cash flows from investing activities
Purchase of intangible assets (W2) (300)
Purchase of property, plant and equipment (W3) (1,600)
Proceeds from sale of equipment 150
Purchase of long-term investments (200)
Finance income received 50
––––––
Net cash used in investing activities (1,900)

Cash flows from financing activities


Proceeds from issue of share capital (1,000 + 278) 1,278
Payments to redeem debentures (400)
Dividends paid (400)
––––––
Net cash used in financing activities 478
–––––
Net decrease in cash and cash equivalents (962)
Cash and cash equivalents at 1 April 2014 580
–––––
Cash and cash equivalents at 31 March 2015 (32 - 414) (382)

© Emile Woolf International 113 The Institute of Chartered Accountants of Pakistan


Financial accounting and reporting II

(Note: Alternative classifications of the cash flows in accordance with IAS 7 should receive full credit
– i.e. interest and dividends received as investing activities or operating cash flows, interest and
dividends paid as financing or operating cash flows.)
Notes
(1) Analysis of cash and cash equivalents Rs. in ‘000

2015 2014
Cash on hand and balances with bank 32 580
Bank overdraft (414) -
–––– ––––
Cash and cash equivalents (382) 580
–––– ––––

(2) Material non-cash transactions


During the year land was re-valued upwards by Rs.1million
Workings Rs. in ‘000

(W1) Taxation paid


Taxation creditor brought forward 400
Taxation expense for period 650
––––––
1,050
Taxation creditor carried forward (420)
––––––
Taxation paid in the year 630
––––––

(W2) Intangible assets


Net book value brought forward 200
Capitalised in the year (from (i)) 300
––––
500
Amortisation charged in year (from (i)) (200)
––––
Intangibles acquired in the year 300
––––

(W3) Property, plant and equipment


Cost brought forward 3,000
Revaluation in year (from (ii)) 1,000
Disposals (from (iii)) (600)
Additions (balancing figure) 1,600
––––––
Cost carried forward 5,000
––––––

© Emile Woolf International 114 The Institute of Chartered Accountants of Pakistan


Answers

(W4) Depreciation and amortisation


Depreciation (150 movement + 500 on disposal) 650
Amortisation 200
Profit on disposal (W5) (50)
––––
Charge shown in statement of profit or loss 800
––––
Hence add back of depreciation and amortisation also takes account of the profit on disposal
of the plant and machinery.
(W5) Disposal
Cost of disposal 600
Accumulated depreciation (500)
––––
Net book value 100
Proceeds of sale 150
––––
Profit on sale 50
––––

3.2 STANDARD INC


Statement of cash flows for the year ended 31 December 2015
Rs. in ‘000 Rs. in ‘000
Cash flows from operating activities
Net profit before tax (W7) 64,000
Adjustments for:
Depreciation, loss on sale (W1-5) 20,000
Interest receivable (450)
Interest and premium payable 8,400
————
Operating profit 91,950

Increase in inventories (14,000)


Increase in receivables (1,200)
Increase in payables 14,440
————
Cash generated from operations 91,190
Interest paid (6,840)
Tax paid (W6) (10,500)
————
Net cash from operating activities 73,850
Cash flows from investing activities

Acquisition of long-term investment (4,600)


Purchase of property plant and equipment (69,000)
Receipt from sale of long-term investment 4,000
Interest received 450
————
Net cash used in investing activities (69,150)

© Emile Woolf International 115 The Institute of Chartered Accountants of Pakistan


Financial accounting and reporting II

Cash flows from financing activities

Proceeds from issuance of shares 70,000


Redemption of long term loan (42,000)
Dividends paid (7,500)
————
Net cash used in financing activities 20,500
————
Net increase in cash and cash equivalents 25,200
————
WORKINGS
(1) Plant and machinery account – at cost

Rs.000 Rs.000
Balance b/d 120,000 Disposals account 8,000
Additions 39,000 Balance c/d 151,000
———— ————
159,000 159,000
———— ————

(2) Fixtures and fittings account – at cost

Rs.000 Rs.000
Balance b/d 24,000 Disposals account 5,000
Additions 10,000 Balance c/d 29,000
——— ———
34,000 34,000
——— ———

Fixed assets – additions summary Rs.000


Freehold property Rs.000(130,000 - 110,000) 20,000
Plant and machinery 39,000
Fixtures and fittings 10,000
———
69,000
———
(3) Plant and machinery account – depreciation

Rs.000 Rs.000
Disposals account 6,000 Balance b/d 45,000
Balance c/d 54,000 Charge for year 15,000
——— ———
60,000 60,000
——— ———

© Emile Woolf International 116 The Institute of Chartered Accountants of Pakistan


Answers

(4) Fixtures and fittings account – depreciation

Rs.000 Rs.000
Disposals account 2,000 Balance b/d 13,000
Balance c/d 15,000 Charge for year 4,000
——— ———
17,000 17,000
——— ———

(5) Fixed assets disposals account

Rs.000 Rs.000
Plant cost 8,000 Plant depreciation 6,000
Fittings cost 5,000 Fittings depreciation 2,000
Cash proceeds
Plant 3,000
Fittings 1,000
Depreciation underprovided
(bal fig) 1,000
——— ———
13,000 13,000
——— ———

(6) Tax account

Rs.000 Rs.000
Cash paid (bal fig) 10,500 Balance b/f – corporation tax 21,500
Balance c/f – corporation tax 33,000 I&E account – corporation tax 22,000
——— ———
43,500 43,500
——— ———
(7) Net profit before tax
Note As profit before tax is required, reconstruct the statement of profit or loss up to this
figure.
Rs. in ‘000
Profit before tax 64,000
Taxation – Corporation tax (22,000)
———
42,000
Dividends (15,000)
———
Retained profit for year 27,000
Balance b/f 14,000
———
Balance c/f 41,000


© Emile Woolf International 117 The Institute of Chartered Accountants of Pakistan


Financial accounting and reporting II

(8) Cash and cash equivalents as shown in the statement of financial position
Cash and cash equivalents consist of cash on hand and balances with banks.
Rs. in ‘000
2015 2014 Change
in
year
Cash at bank 11,400 200 11,200
Bank overdraft – (14,000) 14,000
——— ——— ———
11,400 (13,800) 25,200
  

3.3 FALLEN
Statement of cash flows for the year ended 31 December 2015
Cash flows from operating activities Rs. in ‘000
Net profit before tax 4,625
Adjustments for:
Depreciation, (W1-3) 1,472
Interest payable 152
———
Operating profit 6,249
Increase in deferred repairs provision 186
Increase in inventories (894)
Increase in receivables (594)
Increase in payables 324
———
Cash generated from operations 5,271
Interest paid (152)
Tax paid (W5) (1,775)
———
Net cash from operating activities 3,344

Cash flows from investing activities

Acquisition of long-term investment (198)


Purchase of property plant and equipment (3,800)
Receipt from sale of long-term investment 168
———
Net cash used in investing activities (3,830)

Cash flows from financing activities

Proceeds from issuance of shares (W6-7) 792


Redemption of long term loan (560)
Dividends paid (544)
——
Net cash used in financing activities (312)
——
Net increase in cash and cash equivalents (798)


© Emile Woolf International 118 The Institute of Chartered Accountants of Pakistan


Answers

WORKINGS Rs. in ‘000


(1) Leasehold premises (net)

Brought forward 5,700 Depreciation (to balance) 400


Additions 1,300 Carried forward 6,600
——— ———
7,000 7,000
 

(2) Plant (net)

Brought forward 3,780 Disposals 276


Additions 2,500 Depreciation (to balance) 964
Carried forward 5,040
——— ———
6,280 6,280
 

(3) Disposals

Plant 276 Cash 168


Loss on sale (to balance) 108
—–— —–—
276 276
 

(4) Dividends

Cash (to balance) 544 Brought forward 234


Carried forward 390 I&E account 700
—–— —–—
934 934
 

(5) Taxation

Cash (to balance) 1,775 Brought forward


Carried forward DT 138
DT 202 CT 2,038
CT 1,730
I&E account 1,531
——— ———
3,707 3,707
 

(6) Share capital

Brought forward 1,800


Carried forward 2,280 Cash (to balance) 480
——— ———
2,280 2,280
 

© Emile Woolf International 119 The Institute of Chartered Accountants of Pakistan


Financial accounting and reporting II

(7) Share premium

Carried forward 2,112 Brought forward 1,800


Cash (to balance) 312
——— ———
2,112 2,112
 

(8) Long term loan

Cash (to balance) 560 Brought forward 1,800


Carried forward 1,240
——— ———
1,800 1,800
——— ———

(9) Analysis of the balances of cash and cash equivalents as shown in the statement of
financial position
Cash and cash equivalents consist of cash on hand and balances with banks.
Rs. in ‘000
2015 2014 Change in
year
Cash at bank and in hand – 576 (576)
Bank overdrafts (222) – (222)
—— —— ——
(222) 576 (798)
  

3.4 BIN QASIM MOTORS LIMITED


Note: figures in brackets are in Rs.000
Bin Qasim Motors Limited
Statement of cash flows for the year to 30 September 2015
Rs.000 Rs.000
Cash flows from operating activities
Net profit before interest and tax (3,198 – 1,479) 1,719
Adjustments for:
Depreciation – buildings (W1) 80
– plant (W1) 276
Loss on disposal of plant (W1) 86 442
442
Amortisation of government grants (W2) (125)
Negligence claim previously provided (120)
Operating profit before working capital changes 1,916
Increase in inventories (1,046 – 785) (261)
Increase in accounts receivable (935 – 824) (111)
Decrease in accounts payable (760 – 644) (116)
Cash generated from operations 1,428

© Emile Woolf International 120 The Institute of Chartered Accountants of Pakistan


Answers

Interest paid (260 + 25 – 40) (245)


Income tax paid (W4) (368)
Dividends paid (180)
Net cash from operating activities 635

Cash flows from investing activities


Purchase of land and buildings (W1) (50)
Purchase of plant (W1) (848)
Purchase of non-current investments (690)
Purchase of treasury bills (120 – 50) (70)
Proceeds of sale of plant (W1) 170
Receipt of cash on servicing contracts (W2) 175
Investment income 120
Net cash used in investing activities (1,193)
Cash flows from financing activities
Issue of ordinary shares (W3) 300
Net decrease in cash and cash equivalents (258)
Cash and cash equivalents at the beginning of the period 122
Cash and cash equivalents at the end of the period (136)

Workings
(W1) Non-current assets

Rs.000
Land and buildings – cost/valuation
Balance b/f 1,800
Revaluation surplus 150
Balance c/f (2,000)
Difference cash purchase (50)

Plant – cost
Balance b/f 1,220
Disposal (500)
Balance c/f (1,568)
Difference cash purchase (848)

Depreciation of non-current assets:


Building (760 – 680) 80
Plant (464 – (432 – 244)) 276

The plant had a carrying value of Rs.256,000 at the date of its disposal (500 cost – 244
depreciation). As there was a loss on sale of Rs.86,000 (given in question), the sale proceeds
must have been Rs.170,000 (i.e. 256 – 86).

© Emile Woolf International 121 The Institute of Chartered Accountants of Pakistan


Financial accounting and reporting II

(W2) Deferred income

Balances b/f – current (125)


– non-current (200)
(325)
Amortisation credited to cost of sales 125
Balances c/f – current 100
– non-current 275
375
Difference cash receipt 175

(W3) Share capital and convertible loan stock


A reconciliation of share capital, share premium and the revaluation reserve shows the shares
issued for cash:

Share Share Revaluation


capital premium reserve
Rs.000 Rs.000 Rs.000
Opening balance (1,000) (60) (40)
Revaluation of land (150)
Bonus issue 1 for 10 (100) 100
Conversion of loan stock (see below) (100) (300)
Closing balance 1,400 460 90
––––– ––––– –––––
Difference issued for cash 200 100 nil
  
The 10% convertible loan stock had a carrying value of Rs.400,000 at the date of conversion
to equity shares. This would be taken as the consideration for the shares issued which would
be 100,000 Rs.1 shares (i.e. 400,000/100 u 25). This would increase issued share capital by
Rs.100,000 and share premium by Rs.300,000.

(W4) Income tax

Rs.000
Tax provision b/f (367)
Deferred tax b/f (400)
Statement of profit or loss tax charge (520)
Tax provision c/f 480
Deferred tax c/f 439
Difference cash paid (368)

© Emile Woolf International 122 The Institute of Chartered Accountants of Pakistan


Answers

3.5 ITTEHAD MANUFACTURING LTD


(a)
Ittehad Manufacturing Ltd
Statement of cash flows for the year to 30 September 2015
Rs.m Rs.m
Cash flows from operating activities
Net profit before interest and tax 920
Adjustments for:
Amortisation – development expenditure (W1) 130
Depreciation – property, plant and equipment 320
Loss on sale of plant 50
Increase in inventory (1,420 – 940) (480)
Increase in accounts receivable (990 – 680) (310)
Increase in accounts payable (875 – 730) 145
Decrease in deferred income (260 – 300) (40)
Cash generated from operations 735
Interest paid (30 – (15 – 5 accrual adjustments)) (20)
Income tax paid (W2) (130)
Net cash from operating activities 585
Cash flows from investing activities
Purchase of property, plant and equipment (W3) (250)
Capitalised development costs (W1) (500)
Proceeds of sale of plant (W3) 20
Net cash from investing activities (730)

Cash flows from financing activities


Issue of ordinary shares (W4) 450
Issue of loan notes (300 – 100) 200
Dividends paid (320)
Net cash generated from financing activities 330
Net increase in cash and cash equivalents 185
Cash and cash equivalents at beginning of period (115)
Cash and cash equivalents at end of period 70

Workings

(W1) Development expenditure

Rs.m
Opening balance 100
Amount capitalised 500
Closing balance (470)
Amortisation: balancing figure 130

© Emile Woolf International 123 The Institute of Chartered Accountants of Pakistan


Financial accounting and reporting II

(W2) Income tax


Rs.m Rs.m
Opening balance: tax provision 160
Opening balance: deferred tax 140
300
Tax charged to statement of profit or loss 270
Closing balance: tax provision (130)
Closing balance: deferred tax (310)
(440)
Tax paid (cash payments) 130

(W3) Property, plant and equipment


Rs.m
Opening balance 1,830
Revaluation surplus 200
Plant acquired 250
Depreciation (320)
1,960
Closing balance 1,890
Disposal at net book value – balancing figure 70

Disposal of plant:
Disposal at net book value (see above) 70
Loss on sale (given in the question) (50)
Difference = Sale proceeds 20

(W4) Share capital


Rs.m
Opening balance, ordinary shares 500
Bonus issue 1 for 10 (from retained earnings) 50
550
Closing balance, ordinary shares 750
Difference: shares issued for cash (nominal value) 200
Plus increase in share premium (350 – 100) 250
Total cash proceeds of issue of ordinary shares 450

(b) The cash flows generated from operations were Rs.685 million and are more than enough to
pay the interest costs and taxation, but these cash flows are not as large as the equivalent
profit figure. For most companies the operating cash flows are higher than the profit before
interest and tax due to the effects of depreciation/amortisation charges (which are not cash
flows). In the case of Ittehad Manufacturing Ltd the depreciation/amortisation effect has been
more than offset by a much higher investment in working capital of Rs.645 million. Inventory
has increased by over 50% and accounts receivable by 45%. This may be an indication of
expanding activity, but it could also be an indication of poor inventory management policy and
poor credit control, or even the presence of some obsolete inventory or unprovided bad
accounts receivable.

© Emile Woolf International 124 The Institute of Chartered Accountants of Pakistan


Answers

A cause of concern is the size of the dividends, which seem high at Rs.320 million. This is a
very high distribution ratio, and it seems odd that the company is returning such large
amounts to shareholders at the same time as they are raising finance. Rs.450 million has
been received from the issue of new shares and Rs.200 million from a further issue of loan
notes.
The company has invested considerably in new plant (Rs.250 million) and even more so in
development expenditure (Rs.500 million). If management has properly applied the
capitalisation criteria in IAS 38 Intangible Assets, then this indicates that they expect good
future returns from the investment in new products or processes. The net investment in non-
current assets is Rs.680 million which closely correlates to the proceeds from financing of
Rs.650 million. In general it is acceptable to finance increases in the capacity of non-current
assets by raising additional finance, however operating cash flows should finance
replacement of consumed non-current assets.

3.6 WASEEM INDUSTRIES LIMITED


Waseem Industries Limited
Statement of cash flows for the year ended December 31, 2015
2015 Workings
Rs.m
Cash flows from operating activities
Profit before taxation 64 W1
Adjustments for:
Depreciation 17
Gain on sale of fixed assets (3)
Provision for gratuity 10 W2
Interest expense 16
104

Increase/decrease in working capital


Increase in stocks-in-trade (7) (55-48)
Increase in trade debts (13) (51-38)
Decrease in advance, prepayments and other receivables 6 W3
Increase in trade and other payables 22 W4
8

Cash generated from operations 112


Gratuity paid (6)
Interest paid (18) W5
Income taxes paid (22) W6
Net cash from operating activities 66

© Emile Woolf International 125 The Institute of Chartered Accountants of Pakistan


Financial accounting and reporting II

Cash flows from investing activities


Sale proceeds from sale of property, plant and equipment 26
Purchase of property, plant and equipment (100) W7
Increase in capital work in progress (2) (20 -18)
Sale of long term investments 25 (75-100)
Net cash used in investing activities (51)

Cash flows from financing activities


Payment of long term finances (21) W8
Increase in short term finances 7 (13 - 6)
*Dividend paid (10)
Net cash used in financing activities (24)
Net increase in cash and cash equivalents (9)
Cash and cash equivalent at the beginning of the year 20
Cash and cash equivalent at the end of the year 11

W1: Profit before taxation Rs.m


Unappropriated profit – closing 85
Income tax expenses for the year 2015 19
Dividend (Rs. 125 million x 8%) 10
114
Less: Unappropriated profit - opening (50)
64

W2: Provision for gratuity Rs.m


Provision for gratuity: Closing 16
Paid during the year 2015 6
22
Less: Provision for gratuity - opening 12
Provision for the year 10

W3: Advances, prepayments and other receivables Rs.m


Advances, payments and other receivables – closing 37
Advance Tax - closing (10)
27

Advances, payments and other receivables – opening 40


Advance Tax – opening (7)
33
Decrease in advances, prepayments and other receivables (6)

© Emile Woolf International 126 The Institute of Chartered Accountants of Pakistan


Answers

W4: Trade and other payables Rs.m


Trade and other payables – closing 66
Accrued mark-up – closing (7)
59

Trade and other payables – opening 46


Accrued mark-up – opening (9)
37
Increase in trade and other payables 22

W5: Interest paid Rs.m


Accrued mark up – opening 9
Expense for the year 16
25
Less: Accrued mark-up – closing (7)
Interest paid during the year 18

W6: Income taxes paid Rs.m


Advances taxes – closing 10
Provision for the year 19
29
Less: advance taxes - opening (7)
Income taxes paid during the year 22

W7: Fixed assets purchase Rs.m


Closing fixed assets 242
Depreciation for the year 17
Carrying amount of disposed off assets 23
302
Less: opening fixed assets (182)
Purchase of fixed assets 100

W8: Payment of long term finances Rs.m


Long term finance including current portion – closing
(118 + 22) 140
Long term finance including current portion – Opening (94 + 25) (119)
Payment during the year 21

© Emile Woolf International 127 The Institute of Chartered Accountants of Pakistan


Financial accounting and reporting II

3.7 JALIB INDUSTRIES LIMITED


Jalib Industries Limited
Statement of cash flow for the year ended December 31, 2015

Rs. in
million
CASH FLOW FROM OPERATING ACTIVITIES
Net profit before tax 191.40
Adjustments for:

Depreciation 27.70

Loss on sale of fixed assets 4.60

Provision for gratuity 15.50

Financial charges 10.50

Bad debt expense 1.20 Working 3


250.90
Working Capital Changes
Increase in creditors, accrued and other liabilities
2.80
([36.2 - 5] - [34.4 - 6])
Increase in stock in trade (80.80)

Increase in trade debts (5.00) Working 4


Decrease in advances and other receivables
6.00
[(42-2.2)-(37.4-3.6)]
Cash generated from operations 173.90
Gratuity paid (4.40) Working 2
Income tax paid [3.6 + 104.6 - 2.2] (106.00)
Financial charges paid (6 + 10.5 - 5) (11.50)
Net cash from operating activities 52.00

CASH FLOW FROM INVESTING ACTIVITIES


Capital expenditure incurred (57.00) Working 1
Proceeds on sale of fixed assets 10.40
Net cash used in investing activities (46.60)

CASH FLOW FROM FINANCING ACTIVITIES


Issue of share capital 99.00 Working 5
Repayment of long term loans (120 - 98) (22.00)
Dividend paid (1.4 + 75 - 3) (73.40)
Net cash from financing activities 3.60
Net increase in cash and cash equivalents 9.00
Cash and cash equivalent at the beginning of year 3.00
Cash and cash equivalent at the end of year 12.00

© Emile Woolf International 128 The Institute of Chartered Accountants of Pakistan


Answers

WORKING 1 Rs. in million


Capital expenditure incurred

Book value of PPE - Closing 129.40

Book value of CWIP - Closing 22.50

Add: Book value of assets sold during the year 15.00

Add: Depreciation for the year 27.70

Less: Book value of PPE - Opening (100.60)

Less: Book value of CWIP - Opening (37.00)


57.00
WORKING 2
Gratuity paid during the year
Opening balance 27.50

Provision for gratuity 15.50


43.00

Less: Closing balance (38.60)

Gratuity paid during the year 4.40

WORKING 3
Bad debts expense for the year
Closing balance (28.5 ÷ 0.95) - 28.5 1.50
Less: Opening balance (24.7 ÷ 0.95) - 24.7 (1.30)
Add: Bad debts written off 1.00
Bad debts expense for the year 1.20

WORKING 4
Increase in trade debts
Closing balance (28.5 ÷ 0.95) 30.00
Less: Opening balance (24.7 / 0.95) (26.00)
Add: Bad debts written off 1.00
5.00
WORKING 5
Issue of share capital
Closing balance of paid up capital 396.00
Closing balance of share premium 45.00
Less:

Opening balance of paid up capital (300.00)


Opening balance of share premium (12.00)
Issue of bonus shares (300 x 10%) (30.00)
99.00

© Emile Woolf International 129 The Institute of Chartered Accountants of Pakistan


Financial accounting and reporting II

3.8 APOLLO INDUSTRY LIMITED


Apollo Industry Limited
Statement of cash flows for the year ended December 31, 2015
Rs. in ‘000
Cash used in operating activities
Profit before taxation 6,500
Adjustment for: (non cash items / separately disclosed items)
Depreciation for the year (7,000-90-1,000) 5,910
Amortization for the year (1140+50-1100) 90
Provision for staff gratuity (1,400+300-1,190) 510
Profit on sale of fixed assets (2,800-1,000) (1,800)
Mark-up on short term placement (1,000)
Operating profit before working capital changes 10,210
Increase in working capital (12,125 – 15,700 + 4,200 – 6,250) (5,625)
Cash generated from operations 4,585
Payment for staff gratuity (300)
Payment for taxation (950 + 4,660 – 800) (4,810)
(525)
Cash used in investing activities
Capital expenditure incurred Note 1 (13,110)
Proceeds from sale of PPE (1,200 + 1,800) 3,000
Acquisition of intangible assets (50)
Mark-up received on short term placement 1,000
Long term deposits (400-300) (100)
(9,260)
Cash used in financing activities
Issue of ordinary share capital (25,000-2,000-20,000) 3,000
Net decrease in cash and cash equivalents (6,785)
Opening balance: cash and cash equivalents 7,225
Closing balance: cash and cash equivalents 440

Note 1 Capital expenditure incurred: Rs.000


Opening book value for PPE 25,500
Opening book value for CWIP 10,000
Book value of assets sold during the year (1,200)
Depreciation for the year (7,000-90-1,000) (5,910)
Revaluation reserve adjustment (1,000)
Closing book value for PPE (35,000)
Closing book value for CWIP (5,500)
(13,110)

© Emile Woolf International 130 The Institute of Chartered Accountants of Pakistan


Answers

3.9 MARVEL ENGINEERING LIMITED


Marvel Engineering Limited
Cash Flow Statement
For the year ended 30 June 2015
Workings 2015
Cash flows from operating activities Rs.m
Profit before taxation 88.00
Adjustment for non-cash charges and other items:
Depreciation 50.00
Impairment of plant and machinery 11.00
Financial charges 75.00
Gain on sale of fixed assets (2.00)
Gain on sale of investments (3.00)
Dividend income (30.00)
Provision for gratuity payable (55 - 50 + 6) 11.00
Working capital changes
Decrease / (increase) in current assets:
Increase in stock-in-trade (97 - 68) (29.00)
Increase in trade debts (see tutorial note) (76.00)
Other current assets (100 - 120) 20.00
Increase / (decrease) in current liabilities:
Trade and other payables ([73 - 7] - [56 - 3]) 13.00
Cash generated from operations 128.00
Financial charges paid (3 + 75 - 7) (71.00)
Income tax paid (5 + 21 + 21 - 12 - 15) (20.00)
Gratuity paid (6.00)
Net cash generated from operating activities 31.00

Cash flows from investing activities


Capital expenditure 1 (289.00)
Proceeds from sale of property, plant and equipment (5+2) 7.00
Proceeds from sale of investments (10+3) 13.00
Purchase of long term investments (130-100+10) (40.00)
Dividend received 30.00
Net cash used in investing activities (279.00)

Cash flows from financing activities


Insurance of ordinary shares 2 40.00
Proceeds from long term loan (330 - 110) 220.00
Payment of dividend (2 + (440 × 5%) - 4) (20.00)
Net cash from financing activities 240.00
Net decrease in cash and cash equivalents (8.00)
Cash and cash equivalent at the beginning of the year 39.00
Cash and cash equivalent at the end of the year 31.00

© Emile Woolf International 131 The Institute of Chartered Accountants of Pakistan


Financial accounting and reporting II

W1: Capital expenditure Rs.m


Closing balance 633.00
Add: Depreciation for the year 50.00
Add: Impairment against plant 11.00
Add: Disposal during the year 5.00
Less: Opening balance (410.00)
289.00

W2: Issuance of ordinary shares


Closing balance of share capital 494.00
Closing balance of share premium 8.00
Less: Bonus shares issued (440 × 5%) (22.00)
Less: Opening balance of share capital (440.00)
40.00

Tutorial note:

The original ICAP answer did not simply adjust for the movement in trade debts but added back the
write off for bad debts (Rs. 6 million) and movement in the doubtful debt provision (Rs. 4 million) and
then adjusted for the movement in trade debt before these write offs (Rs. 86 million).

As the trade debt contains the credit for the write off and the profit for the year contains the debit it is
easier to leave the expense in and adjust for the net movement.

The following working was provided in the official answers.

WORKINGS (All amount in million rupees)


Provision
Trade
W1: for bad
debtors
debts
Closing balance (133 ÷ 0.95) - 133 7.00 (133 ÷ 0.95) 140.00
Add: Bad debts written off 6.00 6.00
Less : Opening balance (57÷ 0.95) - 57 (3.00) (57÷ 0.95) (60.00)
10.00 86.00

© Emile Woolf International 132 The Institute of Chartered Accountants of Pakistan


Answers

CHAPTER 4 – CONSOLIDATED ACCOUNTS: STATEMENTS OF FINANCIAL


POSITION– BASIC APPROACH

4.1 HALL
Consolidated statement of financial position as at 31 December 2015
Rs.000
Assets
Non-current assets
Property, plant and equipment (35,000 + 20,000) 55,000
Goodwill 3,000
————
58,000
Current assets (16,000 + 14,000) 30,000
————
88,000
————
Equity and liabilities
Capital and reserves
Share capital 10,000
Retained earnings (W5) 16,000
————
26,000

Non-controlling interest (W4) 4,000


Long-term liabilities
8% Debenture loans (20,000 + 9,000) 29,000
Current liabilities (20,000 + 9,000) 29,000
————
88,000
————
WORKINGS

(1) Group structure

Hall

75%

Stand

© Emile Woolf International 133 The Institute of Chartered Accountants of Pakistan


Financial accounting and reporting II

(2) Net assets of Stand

Reporting date Date of acquisition Post-acquisition


Rs.000 Rs.000
Share capital 4,000 4,000
Retained earnings 12,000 8,000 4,000
16,000 12,000
(3) Goodwill
Rs.000
Cost of shares 12,000
Less Net assets acquired (75% u 12,000 (W2)) (9,000)
———
3,000
———
(4) Non-controlling interest (25% u 16,000 (W2))
4,000
——–
(5) Retained earnings
Hall Inc 13,000
Stand Inc (75% u 4,000 (W2)) 3,000
——–
16,000
——–

4.2 HASSLE
Consolidated statement of financial position as at 31 December 2015
Rs.
Sundry net assets (207,500 + 226,600) 474,100
————
474,100
————
Equity capital 120,000
Retained earnings (W5) 123,500
————
243,500
Non-controlling interests (W4) 24,000
Sundry liabilities (100,000 + 106,600) 206,600
————
474,100
————
WORKINGS
(1) Group structure
Hassle

80%

Strife

© Emile Woolf International 134 The Institute of Chartered Accountants of Pakistan


Answers

(2) Net assets of Strife

Reporting Date of Post-


date acquisition acquisition
Rs. Rs.
Share capital 50,000 50,000
Retained earnings 70,000 50,000 20,000
120,000 100,000
(3) Goodwill Rs.
Cost 60,000
Net assets acquired (80% u 100,000) (W2) (80,000)
————
(20,000)
————
(4) Non-controlling interest
20% u 120,000) (W2) 24,000
(5) Retained earnings
Hassle 87,500
Strife (80% u (70,000  50,000) (W2)) 16,000
“Negative goodwill” (W3) 20,000
———–
123,500
———–

4.3 HYMN
Consolidated statement of financial position as at 31 December 2015
Assets Rs.
Non-current assets
Property, plant and equipment 170,000
Goodwill 29,000
Current assets 275,000
————
474,000
————
Equity and liabilities
Shareholders’ equity
Share capital 100,000
Retained earnings (W5) 178,200
———–
278,200
Non-controlling interest (W4) 19,800
Current liabilities 176,000
————
474,000
————

© Emile Woolf International 135 The Institute of Chartered Accountants of Pakistan


Financial accounting and reporting II

WORKINGS
(1) Group structure

Hymn

80%

Psalm

(2) Net assets of Psalm

Reporting Date of Post-


date acquisition acquisition
Rs. Rs.
Share capital 50,000 50,000
Retained earnings 49,000 20,000 29,000
99,000 70,000

(3) Goodwill Rs.

Cost of shares 85,000


Net assets acquired
Psalm Inc (80% u 70,000) (W2) (56,000)
————
(29,000)
————
(4) Non-controlling interest Rs.
20% u 99,000 (W2) 19,800

(5) Retained earnings Rs.


Hymn 155,000
Psalm (80% u 29,000 (W2)) 23,200
—————
178,200
—————

4.4 HANG
Consolidated statement of financial position as at 31 December 2015
Assets Rs.
Non-current assets
Property, plant and equipment (240 + 180) 420,000
Goodwill 26,600
Current assets (250 + 196) 446,000
————–
892,600
————–

© Emile Woolf International 136 The Institute of Chartered Accountants of Pakistan


Answers

Equity and liabilities


Shareholders’ equity
Share capital 200,000
Share premium account 25,000
Retained earnings (W5) 198,000
————–
423,000
Non-controlling interest (W4) 87,600
Current liabilities (225 + 157) 382,000
————–
892,600
————–
WORKINGS
(1) Group structure
Hang

60%

Swing

(2) Net assets of Swing Inc


31 Dec 31 Dec
2015 2014
Rs. Rs.

Ordinary shares of Rs.1 each 90,000 90,000


Share premium account 49,000 49,000
Retained earnings 80,000 50,000
———– ———–
219,000 189,000
———– ———–

Reporting Date of Post-


date acquisition acquisition
Rs. Rs.
Share capital 90,000 90,000
Share premium 49,000 49,000
Retained earnings 80,000 50,000 30,000
219,000 189,000

(3) Goodwill Rs.


Cost 140,000
Net assets acquired (60% u 189,000) (W2) (113,400)
————
26,600
————

© Emile Woolf International 137 The Institute of Chartered Accountants of Pakistan


Financial accounting and reporting II

(4) Non-controlling interest Rs.


40% u 219,000 (W2) 87,600

(5) Retained earnings Rs.


Hang 180,000
Swing (60% u 30,000 (W2)) 18,000
————–
198,000
————–

4.5 HASH
Consolidated statement of financial position as at 31 December 2015
Rs.000
Sundry net assets (207,500 + 226,600) 434,100
Goodwill (W2) 8,800
————
442,900
————

Share capital 120,000


Retained earnings (W5) 92,300
————
212,300

Non-controlling interests (W4) 24,000


Sundry liabilities (100,000 + 106,600) 206,600
————
442,900
————

WORKINGS
(1) Group structure

Hash

80%

Stash

© Emile Woolf International 138 The Institute of Chartered Accountants of Pakistan


Answers

(2) Net assets of Stash

Reporting Date of Post-


date acquisition acquisition
Rs.000 Rs.000
Share capital 50,000 50,000
Retained earnings:
At the start of the year (70,000 – 24,000) 46,000
9
Profit for the first 9m (24,000 × /12) 18,000
70,000 64,000 6,000
120,000 114,000

(3) Goodwill Rs.000


Cost 100,000
Net assets acquired (80% u 114,000) (W2) (91,200)
———
8,800
———
(4) Non-controlling interest
20% u 120,000) (W2) 24,000

(5) Retained earnings


Hash 87,500
Stash (80% u (70,000  64,000) (W2)) 4,800
———–
92,300
———–

© Emile Woolf International 139 The Institute of Chartered Accountants of Pakistan


Financial accounting and reporting II

CHAPTER 5 – CONSOLIDATED ACCOUNTS: STATEMENTS OF FINANCIAL


POSITION– COMPLICATIONS

5.1 HAIL
Consolidated statement of financial position as at 31 December 2015
Rs.000 Rs.000
Assets
Non-current assets
Property, plant and equipment 246,000
Investments (68,000 – 65,000) 3,000
Goodwill (W3) 6,500
Current assets
Cash at bank and in hand 39,900
Trade receivables 138,300
Inventories 92,400
———–
526,100
———–
Equity and liabilities
Capital and reserves
Share capital 100,000
Capital reserve (W6) 18,000
Retained earnings (W5) 210,480
———–
328,480
Non-controlling interest (W4) 11,420

Current liabilities
Trade payables 183,000
Proposed dividend – parent company 3,000
– non controlling interest 200
———– 3,200
———–
526,100
———–
WORKINGS
(1) Group structure
Hail

90%

Snow

© Emile Woolf International 140 The Institute of Chartered Accountants of Pakistan


Answers

(2) Net assets of Snow

Reporting Date of Post-


date acquisition acquisition
Rs.000 Rs.000
Share capital 50,000 50,000
Share premium account 5,000 5,000
Revaluation reserve 20,000 
Retained earnings
Per question 41,200
Proposed dividend (2,000)
39,200 10,000 29,200
114,200 65,000
(3) Goodwill Rs.000
Cost of shares 65,000
Net assets acquired (90% u 65,000) (W2) (58,500)
————
6,500
————
(4) Non-controlling interest
10% u 114,200 (W2) 11,420
————
(5) Retained earnings
Hail 185,400
Proposed dividend (3,000)
Dividend receivable from Snow 1,800
Snow (90% u 29,200 (W2)) 26,280
————
210,480
————
(6) Capital reserve
Snow (90% u 20,000 (W2)) 18,000
————

5.2 HAIRY
Consolidated statement of financial position as at 31 December 2015 Rs.000
Assets
Non-current assets
Property, plant and equipment 180,000
Current assets
Cash at bank and in hand 15,500
Investments 3,000
Receivables 91,700
Inventory (17,000 + 11,000 – 800) 27,200
———–
317,400
———–

© Emile Woolf International 141 The Institute of Chartered Accountants of Pakistan


Financial accounting and reporting II

Equity and liabilities


Capital and reserves
Share capital 100,000
Share premium account 20,000
Capital reserve 23,000
Retained earnings (W5) 102,900
———–
245,900
Non-controlling interest (W4) 16,500

Current liabilities 55,000


———–
317,400
———–
WORKINGS
(1) Group structure
Hairy

80%

Spider

(2) Net assets of Spider

Reporting Date of Post-


date acquisition acquisition
Rs.000 Rs.000

Share capital 60,000 60,000


Share premium account 16,000 16,000
Retained earnings
Per question 7,300
Unrealised profit (800)
6,500 2,300 4,200
82,500 78,300

(3) Goodwill Rs.000


Cost of shares 55,000
Less Net assets acquired (80% u 78,300 (W2)) (62,640)
————
(7,640)
————

© Emile Woolf International 142 The Institute of Chartered Accountants of Pakistan


Answers

(4) Non-controlling interest Rs.000


Share of net assets (20% u 82,500 (W2)) 16,500
————

(5) Retained earnings


Hairy 91,900
Spider (80% u 4,200 (W2)) 3,360
Negative goodwill (W4) 7,640
————
102,900
————

5.3 HARD
Consolidated statement of financial position as at 31 December 2015
Assets Rs.000
Non-current assets
Property, plant and equipment (225 + 175 – 17.5 (W6)) 382,500
Goodwill (W3) 14,000
Current assets (271 + 157) 428,000
———–
824,500
———–
Equity and liabilities
Shareholders’ equity
Share capital 100,000
Share premium account 15,000
Retained earnings (W5) 260,500
———–
375,500
Non-controlling interest (W4) 76,000
Current liabilities 373,000
———–
824,500
———–
WORKINGS
(1) Group structure

Hard

60%

Soft

© Emile Woolf International 143 The Institute of Chartered Accountants of Pakistan


Financial accounting and reporting II

(2) Net assets of Soft Inc

31 Dec 31 Dec Post-


acquisition
2015 2014
Rs.000 Rs.000

Share capital 100,000 100,000


Share premium account 10,000 10,000
Retained earnings 80,000 50,000 30,000
190,000 160,000

(3) Goodwill Rs.000


Cost 110,000
Net assets acquired 60% u 160,000 (W2) (96,000)
————
14,000
————
(4) Non-controlling interest
40% u 190,000 (W2) 76,000

(5) Retained earnings


Hard 260,000
Less Adjustment re intra group transfer (17,500)
————
242,500
Soft (60% u (80,000 – 50,000 (W2)) 18,000
————
260,500
————
(6) PURP on non current assets
IS
Cost 50,000
Accumulated depreciation (12,500)
————
37,500
————
SHOULD BE
Cost 100,000
Accumulated depreciation (80,000)
————
20,000
————
Dr Retained earnings 17,500
Cr Non current assets 17,500

© Emile Woolf International 144 The Institute of Chartered Accountants of Pakistan


Answers

5.4 HALE
(a) Consolidated statement of financial position as at 31 December 2015
Rs.000
Assets
Non-current assets
Property, plant and equipment
(152,000 + 129,600 + 28,000 (W2)) 309,600
Goodwill (W3) 61,400
Current assets
Bank (41,000 + 8,000) 49,000
Receivables (104,000 + 84,000) 188,000
Inventory (112,000 + 74,400 – 3,200 (W6)) 183,200
————–
791,200
————–
Equity and liabilities
Capital and reserves
Share capital 100,000
Retained earnings (W5) 555,200
————–
655,200
Non-controlling interest (W3) 60,000
Current liabilities (52,000 + 24,000) 76,000
————–
791,200
————–
WORKINGS
(1) Group structure
Hale

128
160
= 80% ord

Sowen

(2) Net assets of Sowen

Reporting Date of Post-


date acquisition acquisition
Rs.000 Rs.000
Share capital 160,000 160,000
Fair value adjustment on non-
current assets 28,000 28,000
Retained earnings 112,000 (11,000) 123,000
300,000 177,000

© Emile Woolf International 145 The Institute of Chartered Accountants of Pakistan


Financial accounting and reporting II

(3) Goodwill Rs.000


Cost of shares 203,000
Less Net assets acquired (80% u 177,000 (W2)) (141,600)
————–
61,400
————–
(4) Non-controlling interest
Share of net assets (20% u 300,000 (W2)) 60,000
————
(5) Retained earnings
Hale 460,000
PURP (W6) (3,200)
Sowen (80% u 123,000 (W2)) 98,400
————–
555,200
————–
(6) Unrealised profits % Rs.000
SP 125 16,000
Cost (100) (12,800)
—— ———
GP 25 3,200
—— ———

5.5 HELLO
Consolidated statement of financial position as at 31 December 2015
Assets Rs.
Non-current assets
Property, plant and equipment (225 + 175 + 10 – 2) 408,000
Goodwill (W3) 8,000
Current assets (271 + 157) 428,000
———–
844,000
———–
Equity and liabilities
Shareholders’ equity
Called up share capital 100,000
Retained earnings (W5) 291,800
———–
391,800
Non-controlling interest (W4) 79,200
Current liabilities 373,000
———–
844,000
———–

© Emile Woolf International 146 The Institute of Chartered Accountants of Pakistan


Answers

WORKINGS
(1) Group structure

Hello

60%

Solong

(2) Net assets of Solong Inc

Reporting Date of Post-


date acquisition acquisition
Rs. Rs.
Share capital 100,000 100,000
Retained earnings
Per the question 90,000
Less: Fair value adjustment for
depreciation (2/10 × 10,000) (2,000)
88.000 60,000
Fair value adjustment 10,000 10,000
198,000 170,000

(3) Goodwill Rs.


Cost 110,000
Net assets acquired
60% u 170,000 (W2) (102,000)
————
8,000
————
(4) Non-controlling interest
40% u 198,000 (W2) 79,200

(5) Retained earnings

Hello 275,000
Solong (60% u (88,000 – 60,000 (W2)) 16,800
———–
291,800
———–

© Emile Woolf International 147 The Institute of Chartered Accountants of Pakistan


Financial accounting and reporting II

5.6 HASAN LIMITED


Hasan Limited
Consolidated statement of financial position as at 31 March 2015
Rs.000 Rs.000
Assets
Non-current assets
Property, plant and equipment (W1) 4,020
Goodwill (W4) 480
Software (W1) 1,440
Investments (65 + 210) 275
–––––––––––––
6,215
Current assets
Inventories (W2) 1,274
Trade receivables (524 + 328) 852
Cash and bank (20 + 55 cash in transit) 75
–––––––––––––
2,201
–––––––––––––

Total assets 8,416


–––––––––––––

Equity and liabilities


Capital and reserves
Equity capital 2,000
Reserves
Share premium 2,000
Retained earnings (W3) 2,420
–––––––––––––
4,420
–––––––––––––

6,420
Non-controlling interest (W5) 350
Non-current liabilities
Government grants (230 + 40) 270
Current liabilities
Trade payables (475 + 472) 947
Operating overdraft 27
Income tax liability (228 + 174) 402
–––––––––––––
1,376
–––––––––––––

Total equity and liabilities 8,416


–––––––––––––

© Emile Woolf International 148 The Institute of Chartered Accountants of Pakistan


Answers

Workings
(W1) Property, plant and equipment
Rs.000
Balance from question – Hasan Limited 2,120
Balance from question – Shakeel Limited 1,990
Fair value adjustment on acquisition (see below) (120)
Over-depreciation re fair value adjustment year to 31 March 2015 30
–––––––––––––
4,020
–––––––––––––

A fair value of the leasehold based on the present value of the future rentals (receivable in
advance) would be the next (non-discounted) payment of the rental plus the final three
years as an annuity at 10%:

Rs.000
PV of rental receipts: Rs.80,000 + (Rs.80,000 u 2.50) 280
Carrying value on acquisition is (400)
–––––––––––––
Fair value reduction of leasehold (120)
–––––––––––––

The depreciation of the leasehold in Shakeel Limited’s accounts would be Rs.100,000 per
annum. However in the consolidated accounts it should be Rs.70,000 (Rs.280,000/4).
This would require a reduction in depreciation of Rs.30,000 in the consolidated accounts
for the next four years.
Software:

Shakeel Consolidated Difference


Limited’s figures
accounts
Rs.000 Rs.000
Capitalised amount 2,400 2,400
Depreciation to
31 March 2014 (300) 8 year life (480) 5 year life
––––– –––––
Value at date of acquisition 180 fair
2,100 1,920 value adjustment
Depreciation to 180 additional
31 March 2015 (300) (480) amortisation
––––– –––––
Carrying value
31 March 2015 1,800 1,440
––––– –––––
(W2) Inventories

Rs.000
Amounts given in the question (719 + 560) 1,279
Unrealised profit in inventories (25 u 25/125) (5)
–––––––––––––
1,274
–––––––––––––

© Emile Woolf International 149 The Institute of Chartered Accountants of Pakistan


Financial accounting and reporting II

(W3) Retained earnings

Rs.000
Retained profits of Shakeel Limited, 31 March 2015 1,955
Adjustments:
Excess charge for leasehold depreciation 30
Insufficient charge for Software amortisation (180)
Unrealised profit in inventory (W2) (5)
–––––––––––––

Adjusted retained profits at 31 March 2015 1,800


Retained earnings of Shakeel Limited at 1 April 2014 2,200
–––––––––––––

Shakeel Limited: loss for the year (post-acquisition loss) (400)


–––––––––––––

Parent company share of post-acquisition loss (90%) (360)


Hasan Limited reserves at 31 March 2015 2,900
Goodwill impairment (120)
–––––––––––––

Consolidated retained profits at 31 March 2015 2,420


–––––––––––––

(W4) Goodwill

Rs.000
At acquisition date
Shares of Shakeel Limited 1,500
Share premium of Shakeel Limited 500
Retained earnings of Shakeel Limited 2,200
Fair value adjustments:
Leasehold (W1) (120)
Software (W1) (180)
–––––––––––––

3,900
–––––––––––––

Acquired by Hasan Limited (90%) 3,510


Cost of investment 4,110
–––––––––––––

Goodwill at acquisition 600


Impairment 120
–––––––––––––

Goodwill at 31 March 2015 480


–––––––––––––

(W5) Non-controlling interests


Rs.000
Share capital of Shakeel Limited 1,500
Share premium of Shakeel Limited 500
Adjusted retained earnings of Shakeel Limited, 31 March 2015 1,800
(W3)

© Emile Woolf International 150 The Institute of Chartered Accountants of Pakistan


Answers

Rs.000
Fair value adjustments:
Leasehold (120)
Software (180)
–––––––––––––

Total net assets at 31 March 2015 3,500


–––––––––––––

Non-controlling interests (10%) 350


–––––––––––––

(W6) Elimination of current accounts:

Rs.000
Shakeel Limited’s current account with Hasan Limited per question 75
Deduct cash in transit regarding this balance (15)
–––––––––––––

Adjusted figure to cancel 60


–––––––––––––

(W7) Elimination of intra-group loan:

Rs.000
Investment in Hasan Limited’s books 200
Deduct repayment in transit (40)
–––––––––––––

Non-current liability in Shakeel Limited’s books 160


–––––––––––––

© Emile Woolf International 151 The Institute of Chartered Accountants of Pakistan


Financial accounting and reporting II

CHAPTER 6 – CONSOLIDATED ACCOUNTS: STATEMENTS OF COMPREHENSIVE


INCOME
6.1 HARRY
Consolidated statement of profit or loss for the year ended 31 December 2015
Rs.000

Revenue 1,410
Cost of sales (733)
——–
Gross profit 677
Distribution costs (90)
Administrative expenses (100)
——–
Operating profit 487
Investment income 9
Finance costs (22)
——–
Profit before tax 474
Income tax expense (165)
——–
Profit after tax 309
Non-controlling interest (W3) (15)
——–
Profit 294
——–
Movement on consolidated retained earnings for the
year ended 31 December 2015
Retained earnings at 1 January 2014 (W4) 127
Retained earnings for the year 294
Dividends (50)
——–
Retained earnings at 31 December 2015 (W5) 371
——–
WORKINGS
(1) Group structure

Harry

75%

Sally

© Emile Woolf International 152 The Institute of Chartered Accountants of Pakistan


Answers

(2) Consolidated statement of profit or loss


Harry Sally Adj Consol
Rs.000 Rs.000 Rs.000 Rs.000

Revenue 1,120 390 (100) 1,410


C of S – per Q (610) (220) 100
– PURP (3) – – (733)
Distribution costs (50) (40) (90)
Administrative expenses (55) (45) (100)
Investment income (20 – 15) 5 4 9
Interest payable (18) (4) (22)
Tax (140) (25) (165)
—–
PAT 60
—–
(3) Non-controlling interest Rs.000
25% u 60,000 (W1) or as per PAT in question 15
——
(4) Reserves brought forward
Harry 100
Sally (75% u (45 – 9)) 27
——
127
——
(5) Reserves carried forward (proof)
Harry 317
PURP (3)
Sally (75% u (85 – 9)) 57
——
371
——
(6) Inter-company dividend
Payable by Sally 20
—–
Receivable by Harry (75% u 20) 15
—–
6.2 HORNY
Consolidated statement of profit or loss for the year ended 31 December 2015
Rs.000

Revenue 362,000
Cost of sales (169,050)
———–
Gross profit 192,950
Operating costs (93,817)
———–
Operating profit 99,133

© Emile Woolf International 153 The Institute of Chartered Accountants of Pakistan


Financial accounting and reporting II

Investment income 13,100


Negative goodwill 3,800
———–
Profit before tax 116,033
Income tax (48,400)
———–
Profit after tax 67,633
Non-controlling interest (W3) (2,996)
———–
Profit 64,637
———–
Movement on consolidated retained earnings for the year ended
31 December 2015
Rs.000

Retained earnings at 1 January 2015 80,200


Retained profit for the year 64,637
Dividend (20,000)
———–
Retained earnings at 31 December 2015 124,837
———–
WORKINGS
(1) Group structure

Horny

75% (acq 31 August 2005)

Smooth
(2) Consolidation schedule
Horny Smooth Adj Consol
4
12
Rs.000 Rs.000 Rs.000 Rs.000
Revenue 304,900 65,100 (8,000) 362,000
Cost of sales (144,200) (32,850) 8,000 (169,050)

Operating costs (76,450) (17,367) (93,817)


Investment income
of H 10,500
of S (all of it) 2,600 13,100
Tax (42,900) (5,500) (48,400)
———
PAT 11,983
———

© Emile Woolf International 154 The Institute of Chartered Accountants of Pakistan


Answers

(3) Non-controlling interest @ 25%


——–
= 2,996
——–

(4) Consolidated retained earnings carried forward - proof


Rs.000

Horny 112,050
Simpson (11,983 – 2,996) 8,987
Negative goodwill 3,800
———
124,837
———

6.3 HERON
Consolidated statement of financial position as at 30 June 2015
Rs.000
Assets
Non-current assets
Property, plant and equipment (31,000 + 15,000) 46,000
Current assets (23,000 + 11,000) 34,000
———
80,000

Equity and liabilities
Shareholders’ equity
Share capital 10,000
Share premium account 5,000
2
Retained earnings (20,000 + ( u 18,500)) 32,333
3
———
47,333
1
Non-controlling interest (3 u 20,000) 6,667

Non-current liabilities 15,000


Current liabilities (5,000 + 6,000) 11,000
———
80,000

Consolidated statement of profit or loss for the
year ended 30 June 2015
Revenue (30,000 + 25,000) 55,000
Cost of sales (9,000 + 10,000) (19,000)
———
Gross profit 36,000
Distribution costs (3,000 + 1,200) (4,200)

© Emile Woolf International 155 The Institute of Chartered Accountants of Pakistan


Financial accounting and reporting II

Administrative expenses (1,000 + 2,800) (3,800)


Finance costs (2,000)
———
Profit before tax 26,000
Income tax expense (3,000 + 3,000) (6,000)
———
Profit for the period 20,000
1
Non-controlling interest (3 u 8,000) (2,667)
———
Profit for the financial year attributable to the members of Heron Inc 17,333
———
Consolidated statement of changes in equity for the
year ended 30 June 2015 (extract)
2
Retained earnings brought forward (8,000 + (3 u 10,500)) 15,000

Profit for the financial year attributable to the members of Heron Inc 17,333
———
Retained earnings carried forward 32,333
———

6.4 HANKS
Consolidated statement of financial position as at 31 December 2015
Assets Rs.000 Rs.000
Non-current assets
Property, plant and equipment
(32,000 + 25,000 + 20,000 + 6,000) 83,000
Goodwill 4,500
———–
87,500
Current assets
Cash at bank and in hand (9,500 + 2,000 + 4,000) 15,500
Receivables (20,000 + 8,000 + 17,000) 45,000
Inventory (30,000 + 18,000 + 18,000 – 2,100) 63,900
———–
124,400
———–
Total assets 211,900

Equity and liabilities
Share capital 40,000
Share premium account 6,500
Retained earnings (W5) 88,300
———–
134,800
Non-controlling interest (W4) 28,100

© Emile Woolf International 156 The Institute of Chartered Accountants of Pakistan


Answers

Current liabilities
Trade payables (23,500 + 6,000 + 17,000) 46,500
Proposed dividends – to minority shareholders (2,500 – 2,000) 500
– to Hanks’s shareholders 2,000
———– 49,000
———–
Total equity and liabilities 211,900

Consolidated statement of profit or loss for the
year ended 31 December 2015
Rs.000

Revenue (W6) 310,000


Cost of sales (W6) (159,100)
———–
Gross profit 150,900
Distribution costs (W6) (51,000)
Administrative expenses (W6) (29,500)
———–
Profit before taxation 70,400
Tax (W6) (24,000)
———–
Profit after taxation 46,400
Non-controlling interest (W6) (9,200)
———–
Profit 37,200


Statement of movements on reserves for the year ended 31 December 2015


Share
Share premium Retained
Capital account earnings Total
Rs.000 Rs.000 Rs.000

At 1 January 2015 40,000 6,500 53,100 (W7) 99,600


Profit for the year – 37,200 37,200
Dividends (proposed) (2,000) (2,000)
——–— –––––– ––––––– –––––––
At 31 December 2015 40,000 6,500 88,300 134,800
   
WORKINGS
(1) Group structure
Hanks

80% 60%

Streep Scott

© Emile Woolf International 157 The Institute of Chartered Accountants of Pakistan


Financial accounting and reporting II

(2) Net assets


Streep

Reporting Date of Post-


date acquisition acquisition
Rs.000 Rs.000
Share capital 10,000 10,000
Retained earnings
Per question 37,000
Proposed dividend (2,500)
34,500 7,500 27,000
44,500 17,500

Scott

Reporting Date of Post-


date acquisition acquisition
Rs.000 Rs.000
Share capital 15,000 15,000
Retained earnings 27,000 3,000 24,000
Revaluation reserve 6,000 6,000
48,000 24,000
(3) Goodwill on Streep
Rs.000
Cost of shares 20,500
Net assets acquired (80% u 17,500) (W2) (14,000)
———
6,500
———
Of which:
Written off by start of the year (6,500 – 5,000) 1,500
Written off by end of the year (6,500 – 4,500) 2,000
———
Recognised as impairment during the year (balancing figure) 500
———
Goodwill on Scott
Cost of shares 13,000
Net assets acquired (60% u 24,000 (W2)) (14,400)
———
(1,400)
———
(4) Non-controlling interest
Streep (20% u 44,500 (W2)) 8,900
Scott (40% u 48,000 (W2)) 19,200
———
28,100
———

© Emile Woolf International 158 The Institute of Chartered Accountants of Pakistan


Answers

(5) Consolidated retained earnings c/f Rs.000


Hanks 55,000
Dividend receivable from Streep (80% of 2,500) 2,000
Proposed dividend (2,000)
Streep (80% u 27,000 (W2)) 21,600
Scott (60% u 24,000 (W2)) 14,400
30
PURP ((5,200 + 3,900) u 130 ) (2,100)

Goodwill impairment – Streep (2,000)


Negative goodwill – Scott 1,400
———
88,300
———
(6) Consolidation schedule
Hanks Streep Scott Adj Consol
Rs.000 Rs.000 Rs.000 Rs.000 Rs.000

Sales revenue 125,000 117,000 82,000 (14,000) 310,000


C of S – per Q (65,000) (64,000) (42,000) 14,000
– PURP (W5) (2,100)
(159,100)
Distrib (21,000) (14,000) (16,000)
(51,000)
Admin (14,000) (8,000) (7,000) (500)
(29,500)
Tax (10,000) (9,000) (5,000) (24,000)
——— ———
PAT 22,000 12,000
Non-controlling interest in
profit after tax @20% @40%
——– ——–
4,400 + 4,800 = 9,200
——– ——– ——–

(7) Consolidated retained earnings b/f Rs.000


Hanks 40,000
Share of post acquisition profits of Streep (80% (15,000 – 7,500)) 6,000
Share of post acquisition profits of Scott (60% (15,000 – 3,000)) 7,200
Goodwill impairment - Streep (1,500)
Negative goodwill credited 1,400
———
53,100
———

© Emile Woolf International 159 The Institute of Chartered Accountants of Pakistan


Financial accounting and reporting II

CHAPTER 7 – IAS 16: PROPERTY, PLANT AND EQUIPMENT

7.1 ROONEY
(a) Borrowing costs
IAS 23 should be applied in accounting for borrowing costs.
Borrowing costs are recognised as an expense in the period in which they are incurred unless
they are capitalised in accordance with IAS 23 which says that borrowing costs that are
directly attributable to the acquisition, construction or production of a qualifying asset can be
capitalised as part of the cost of that asset.
‰ A qualifying asset is an asset that necessarily takes a substantial period of time to get
ready for its intended use or sale.
‰ Borrowing costs that are directly attributable to acquisition, construction or production
are taken to mean those borrowing costs that would have been avoided if the
expenditure on the qualifying asset had not been made.
When an enterprise borrows specifically for the purpose of funding an asset, the identification
of the borrowing costs presents no problem as the amount capitalised is the actual borrowing
costs net of any income earned on the temporary investment of those borrowings.
If funds are borrowed, generally, the amount of borrowing costs eligible for capitalisation is
determined by applying a capitalisation rate to the expenditures on that asset calculated as
the weighted average of the borrowing costs applicable to general borrowings.
IAS 23 also contains rules on commencement of capitalisation, suspension of capitalisation
and cessation of capitalisation.

Amount capitalised Rs.000


Cost of manufacture 28,000
Interest capitalised (Rs.20m × 5% × 2 years) 2,000
–––––––
30,000
–––––––
(b) Accounting
Rule
IAS 16 requires that each part of an item (that has a cost that is significant in relation to the
total cost) is depreciated separately. Therefore the cost recognised at initial recognition must
be allocated to each part accordingly.
Accounting
(i) 31st March 2016

Carrying Depreciation Carrying


value value
1.4.2015 31.3.2016
Rs.000 Rs.000 Rs.000
Hydraulic system 9,000 3,000 6,000
“Frame” 21,000 2,625 18,375
–––––– –––––– ––––––
30,000 5,625 24,375
–––––– –––––– ––––––
Revaluation loss (to profit and loss) (3,375)
––––––
Fair value. 21,000
––––––

© Emile Woolf International 160 The Institute of Chartered Accountants of Pakistan


Answers

The carrying value of the assets should be written down by a factor of 21,000/24,375.
This gives a carrying value for the hydraulic system (in Rs.000) of 5,169 and for the
‘frame’ 15,831.
The hydraulic plant should be depreciated over two more years and the ‘frame’ over 7
more years.
(ii) 31st March 2017

Carrying Depreciation Carrying


value charge value
1.4.2016 31.3.2017
Rs.000 Rs.000 Rs.000
Hydraulic system 5,169 2,585 2,584
“Frame” 15,831 2,262 13,569
––––––– ––––––– –––––––
21,000 4,847 16,153
Revalued amount 19,600
–––––––
Total gain 3,447
–––––––

To statement of profit or loss 3,375


Other comprehensive income 72
–––––––
Fair value 19,600
–––––––
The total revaluation gain is 3,447. Of this total amount, 3,375 reverses the loss in the
previous year and is therefore reported in profit and loss for the year. The remaining 72 is
reported as other comprehensive income.
(Tutorial note: Deferred tax is ignored by this question.)

7.2 EHTISHAM
IAS 16 permits assets to be carried at cost or revaluation. Where the latter is chosen, the asset must
be stated at its fair value.
The original depreciation was Rs. 40,000 (Rs. 1,000,000/25 years) per annum.
On 31st March 2014 the asset is two years old. Its carrying value before revaluation was therefore
Rs.1million less accumulated depreciation of Rs.80,000 (2/25 × Rs. 1 million).

Rs.
Cost/valuation 1,000,000
Accumulated depreciation (80,000)
Net book value 920,000

In order to effect the revaluation, the cost is uplifted to fair value of Rs.1.15m, the accumulated
depreciation is eliminated, and the uplift to the net book value is credited to a revaluation surplus
account.

Debit Credit
Cost/valuation 150,000
Accumulated depreciation 80,000
Revaluation surplus 230,000

© Emile Woolf International 161 The Institute of Chartered Accountants of Pakistan


Financial accounting and reporting II

The impact of the journal is as follows:

Before Adjustment After


Cost/valuation 1,000,000 150,000 1,150,000
Accumulated depreciation (80,000) 80,000 nil
Net book value 920,000 1,150,000

The asset is depreciated over its remaining useful economic life of 23 years giving a charge of Rs.
50,000 (Rs. 1,150,000/23 years) per annum in the year to 31st March 2015.

Debit Credit
Statement of profit or loss 50,000
Accumulated depreciation 50,000

This results in a carrying value as at 31st March 2015 of:

Rs.
Cost/valuation 1,150,000
Accumulated depreciation (50,000)
Net book value 1,100,000

Transfer from revaluation surplus to retained earnings


As a result of the revaluation, the annual depreciation has increased from Rs.40,000 to Rs.50,000.
This extra depreciation of Rs.10,000 is transferred from the revaluation reserve to accumulated
profits each year.

Debit Credit
Revaluation surplus 10,000
Accumulated profits 10,000

By the 31st March 2015, the balance remaining on the revaluation reserve will be Rs.220,000.

Rs.
Surplus recognised at 31 March 2014 230,000
Transfer to accumulated profits (10,000)
Net book value 220,000

The fall in property values at the year-end. The asset must be revalued downwards to Rs.0.8million,
a write-down of Rs.300,000.
Rs.220,000 of this is charged against the revaluation reserve relating to this asset, and the
remaining Rs.80,000 must be charged against profits.
The reduction of the carrying amount of the asset is achieved by removing the accumulated
depreciation and adjusting the asset account by the balance.

Debit Credit
Revaluation surplus 220,000
Statement of profit or loss 80,000
Asset at valuation 350,000
Accumulated depreciation 50,000

© Emile Woolf International 162 The Institute of Chartered Accountants of Pakistan


Answers

The impact of the journal is as follows:

Before Adjustment After


Cost/valuation 1,150,000 350,000 800,000
Accumulated depreciation (50,000) 50,000 nil
Net book value 1,100,000 800,000

This balance is depreciated over the remaining useful life of the asset (22 years).

7.3 CARLY
Financial statements for the year ended 31 December 2015 (extract)
Property, plant and equipment

Land and Plant and Computer


buildings machinery equipment
Total
Rs. Rs. Rs. Rs.
Cost/valuation
At 1 January 2015 1,500,000 340,500 617,800 2,458,300
Revaluation 250,000 - - 250,000
Additions (W2) - 17,550 - 17,550
Disposals - (80,000) - (80,000)
–––––––––– –––––––– –––––––– ––––––––––
At 31 December 2015 1,750,000 278,050 617,800 2,645,850
–––––––––– –––––––– –––––––– ––––––––––
Accumulated depreciation
At 1 January 2015 600,000 125,900 505,800 1,231,700
Charge for the year (W1) 20,000 51,191 44,800 115,991
Revaluation (620,000) - - (620,000)
Disposals - (57,000) - (57,000)
–––––––––– –––––––– –––––––– ––––––––––
At 31 December 2015 nil 120,091 550,600 670,691
–––––––––– –––––––– –––––––– ––––––––––
Carrying amount
At 31 December 2014 900,000 214,600 112,000 1,226,600
–––––––––– –––––––– –––––––– ––––––––––
At 31 December 2015 1,750,000 157,959 67,200 1,975,159
–––––––––– –––––––– –––––––– ––––––––––
Workings
(1) Depreciation charges
Buildings = (1,500,000 – 500,000) u 2% = 20,000.
Plant and machinery:
Rs.
9
New machine (17,550 u 25% u /12) 3,291
Existing plant (((340,500 – 80,000) – (125,900 – 57,000)) u 25%) 47,900
–––––––
51,191
–––––––
Computer equipment = 112,000 u 40% = Rs.44,800

© Emile Woolf International 163 The Institute of Chartered Accountants of Pakistan


Financial accounting and reporting II

(2) Cost of new machine

Rs.
Purchase price (20,000 – 3,000 – 1,000) 16,000
Delivery costs 500
Installation costs 750
Interest on loan taken out to finance the purchase 300
–––––––
17,550
–––––––

7.4 ADJUSTMENTS LIMITED


(a) Lathe
The lathe was purchased in 2009 and was originally being written off over an estimated useful
life of twelve years. As at 1 January 2015 six of the years have elapsed with a further six
years remaining. It was decided that the machine will now only be usable for a further four
years.
IAS 16 Property, plant and equipment requires that where the original estimate of useful life is
revised, adjustments should be made in current and future periods (not in prior periods). The
unamortised cost of the asset should be charged to revenue over the remaining useful life of
the asset. The net book value of Rs.75,000 should therefore be charged over the remaining
four years of useful life, giving an annual depreciation charge of Rs.18,750.
The revision is not a change in accounting policy, or a fundamental error but a change in
accounting estimate. It is therefore not appropriate to deal with any excess depreciation by
adjusting opening retained earnings.
(b) Grinder
The grinder was purchased in 2012 and was originally being depreciated on a straight line
basis. It has now been decided to depreciate this on the sum of digits basis.
IAS 16 requires that depreciation methods be reviewed periodically and if there is a significant
change in the expected pattern of economic benefits, the method should be changed.
Depreciation adjustments should be made in current and future periods. This change might be
appropriate if, for instance, usage of the machine is greater in the early years of an asset’s life
when it is still new and consequently it is appropriate to have a higher depreciation charge.
If the change is implemented, the unamortised cost (the net book value) of the asset should
be written off over the remaining useful life commencing with the period in which the change is
made. The depreciation charge for the remaining life of the asset will therefore be as follows.
Year No of digits Depreciation
Rs.
2015 7 7/28 u Rs.70,000 17,500
2016 6 6/28 u Rs.70,000 15,000
2017 5 12,500
2018 4 10,000
2019 3 7,500
2020 2 5,000
2021 1 2,500
—— ———–
1/2 u 7 (7 + 1) 28 Rs. 70,000
—— ———–
Disclosure will need to be made in the accounts of the details of the change, including the
effect on the charge in the year.

© Emile Woolf International 164 The Institute of Chartered Accountants of Pakistan


Answers

(c) Leasehold land


IAS 16’s allowed alternative treatment in respect of measurement of property plant and
equipment (subsequent to initial recognition), is that of revaluation. Revaluation is made at fair
value.
Where any item of property plant or equipment is revalued, the entire class to which the asset
belongs should be revalued. Revaluations must be kept up to date. Where there are volatile
movements in fair value, the revaluation should be performed annually. Where there are no
such movements, revaluations every three to five years may be appropriate.
Accumulated depreciation at the date of revaluation is either
(i) restated proportionately with the change in the gross carrying amount so that the
carrying amount after the revaluation equals the revalued amount (e.g. where
revaluations are made to depreciated replacement cost using indices)
(ii) eliminated against the gross carrying amount of the assets and the net amount
restated to the revalued amount of the asset (e.g. where buildings are revalued to their
market value).
IAS 16 requires that the subsequent charge for depreciation should be based on the revalued
amount. The annual depreciation will therefore be Rs.62,500, i.e. Rs.1,500,000 divided by the
24 years of remaining life.
There will then be a difference between the revalued depreciation charge and the historical
depreciation charge.
The resulting excess depreciation may be dealt with by a movement in reserves, i.e. by
transferring from the revaluation reserve to retained earnings a figure equal to the
depreciation charged on the revaluation surplus each year.

7.5 FAM
Accounting policies
(a) Property, plant and equipment is stated at historical cost less depreciation, or at valuation.
(b) Depreciation is provided on all assets, except land, and is calculated to write down the cost or
valuation over the estimated useful life of the asset.
The principal rates are as follows.
Buildings 2% pa straight line
Plant and machinery 20% pa straight line
Fixtures and fittings 25% pa reducing balance

Fixed asset movements Land Plant Fixtures, Payments on


and and fittings, account and
buildings machinery tools and assets in the Total
equipment course of
construction
Cost/valuation Rs.000 Rs.000 Rs.000 Rs.000 Rs.000

Cost at 1 January 2015 900 1,613 390 91 2,994


Revaluation adjustment 600 – – – 600
Additions – 154 40 73 (W1) 267
Reclassifications 100 – – (100) –
Disposals – (277) (41) – (318)
——— ——— —— —— ———
Cost at 31 December 2015 100 1,490 389 64 2,043
2015 valuation 1,500 1,500
——— ——— —— —— ———

© Emile Woolf International 165 The Institute of Chartered Accountants of Pakistan


Financial accounting and reporting II

Depreciation
At 1 January 2015 80 458 140 – 678
Revaluation adjustment (80) – – – (80)
Provisions for year (W2) 17 298 70 – 385
Disposals – (195) (31) – (226)
—— —— —— —— ——
At 31 December 2015 17 561 179 – 757
—— —— —— —— ——
Net book value
At 31 December 2015 1,583 929 210 64 2,786
——— ——— —— —— ———
At 31 December 2014 820 1,155 250 91 2,316
——— ——— —— —— ———
Land and buildings have been revalued during the year by Messrs Jackson & Co on the basis
of an existing use value on the open market.

The corresponding historical cost information is as follows.


Land and buildings
Rs.000
Cost
Brought forward 900
Reclassification 100
———
Carried forward 1,000
———
Depreciation
Brought forward 80
Provided in year 10
———
Carried forward 90
———
Net book value 910
———
WORKINGS Rs. 000
(1) Additions to assets under construction 53
Deposit on computer 20
——
73
——
600
(2) Depreciation on buildings 40 + (100 u 2%) 17

2% straight line depreciation is equivalent to a 50 year life.


The buildings are ten years old at valuation and therefore
have 40 years remaining.
Depreciation on plant (1,613 + 154 – 277) u 20% 298
Depreciation on fixtures (390 + 40 – 41 – 140 + 31) u 25% 70

© Emile Woolf International 166 The Institute of Chartered Accountants of Pakistan


Answers

7.7 HUMAYUN CHEMICALS LIMITED


(a) If review is performed on June 30, 2015
Cost of machine Rs. 10,000,000
Depreciation charged @ 20% for the year ended June 30, 2013 and June
30, 2014 (Rs. 10,000,000  Rs. 3,000,000) x 20% x 2 Rs. 2,800,000
WDV as at June 30, 2014 Rs. 7,200,000
Residual value (10% of the cost of machine) Rs. 1,000,000
Depreciable amount - on July 1, 2014 Rs. 6,200,000

Remaining useful lives 6 years


Depreciation charge for the year ended June 30, 2015 Rs. 1,033,333

If review is performed on June 30, 2014


Cost of machine Rs. 10,000,000
Depreciation for the year ended June 30, 2013
(Rs. 10,000,000 - Rs. 3,000,000) x 20% Rs. 1,400,000

WDV as at June 30, 2013 Rs. 8,600,000


Residual value (10% of the cost of machine) Rs. 1,000,000
Depreciable amount - on July 1, 2013 Rs. 7,600,000

Remaining useful lives 6 years


Depreciation charge for the year ended June 30, 2015 Rs. 1,266,667

Depreciation charged in the financial statement for the year ended June
30, 2014 Rs. 1,400,000

Effect of change in estimate to be incorporated (Reversal)

[Rs. 1,400,000  1,266,667) (Rs. 133,333)

(b) According to IAS-16, the following factors should be considered when estimating the useful
life of a depreciable asset:

(i) Expected usage

(ii) Expected physical wear and tear

(iii) Obsolescence

(iv) Legal or other limits on the use of the assets.

Once the useful life of a depreciable asset is determined, it shall be reviewed at least at each
financial year-end.

If expectations vary from the previous estimates, then change should be adjusted for current
and future periods in accordance with the requirements of IAS 8.

© Emile Woolf International 167 The Institute of Chartered Accountants of Pakistan


Financial accounting and reporting II

7.8 FARADAY PHARMACEUTICAL LIMITED


Debit Credit
Date Particulars
Rs.000 Rs.000
01.07.2011 Building 200,000
Bank 200,000
(Record purchase of plant)
30.06.2012 Depreciation 10,000
Accumulated depreciation – Building 10,000
(Record depreciation for the year 2012)
Working: Rs. 200,000 ÷ 20 = Rs. 10,000
01.07.2012 Accumulated depreciation – Building 10,000
Building 10,000
(Reversal of prior year depreciation)
01.07.2012 Building 40,000
Surplus on revaluation of fixed assets 40,000
(Increase in value through revaluation)
Working: Rs. 230,000 – Rs. 190,000 =
Rs. 40,000
30.06.2013 Depreciation 12,105
Accumulated depreciation – Building 12,105
(Record depreciation for the year 2013)
Working: Rs. 230,000 ÷ 19 = Rs. 12,105
30.06.2013 Surplus on revaluation of fixed assets 2,105
Retained earnings/Profit & loss account 2,105
(transfer of surplus through retained earning to the
extent of excess depreciation)
Working: Rs. 40,000 ÷ 19 = Rs. 2,105
01.07.2013 Accumulated depreciation – Building 12,105
Building 12,105
(Reversal of prior year depreciation)
01.07.2013 Surplus on revaluation of fixed assets 37,895
Revaluation expense 10,000
Building 47,895
(Decrease in value through revaluation)
Working:
Reversal of Surplus balance (Rs. 40,000 – Rs.
2,105) Rs. 37,895.
Balancing figure of Rs. 10,000 charged to Profit
and Loss
Building value decline: (Rs. 230,000 – Rs. 12,105)
– Rs. 170,000 =Rs. 47,895

© Emile Woolf International 168 The Institute of Chartered Accountants of Pakistan


Answers

Debit Credit
Date Particulars
Rs.000 Rs.000
30.06.2014 Depreciation 9,444
Accumulated depreciation – Building 9,444
(Record depreciation for the year 2014)
Working: Rs. 170,000 ÷ 18 = Rs. 9,444
01.07.2014 Accumulated depreciation – Building 9,444
Building 9,444
(Reversal of prior year depreciation)
01.07.2014 Building 19,444
Revaluation income 9,444
Surplus on revaluation of fixed assets (balancing) 10,000
(Reversal of prior year impairment)
Working:
Revaluation income = Rs. 10,000 – [ Rs. 10,000 –
Rs. 9,444] = Rs. 9,444
Building: [Rs. 170,000 – Rs. 9,444] – Rs. 180,000
=Rs. 19,444
30.06.2015 Depreciation 10,588
Accumulated depreciation – Building 10,588
(Record depreciation for the year 2015)
Working: Rs. 180,000 ÷ 17 = Rs. 10,588
30.06.2015 Surplus on revaluation of fixed assets 588
Retained earnings 588
(Reverse the excess depreciation)
Working: Rs. 10,000 ÷ 17 = Rs. 588

7.9 SCIENTIFIC PHARMA LIMITED


Scientific Pharma Limited
Journal entries for the year ended June 30, 2015
Debit Credit
Rs.000 Rs.000
30.06.2015 Repair and maintenance expenses 1,500
Account payable / Bank 1,500
(Repair cost of major break down of the plant)
30.06.2015 Depreciation expense (45,000-2,000)/10.5 years 4,095
Accumulated depreciation 4,095
(Depreciation expense for the year)
30.06.2015 Revaluation surplus (10,380/10.5) 989
Retained earnings 989
(Incremental depreciation credited to retained
earnings)

© Emile Woolf International 169 The Institute of Chartered Accountants of Pakistan


Financial accounting and reporting II

Debit Credit
Rs.000 Rs.000
30.06.2015 Impairment loss W1 5,296
Property, plant and equipment 5,296
(Impairment of plant due to break down)
30.06.2015 Revaluation surplus 5,296
Impairment loss W1 5,296
(Impairment loss adjusted against revaluation)

W1: Impairment loss


Recoverable amount 19,227
WDV of the plant on impairment date W2 (24,523)
Impairment loss as on 30.06.2015 (5,296)

W2: WDV of the plant on impairment date Rs.000


FOB price (US$ 800,000 at Rs. 52) 41,600
Other charges including installation cost 7,000

48,600

Accumulated depreciation
(1-1-2006 to 30-6-2010) {(48,600-2,000)/15*4.5} (13,980)
WDV as on 30-6-2010 34,620
Revaluation surplus (45,000-34,620) 10,380
Revalued amount as of July 1, 2010 45,000
Accumulated depreciation
(1-7-2010 to 30-6-2015) {(45,000-2,000)/10.5*5) (20,476)
WDV as on 30-6-2015 24,523

W3: Revaluation surplus on impairment date


Revaluation surplus W2 10,380
Transferred to retained earnings
(01.07.2010 to 30.06.2015) (10,380/10.5*5) (4,943)
Revaluation surplus balance on impairment date 5,437

Since impairment loss is less than the revaluation surplus on impairment date, the full amount
of impairment would be adjusted against the revaluation surplus.

© Emile Woolf International 170 The Institute of Chartered Accountants of Pakistan


Answers

CHAPTER 8 – IAS 36: IMPAIRMENT OF ASSETS

8.1 ABA LIMITED


Aba Limited statement of profit or loss (extracts) – year to 31 March 2016

Note: workings in brackets are in Rs.000 Rs. Rs.


Depreciation: head office – 6 months to 1 October 2015
(1,200/25 u 6/12) 24,000
– 6 months to 31 March 2016
(1,350/22.5 (W1) u 6/12) 30,000
–––––––
54,000
–––––––
Depreciation: training premises
– 6 months to 1 October 2016
(900/25 u 6/12) 18,000
– 6 months to 31 March 2016
(600/10 u 6/12) 30,000
––––––––
48,000
––––––––
Impairment loss (W2) 210,000
––––––––
258,000
––––––––
Statement of financial position (extracts) as at 31 March 2016 Rs. Rs.
Non-current assets
Land and buildings– head office (700 + 1,350 – 30) 2,020,000
– training premises (350 + 600 – 30) 920,000
––––––––
2,940,000
––––––––
Revaluation reserve
Head office land (700 – 500) 200,000
Building (1,350 – 1,080 (W1)) 270,000
Training premises land (350 – 300) 50,000
––––––––
520,000
Transfer to realised profit (270/22.5 (W1) u 6/12
re depreciation of buildings) (6,000)
––––––––
514,000
––––––––
Workings
(W1) The date of the revaluation is two and a half years after acquisition. This means the remaining
life of the head office would be 22.5 years. The carrying value of the head office building at the
date of revaluation is Rs.1,080,000 i.e. its cost less two and a half years at Rs.48,000 per
annum (Rs.1,200,000 – Rs.120,000).
(W2) Impairment loss: the carrying value of training premises at date of revaluation is Rs.810,000 i.e.
its cost less two and a half years at Rs.36,000 per annum (Rs.900,000 – Rs.90,000). It is
revalued down to Rs.600,000 giving a loss of Rs.210,000. As the land and the buildings are
treated as separate assets the gain on the land cannot be used to offset the loss on the
buildings.

© Emile Woolf International 171 The Institute of Chartered Accountants of Pakistan


Financial accounting and reporting II

8.2 HUSSAIN ASSOCIATES LTD


(a) Impairment of plant
The plant had a carrying amount of Rs.240,000 on 1 October 2015. The accident that may
have caused impairment occurred on 1 April 2016 and an impairment test would be done at
this date. The depreciation on the plant from 1 October 2015 to 1 April 2016 would be
Rs.40,000 (640,000 x 121/2% x 6/12) giving a carrying amount of Rs.200,000 at the date of
impairment. An impairment test requires the plant’s carrying amount to be compared with its
recoverable amount. The recoverable amount of the plant is the higher of its value in use of
Rs.150,000 or its fair value less costs to sell. If Hussain Associates Ltd trades in the plant it
would receive Rs.180,000 by way of a part exchange, but this is conditional on buying new
plant which Hussain Associates Ltd is reluctant to do. A more realistic amount of the fair
value of the plant is its current disposal value of only Rs.20,000. Thus the recoverable amount
would be its value in use of Rs.150,000 giving an impairment loss of Rs.50,000 (Rs.200,000 –
Rs.150,000).
The remaining effect on income would be that a depreciation charge for the last six months of
the year would be required. As the damage has reduced the remaining life to only two years
(from the date of the impairment) the remaining depreciation would be Rs.37,500
(Rs.150,000/ 2 years u 6/12).Thus extracts from the financial statements for the year ended
30 September 2016 would be:
Statement of financial position
Non-current assets Rs.
Plant (150,000 – 37,500) 112,500
Statement of profit or loss
Plant depreciation (40,000 + 37,500) 77,500
Plant impairment loss 50,000
(b) Purchase of Sparkle
There are a number of issues relating to the carrying amount of the assets of Sparkle Limited
that have to be considered. It appears the value of the brand is based on the original
purchase of the ‘Sparkle Spring’ brand. The company no longer uses this brand name; it has
been renamed ‘Refresh’. Thus it would appear the purchased brand of ‘Sparkle Spring’ is now
worthless. Sparkle Limited cannot transfer the value of the old brand to the new brand,
because this would be the recognition of an internally developed intangible asset and the
brand of ‘Refresh’ does not appear to meet the recognition criteria in IAS 38. Thus prior to the
allocation of the impairment loss the value of the brand should be written off as it no longer
exists.
The inventories are valued at cost and contain Rs.2 million worth of old bottled water (Sparkle
Spring) that can be sold, but will have to be relabelled at a cost of Rs.250,000. However, as
the expected selling price of these bottles will be Rs.3 million (Rs.2 million u 150%), their net
realisable value is Rs.2,750,000. Thus it is correct to carry them at cost i.e. they are not
impaired. The future expenditure on the plant is a matter for the following year’s financial
statements.
Applying this, the revised carrying amount of the net assets of Sparkle Limited’s cash-
generating unit (CGU) would be Rs.25 million (Rs.32 million – Rs.7 million re the brand). The
CGU has a recoverable amount of Rs.20 million, thus there is an impairment loss of Rs.5
million. This would be applied first to goodwill (of which there is none) then to the remaining
assets pro rata. However under IAS2 the inventories should not be reduced as their net
realisable value is in excess of their cost. This would give revised carrying amounts at 30
September 2016 of:
Rs.000
Brand nil
Land containing spa: 12,000 – [(12,000/20,000) u 5,000] 9,000
Purifying and bottling plant: 8,000 – [(8,000/20,000) u 5,000] 6,000
Inventories 5,000
20,000

© Emile Woolf International 172 The Institute of Chartered Accountants of Pakistan


Answers

8.3 IMPS
(a) Impairment loss
Rs.m
Carrying value 500
Recoverable amount (385)
Impairment loss 115
Recoverable amount is value in use (Working 1) as this is higher than the fair value less costs of
disposal (Working 2).
Workings
(1) Value in use:
Forecast cash flows discounted at 12%:
Rs.m
Year 1 (185 × 0.893) 165.2
Year 2 (160 × 0.797) 127.5
Year 3 (130 × 0.712) 92.6
Total 385.3
(2) The fair value less costs of disposal:
Rs.m
Goodwill 0
Freehold 270
Freehold land and buildings 50
320

(b) Treatment of impairment loss


IAS 36 requires the impairment loss to be allocated to the various non-current assets in the
following order: firstly, goodwill, secondly, to other assets, either pro-rata or on another more
appropriate basis.

Before Impairment After


impairment loss (W1) impairment
Rs.m Rs.m Rs.m
Goodwill 70 (70) -
Land and buildings 320 (33) 287
Plant and machinery 110 (12) 98
500 (115) 385

Because the land and buildings have been re-valued, the impairment is treated as a
revaluation decrease until the carrying amount of the asset reaches its depreciated historical
cost. The revaluation reserve relating to the asset is Rs.65 million and so is adequate to cover
the full impairment of Rs.33m. The impairment must be separately disclosed and the notes to
the accounts must specify by class of asset the impairment recognised directly to equity.
The impairment loss on the goodwill and plant (Rs.82 million) must be recognised in profit or
loss for the year. The notes to the accounts must specify the line item in which the impairment
loss has been included.

© Emile Woolf International 173 The Institute of Chartered Accountants of Pakistan


Financial accounting and reporting II

Where the impairment write-down is material, information must also be provided as to the
events and circumstances that led to the loss, the nature of the assets affected, the segment
to which the asset belongs, that recoverable amount was based on value in use and the
discount rate used to calculate this.
Workings
Loss on the various non-current assets
After the impairment loss has been recognised on the goodwill there is still 115 - 70 = 45 loss
to be allocated to the other noncurrent assets, on a pro-rata basis.
320
Loss on land and buildings: x 45 33
320 110
110
Loss on plant and machinery: x 45 12
320 110

© Emile Woolf International 174 The Institute of Chartered Accountants of Pakistan


Answers

CHAPTER 9 – IAS 38: INTANGIBLE ASSETS


9.1 FAZAL
In accordance with IAS 38, expenditure on intangible assets must be expensed unless it meets the
recognition criteria for capitalisation. These criteria require the demonstration that future benefits will
arise from the incurred costs. It would be difficult to prove that this is the case in relation to training
costs and IAS 38 specifically states that training costs should always be expensed as they are
incurred and not treated as an intangible asset.
Hence the treatment adopted by Fazal is not correct and the costs being carried forward must be
expensed to the year’s profits.

9.2 HENRY
Property, plant and equipment
Plant and machinery
Cost Rs.
On 1 January 2015 X
Additions 30,000
–––––––
On 31 December 2015 X
–––––––
Accumulated depreciation
On 1 January 2015 X
Charge for the year (30,000 u 9/12 ÷ 5) 4,500
–––––––
On 31 December 2015 X
–––––––
Carrying amount
On 31 December 2014 X
–––––––
On 31 December 2015 25,500
–––––––
Intangible assets

Internally generated research and development expenditure


Cost Rs.
On 1 January 2015 412,500
Additions 45,000
––––––––
On 31 December 2015 457,500
––––––––

Accumulated amortisation
On 1 January 2015 -
Charge for the year (W) 68,750
––––––––
On 31 December 2015 68,750
––––––––
Carrying amount
On 31 December 2014 412,500
––––––––
On 31 December 2015 388,750
––––––––

© Emile Woolf International 175 The Institute of Chartered Accountants of Pakistan


Financial accounting and reporting II

Working
Amortisation charge (Project A)

Rs.
Total savings (100,000 + 300,000 + 200,000) 600,000
2015 amortisation charge (100,000/600,000 u 412,500) 68,750

Tutorial notes
The costs in respect of Project B cannot be capitalised as there are uncertainties surrounding the
successful outcome of the project – but the machine bought may be capitalised in accordance with
IAS16.
The 2015 costs in respect of Project C can be capitalised as the uncertainties have now been
resolved. However, the 2014 costs cannot be reinstated.

9.3 TOBY
Intangible assets
Goodwill Patents Brands Total
Rs. Rs. Rs. Rs.
Cost
On 1 January 2015 - - - -
Additions (W1) 10,000 20,000 50,000 80,000
––––––– ––––––– ––––––– –––––––
On 31 December 2015 10,000 20,000 50,000 80,000
––––––– ––––––– ––––––– –––––––
Accumulated amortisation/impairment
On 1 January 2015 - - - -
Written off/amortised during the year
(W1 and W2) 3,000 2,500 7,500 13,000
––––––– ––––––– ––––––– –––––––
On 31 December 2015 3,000 2,500 7,500 13,000
––––––– ––––––– ––––––– –––––––
Carrying amount
On 31 December Year 0 - - - -
––––––– ––––––– ––––––– –––––––
On 31 December 2015 7,000 17,500 42,500 67,000
––––––– ––––––– ––––––– –––––––
Workings
(1) Goodwill on acquisition of George

Rs.
Cost of acquisition 105,000
Minus fair value of net assets acquired (100,000 – 5,000) (95,000)
––––––––
Goodwill 10,000
Recoverable value (7,000)
––––––––
Impairment write off 3,000
––––––––
(2) Amortisation of patent

20,000 ÷ 8 = Rs.2,500

© Emile Woolf International 176 The Institute of Chartered Accountants of Pakistan


Answers

(3) Amortisation of brand

50,000 ÷ 5 × 9/12 = Rs.7,500


Tutorial note
IAS38 Intangible assets prohibits the recognition of internally generated brands (3) or internally-
generated goodwill (4).

9.4 BROOKLYN
1 Development expenditure
IAS 38 on intangibles requires that research and development be considered separately:
‰ research – which must be expensed as incurred
‰ development – which must be capitalised where certain criteria are met.
It must first be clarified how much of the Rs.3 million incurred to date (10 months at
Rs.300,000) is simply research and how much is development. The development element will
only be capitalised where the IAS 38 criteria are met. The criteria are listed below together
with the extent to which they appear to be met.
‰ The project must be believed to be technically feasible. This appears to be so as the
feasibility has been acknowledged.
‰ There must be an intention to complete and use/sell the intangible. Completion is
scheduled for June 2016
‰ The entity must be able to use or sell the intangible. Interest has been expressed in
purchasing the knoWhow on completion
‰ It must be considered that the asset will generate probable future benefits. Confirmation
is required from Brooklyn as to the extent of interest shown by the pharmaceutical
companies and whether this is of a sufficient level to generate orders and to cover the
deferred costs.
‰ Availability of adequate financial and technical resources must exist to complete the
project. The financial position of Brooklyn must be investigated. A grant is being
obtained to fund further work and the terms of the grant, together with any conditions,
must be discussed further.
‰ Able to identify and measure the expenditure incurred. A separate nominal ledger
account has been set up to track the expenditure.
If all of the above criteria are met, then the development element of the Rs.3m incurred to
date must be capitalised as an intangible asset. Amortisation will not begin until commercial
production commences.
2 Provision
Although the claim was made after the reporting period, IAS 10 considers this to be an
adjusting event after the reporting period. The employment of the individual dates back to
20X2 and so the lawsuit constitutes a current obligation for the payment of damages as a
result of this past event (the employment).
The amount and the timing are not precisely known but the likelihood of payment of damages
by Brooklyn is probable and so a provision should be made for the estimated amount of the
liability, as advised by the lawyer. Disclosure, rather than provision, would only be appropriate
if the expected settlement was possible or remote, and the lawyer’s view is that a payment is
more likely than not.
It is not appropriate to calculate an expected value where there is only one event, instead a
provision should be made for the most likely outcome. The lawyer has various views on the
possible payout, but the most likely payout is Rs.500,000 as this has a 50% probability. As
settlement of the provision is not anticipated until 2018, the provision should be discounted
back at 8% to give a liability of Rs.476,280.

© Emile Woolf International 177 The Institute of Chartered Accountants of Pakistan


Financial accounting and reporting II

Provided that the payment from the insurance company is virtually certain, this should be
shown as an asset, also at its discounted value of Rs.47,628, being 10% of the provision.
In both cases the discounting should be unwound over the coming three years through profit
or loss.
3 Revaluation
IAS 16 on Property, Plant and Equipment does not impose a frequency for updating
revaluations. It simply requires a revaluation where it is believed that the fair value of the
asset has materially changed. Hence, if in the past there have been material differences
between the carrying amount and fair value at the 5 yearly review then Brooklyn should
consider having more frequent valuations following on from this year’s valuation.
Revaluations should be regular and not timed simply when property prices are at a peak. It is
not acceptable for Brooklyn to defer its next revaluation while values are low. If property prices
do fall in 2016, then it may be necessary to perform an impairment test in accordance with
IAS 36 Impairment of assets.
If it is believed that an asset value has moved materially, then all assets in that class must be
revalued. Hence it is not sufficient for Brooklyn to just revalue the London property.
IAS 16 does not require the valuation to be performed by an external party, and so the use of
the property manager to conduct the valuations is acceptable. Notes to the financial
statements will disclose that he is not independent of the company.

9.5 ZOUQ INC


(a) (i) The depreciable amount of an intangible asset with a finite useful life shall be allocated
on a systematic basis over its useful life.
(ii) Amortization shall begin when the asset is available for use
(iii) Amortization shall cease at the earlier of the date that the asset is classified as held for
sale and the date that the asset is derecognised.
(iv) The amortization method used shall reflect the pattern in which the asset's future
economic benefits are expected to be consumed by the entity.
(v) The amortization charge for each period shall be recognised in statement of profit or
loss.

(b) Goodwill Account


Rupees Rupees
Goodwill recognised Impairment of
01.01.2014 (W1) 270,000,000 31.12.2014 goodwill 50,000,000
31.12.2014 Balance b/d 220,000,000

270,000,000 270,000,000

01.01.2015 Balance b/d 220,000,000


31.12.2015 Balance b/d 220,000,000

220,000,000 220,000,000

Brand Account
Rupees Rupees
01.01.2014 Brand
recognised 100,000,000 31.12.2014 Amortization 10,000,000

31.12.2014 Balance c/d 90,000,000

100,000,000 100,000,000

© Emile Woolf International 178 The Institute of Chartered Accountants of Pakistan


Answers

01.01.2015 Balance b/d 90,000,000 31.12.2015 Amortization 10,000,000


Impairment of
31.12.2015 Brand 13,500,000
- 31.12.2015 Balance c/d 68,000,000
90,000,000 90,000,000

W1: Value of goodwill


Rupees
Purchase price (50,000,000 x Rs. 30 x 90%) 1,350,000,000
Less: Fair value of net identifiable assets and liabilities
(Rs. 1,100,000,000 x 90%) (990,000,000)
Less: Value of brand (Rs. 100,000,000 x 90%) (90,000,000)
Goodwill recognised 270,000,000

9.6 STAR-BRIGHT PHARMACEUTICAL LIMITED


Star-Bright Pharmaceutical Limited
Statement of financial position
As at December 31, 2015
2014
2015 Restated
Rs. in million
Non-current assets
Intangible asset – brand [Note 8] 274 285
Shareholders’ equity
Retained earnings (W5 and 6) 2,071 1,879
Star-Bright Pharmaceutical Limited
Statement of Financial Position
As at December 31, 2015
8- Intangible assets – Brand
Cost
At beginning of the year (2015: 382+24+54+38,
2014: 382+ 24+54) 498 460
Capitalised during the year 43 38
541 498
Amortization
At beginning of the year (W1 and 2) (213) (163)
*3 *4
During the year (W3 and 4) (54) (50)
(267) (213)
274 285

© Emile Woolf International 179 The Institute of Chartered Accountants of Pakistan


Financial accounting and reporting II

W1 : 382 x 50% + 24 x 30% + 54 x 20% + 38 x 10% = 213


W2 : 382 x 40% + 24 x 20% + 54 x 10% = 163
W3 : 541 x 10% = 54
W4 : 498 x 10% = 50
W5 : 1,950 + 24 + 54 + 38 + 43 – [267 – (382 x 60%)] = 2,071
W6 : 1,785 + 24 + 54 + 38 – [213 – (382 x 50%)] = 1,879

9.7 RAISIN INTERNATIONAL


(a) Following are the criteria that should be used while recognizing intangible assets from
research and development work.
(i) No intangible asset arising from research shall be recognised.
(ii) An intangible arising from development shall be recognised if, and only if , an entity
can demonstrate all of the following:
‰ the technical feasibility of completing the intangible asset so that it will be
available for use or sale.

‰ its intention to complete the intangible asset and use or sell it.
‰ its ability to use or sell the intangible asset.
‰ how the intangible asset will generate probable future economic benefits. Among
other things, the entity can demonstrate the existence of a market for the output
of the intangible asset or the intangible asset itself or, if it is to be used internally,
the usefulness of the intangible asset.

‰ the availability of adequate technical, financial and other resources to complete


the development and to use or sell the intangible asset.
‰ its ability to measure reliably the expenditure attributable to the intangible asset
during its development.
(b) (i) Since the product met all the criteria for the development of the product, it should be
recognised as an intangible in the statement of financial position (SOFP) of the
company. However, RI should capitalise only the development work (i.e. Rs. 9
million) as intangible asset. IAS-38 does not allow capitalization of cost relating to
the research work, training of staff and cost of trial run.
Since the product has a useful life of 7 years, the amortization expense amounting to
Rs. 0.32 million (Rs. 9 million × 3/12 ÷ 7 years) should be recorded in the statement
of profit or loss.
(ii) This purchasing of right to manufacture should be recognised as an intangible in the
SOFP because:
‰ it is for an established product which would generate future economic benefits.

‰ cost of the patent can be measured reliably.


Since there is a finite life, the patent must be amortised over its useful life. The useful
life will be shorter of its actual life (i.e. 10 years) and its legal life (i.e. 5 years. The
amortization to be recorded in SOCI is Rs. 2.83 million (Rs. 17 million × 10/12 ÷ 5).
(iii) The acquired brand should be recognised as an intangible in the SOFP because
acquisition price is a reliable measure of its value. The amortization to be recorded in
SOCI is Rs. 0.12 million (Rs. 2 million ÷ 10 years x 7/12).
(iv) The carrying value of the intangible asset should be increased to Rs. 10 million in the
SOFP. Since there is an indefinite useful life of the intangible assets, it should not be
amortised. Instead, RI should test the intangible asset for impairment by comparing
its recoverable amount with its carrying amount.

© Emile Woolf International 180 The Institute of Chartered Accountants of Pakistan


Answers

CHAPTER 10 – IFRS 16: LEASES

10.1 X Ltd
(a)

A B C D E
Period Opening Fin. Charge Rentals Closing Balance
Balance at 15% of B
(B – (D - C)
Rs.’000 Rs.’000 Rs.’000 Rs.’000
2016 11,420 1,713 4,000 9,133
2017 9,133 1,370 4,000 6,503
2018 6,503 975 4,000 3,478
2019 3,478 522 4,000
 
4,580 16,000
 
(b)
Statement of Financial Position (Extract) as at 31 December 2016

Rs.’000
Non-Current assets (Rs.11,420,000 – Rs.2,855,00) 8,565
Non-Current Liabilities (Obligation under lease) 6,503
Current Liabilities Obligation under lease
(Rs.9,133,000 – Rs.6,503,000) 2,630

11,420,000
Note: Annual Depreciation = = Rs.2,855,000
4

10.2 Progress Ltd


(a) Annuity method

Year 1 Year 2 Year 3


Rs. Rs. Rs.
Cash flow 3,200,000 - -
Outstanding - 1,920,000 1,350,400
Capital repayment 1,280,000 569,600 637,952
Balance 1,920,000 1,350,400 712,448
Interest @ 12% of balance 230,400 162,048 85,494
Capital repayment 569,600 637,952 714,506
800,000 800,000 800,000

© Emile Woolf International 181 The Institute of Chartered Accountants of Pakistan


Financial accounting and reporting II

(b) Journal entries

Dr Cr
Rs. Rs.
2016
Jan. 3 Right of use - Plant and machinery 3,200,000
Fine Rentals Limited 3,200,000
Initial recognition of machine
Jan. 3 Fine Rentals Limited 1,280,000
Bank 1,280,000
Payment of initial deposit under lease
Dec. 31 Fine Rentals Limited 569,600
Interest expense 230,400
Bank 800,000
Apportionment of annual installment
between Principal repayment and
interest
Dec. 31 Profit and Loss Account 230,400
Interest Expense 230,400
Write-off of FL interest expense to
Profit and loss account
2017
Dec. 31 Fine Rentals Ltd 637,952
Interest expense 162,048
Bank 800,000
Apportionment of annual installment
for the year between Principal
repayment and interest
Dec. 31 Profit and Loss Account 162,048
Interest Expense 162,048
Write-off of FL interest expense to
Profit and loss account
2018
Dec. 31 Fine Rentals Limited 714,506
Interest expense 85,494
Bank 800,000
Apportionment of annual installment
for the year between Principal
repayment and interest
Dec. 31 Profit and Loss Account 85,494
Interest Expense 85,494
Write-off of FL interest expense to
Profit and loss account

© Emile Woolf International 182 The Institute of Chartered Accountants of Pakistan


Answers

10.3 MiracleTextileLimited
Miracle Textile Limited
Statement of financial position (extracts) as at 30 June 2016
Note 2016 2015
ASSETS Rs. Rs.
Non-current assets
Right of use - Machinery 4 16,000,000 18,000,000

LIABILITIES
Non-current liabilities
Obligation under lease 9 6,505,219 10,633,074

Current liabilities
Current portion of obligation 9 4,127,856 3,566,925

Miracle Textile Limited


Notes to the financial statements (extracts) for the year ended 30 June 2016
4- Property, plant and equipment 2016 2015
Leased assets
Cost
Opening balance 20,000,000 -
Addition during the year - 20,000,000
20,000,000 20,000,000
Accumulated depreciation
Opening balance (2,000,000) -
Depreciation for the year (2,000,000) (2,000,000)
(4,000,000) (2,000,000)
Balance as at 30 June 16,000,000 18,000,000

9- Obligations under lease (W1)


30-Jun-16 30-Jun-15
Lease Financial Financial
payment charges Present Lease charges Present
for future Value payment for future Value
periods periods

Rs. Rs. Rs. Rs. Rs. Rs.


Not later than
one year 5,800,000 - 5,800,000 5,800,000 - 5,800,000
Later than one
year but not later
than five years 7,800,000 1,294,781 6,505,219 13,600,000 2,966,925 10,633,075
Later than five
years - - - - - -
13,600,000 1,294,781 12,305,219 19,400,000 2,966,925 16,433,075

© Emile Woolf International 183 The Institute of Chartered Accountants of Pakistan


Financial accounting and reporting II

9.1 The Company has entered into a lease agreement with a bank in respect of a machine.
The lease liability bears interest at the rate of 15.725879% per annum. The company
has the option to purchase the machine by paying an amount of Rs.2 million at the end
of the lease term. The lease rentals are payable in annual instalments ending in June
2016. There are no financial restrictions in the lease agreement.

W1: Lease Schedule


Payment Opening Principal Interest @ Closing
Instalment
date principal repayment 15.73% principal
01-Jul-14 20,000,000 5,800,000 5,800,000 - 14,200,000
01-Jul-15 14,200,000 5,800,000 3,566,925 2,233,075 10,633,075
01-Jul-16 10,633,075 5,800,000 4,127,856 1,672,144 6,505,219
01-Jul-17 6,505,219 5,800,000 4,776,997 1,023,003 1,728,222
30-Jun-18 1,728,222 2,000,000 1,728,222 271,778 -
20,000,000 5,200,000

10.4 Acacia Ltd


Relevant extracts
Statements of profit or loss for the year ended 31 March 2016 (extracts)
Rs.
Depreciation (272,850 ÷ 6) 45,475
Lease payments 6,000*
Finance costs (W) 19,460
* Considering low value item as described in IFRS16
Statement of financial position as at 31 March 2016 (extracts)
Rs.
Non-current assets
Right of use(272,850 – 45,475) 227,375
Non-current liabilities
Lease liabilities (Note 1) 135,810
Current liabilities
Lease liabilities (Note 1) 78,250
Statement of cash flows for the year ended 31 March 2016 (extracts)
Cash flows from financing activities
Payment of lease liabilities (78,250)
Notes to the financial statements (extracts)
(1) Analysis of lease liabilities
Gross basis
Rs.
Lease liabilities include the following:
Gross payments due within
One year 78,250
Two to five years (2 × 78,250) 156,500
234,750
Less: Finance charges allocated to future periods
((78,250 × 4) – 272,850 – 19,460) (20,690)
214,060

© Emile Woolf International 184 The Institute of Chartered Accountants of Pakistan


Answers

(Alternatively) Net basis

Rs.
Lease liabilities include the following:
Amounts due within
One year 78,250
Two to five years 135,810
214,060

WORKING:
Lease of plant
Year to 31 Interest @
March B/f Payment Capital 10% C/f
Rs. Rs. Rs. Rs. Rs.
2016 272,850 (78,250) 194,600 19,460 214,060
2017 214,060 (78,250) 135,810

10.5 Shoaib Leasing Limited


(a) Entries in the books of Lessor
Date Particulars Dr. Cr.
1-Jul-16 Lease payments receivable (W1) 2,680,000
Machine 2,100,000
Unearned finance income (W1) 580,000
30-Jun-17 Bank 860,000
Lease payments receivable 860,000
30-Jun-17 Unearned finance income 272,941
Finance income (W2) 272,941
30-Jun-18 Bank 860,000
Lease payments receivable 860,000
30-Jun-18 Unearned finance income 196,640
Finance income (W2) 196,640
30-Jun-19 Bank 960,000
Lease payments receivable 960,000
30-Jun-19 Unearned finance income 110,419
Finance income (W2) 110,419

W1: Total finance income Rs.


Total future lease payments (Rs.860,000 x 3) 2,580,000
Add: Purchase bargain option 100,000
Gross investment in finance lease 2,680,000
Less: Cost of assets 2,100,000
Total finance income 580,000

© Emile Woolf International 185 The Institute of Chartered Accountants of Pakistan


Financial accounting and reporting II

W2: Amortization schedule


Principal Principal
Instalment Interest Principal
Date Opening Closing
Rs.
30-Jun-17 2,100,000 860,000 272,941 587,059 1,512,941
30-Jun-18 1,512,941 860,000 196,640 663,360 849,581
30-Jun-19 849,581 960,000 110,419 849,581 nil
580,000 2,100,000

(b) Shoaib Leasing Limited


Extracts from the statement of financial position as at June 30, 2017
Note 2017
Rs.
Non-current Assets
Net investment in leases 10 849,578
Current Assets
Current portion of net Investment in leases 663,360

10 Net investment in leases


Lease payments receivables 10.1 1,720,000
Add: Residual value of leased assets (part
of LP) 100,000
Gross Investments in leases 1,820,000
Less: Unearned lease income (307,062)
Net investment in leases 10.2 1,512,938
Less: Current portion of net investment in leases (663,360)
849,578

10.1 Lease payments


Less than one year 860,000
More than one year and less than 5 years 960,000
1,820,000

10.2 Net investment in leases


Less than one year 663,360
More than one year and less than 5 years 849,578
1,512,938

© Emile Woolf International 186 The Institute of Chartered Accountants of Pakistan


Answers

10.6 Akbar Ltd.


a) Right-of-use retained by AL
Financing
Since the consideration (Rs.850,000) exceeds the fair value (Rs.550,000) of the machine,
the agreement contains a financing transaction.
AL initially recognises a right-of-use asset as the proportion of the carrying amount that
reflects the right of use retained. The proportion is calculated by dividing the present value
of the lease payment by fair value
=> 440,000 CV ÷ 550,000 FV × 314,457 (W-1) = Rs.251,565
W-1
Fair value of Rs.614,456 less the part of the lease payments that is just a repayment of the
financing granted to the seller-lessee (Rs.300,000) = Rs.314,456

b) Gain / loss on rights transferred


Gain (refer below) = Rs.47,109
Rs.
Consideration received 850,000
Less: Financial liability
Financing 300,000
PV of lease liability 314,456
(A) 235,544
Less: Carrying value of machine transferred
Total carrying value 440,000
Less: Right-of-use asset 251,565
(B) 188,435
Gain on rights transferred (A-B) 47,109

Accounting Entry by Akbar Ltd.


Dr. Cr.
Rs. Rs.
Cash 850,000
Right-of-use asset 251,565
Machine 440,000
Financial liability 614,456
Gain on rights transferred to lessor 47,109

10.7 ALI LIMITED


Since transfer of an asset does not satisfy the requirements of IFRS 15 therefore Ali Ltd. treats
the transaction as a financing arrangement.
The sale proceeds have been incorrectly credited to operating income, and the operating costs
have been incorrectly debited with the lease payment. Both amounts should be reversed.
Therefore, Ali Ltd. is required to adjust its books by passing the following accounting entries:

Dr. (Rs.) Cr. (Rs.)


Operating income 1,440,000
Financial liability 1,440,000

© Emile Woolf International 187 The Institute of Chartered Accountants of Pakistan


Financial accounting and reporting II

Operating expense of Rs.360,000 booked erroneously is rectified by reversing it and debiting:


‰ Interest expense of Rs.115,200 (i-e. Rs.1,440,000 x 8%)
‰ Financial liability of Rs.244,800 (i-e. principal portion)
Rs.360,000
The accounting entry would be:

Dr. Cr.
Rs. Rs.
Interest expense 115,200
Financial liability 244,800
Operating Expense 360,000

The remaining liability of Rs.1,195,200 should be shown as Rs.931,200 non-current and


Rs.264,000 as current.

10.8 Moazzam Textile Mills Limited

Generator A
The ratio between the carrying value (Rs.7,500,000) and fair value (Rs.6,000,000) will determine
the value of right-of-use as against PV of lease payments.

Lease liability
The PV of lease payments is computed by the following formula:
PV = R[1-(1+i)^-n]/i
R = Yearly payment; i = rate per annum; n = number of years
PV = 1,000,000x[1-(1+4.5%)^-5}/4.5%
PV = Rs.4,389,977

Right-of-use
ROU = CV/FV*PV
ROU => 7,500,000/6,000,000*4,389,977 = Rs.5,487,471

Loss on sale
Loss (refer working) = Rs.402,506

Working
Consideration received 6,000,000
Less: PV of lease liability (4,389,977)
Less: Carrying value of machine transferred
Total carrying value 7,500,000
Less: Right-of-use asset (5,487,471) (2,012,529)
Loss on sale = 402,506

Particulars Debit (Rs.) Credit (Rs.)


Rs.
Cash / Bank 6,000,000

© Emile Woolf International 188 The Institute of Chartered Accountants of Pakistan


Answers

Particulars Debit (Rs.) Credit (Rs.)


Right-of-use 5,487,471
Loss on sale 402,506
Generator – Carrying value 7,500,000
Lease Liability 4,389,977

Generator B
Financing transaction
Since the consideration received (Rs.6,000,000) exceeds the fair value (Rs.5,000,000) of the
power generator, the agreement contains a financing transaction.

Sale and lease back


The ratio between the carrying value (Rs.6,000,000) and fair value (Rs.5,000,000) will determine
the value of right-of-use as against PV of lease payments.

Lease liability
The PV of lease payments is computed by the following formula:
PV = R[1-(1+i)^-n]/i
R = Yearly payment; i = rate per annum; n = number of years
PV = 1,000,000x[1-(1+4.5%)^-5}/4.5%
= Rs.4,389,977
Less: Financing = Rs.1,000,000
PV = Rs.3,389,977

Right-of-use
ROU = CV/FV*PV
ROU = 6,000,000/5,000,000*3,389,977
ROU = Rs.4,067,972

Loss on sale
Loss (refer W1) = Rs.322,005

W1
Consideration received 6,000,000
Less:
PV of lease liability (3,389,977)
Financing (1,000,000)
1,610,023
Less: Carrying value of machine transferred
Total carrying value 6,000,000
Less: Right-of-use asset (4,067,972) 1,932,028
Loss = Rs.322,005
Particulars Debit Credit
Rs. Rs.
Cash / Bank 6,000,000
Right-of-use 4,067,972
Loss 322,005
Generator – Carrying value 6,000,000
Lease Liability 4,389,977

© Emile Woolf International 189 The Institute of Chartered Accountants of Pakistan


Financial accounting and reporting II

Generator C
The ratio between the carrying value (Rs.7,000,000) and fair value (Rs.10,000,000) will
determine the value of right-of-use as against PV of lease payments.

Lease liability
The PV of lease payments is computed by the following formula:
PV = R[1-(1+i)^-n]/i
R = Yearly payment; i = rate per annum; n = number of years
PV = 1,500,000x[1-(1+4.5%)^-5}/4.5%
PV = Rs.6,584,965

Right-of-use
ROU = CV/FV*PV
ROU => 7,000,000/10,000,000*6,584,965 = Rs.4,609,475

Gain on sale
Gain (refer W2) = Rs.1,024,510

W2
Consideration received 10,000,000
Less: PV of lease liability 6,584,965
Less: Carrying value of machine transferred
Total carrying value 7,000,000
Less: Right-of-use asset (4,609,475) 2,309,525
Gain = Rs.1,024,510

Particulars Debit Credit


Rs. Rs.
Cash / Bank 10,000,000
Right-of-use 4,609,475
Generator – Carrying value 7,000,000
Lease Liability 6,584,965
Gain on sale 1,024,510

© Emile Woolf International 190 The Institute of Chartered Accountants of Pakistan


Answers

CHAPTER 11 – IAS 37: PROVISIONS CONTINGENT LIABILITIES AND


CONTINGENT ASSETS AND IAS 10: EVENTS AFTER THE REPORTING PERIOD

11.1 BADAR
Decommissioning costs
IAS 37 Provisions, Contingent Liabilities and Contingent Assets only permits a provision to be made
if three conditions are met:
(i) The company has a present obligation, either legally or constructively, as a result of a past
event;
(ii) Probable outflow of resources is required to settle the obligation; and
(iii) A reliable estimate is available.
Although there is no legal requirement to restore the site, the company has established a
constructive obligation by setting a valid expectation in the market, due to its published policies and
past practice, from which it cannot realistically withdraw.
It therefore appears probable that Badar will have to pay money to improve the site and so a
provision should be created for the expected amount. As the expected payment of Rs.100,000 will
not be settled for three years, the provision should be discounted and entered at its net present
value of Rs.75,131 (Rs.100,000/(1.1)3). Over the three years, the discounting should be unwound
and charged to profit or loss as finance costs, resulting in a provision of Rs.100,000 by the end of
the third year.
The cost of the construction work has been correctly capitalised. The cost of the future
decommissioning work should be added to this asset so that the total costs of the site can be
matched to the revenue from the copper over the period of mining. This will result in an asset of
Rs.575,131 which should be depreciated over the three year life in line with anticipated revenues.

11.2 GEORGINA
(1) Litigation for damages
Under IAS37, a provision should only be recognised when:
‰ an entity has a present obligation as a result of a past event
‰ it is probable that an outflow of economic benefits will be required to settle the
obligation
‰ a reliable estimate can be made of the amount of the obligation.
Applying this to the facts given:
‰ Georgina’s legal advisors have confirmed that there is a legal obligation. This arose
from the past event of the sale, on 1 September 2015 (i.e. before the year end).
‰ Probable is defined as ‘more likely than not’. The legal advisors have confirmed that it is
likely that the claim will succeed.
‰ A reliable estimate of Rs.500,000 has been made.
Therefore a provision of Rs.500,000 should be made.
Counter-claim
IAS37 requires that such a reimbursement should only be recognised where receipt is
‘virtually certain’. Since the legal advisors are unsure whether this claim will succeed no asset
should be recognised in respect of this claim.

© Emile Woolf International 191 The Institute of Chartered Accountants of Pakistan


Financial accounting and reporting II

(2) Claim for unfair dismissal


In this case, the legal advisers believe that success is unlikely (i.e. possible rather than
probable). Therefore this claim meets the IAS37 definition of a contingent liability:
‰ a possible obligation
‰ arising from past events
‰ whose existence will be confirmed only by the occurrence or non-occurrence of one
or more uncertain future events.
The liability is a possible one, which will be determined by a future court case or tribunal. It did
arise from past events (the dismissal had taken place by the year end).
This contingent liability should be disclosed in the financial statements (unless the legal
advisors believe that the possibility of success is in fact remote, and then no disclosure is
necessary).

(3) Returns
Applying the IAS37 conditions in (1) to the facts given:
‰ Although there is no legal obligation, a constructive obligation arises from Georgina’s
past actions. Georgina has created an expectation in its customers that such refunds
will be given.
‰ As at the year end, based on past experience, an outflow of economic benefits is
probable.
‰ A reliable estimate can be made. This could be 1% × 400,000 but since the returns are
now all in the actual figure of Rs.3,500 can be used.
Therefore a provision of Rs.3,500 should be made.

(4) Closure of division


Applying the above IAS37 conditions in (1) to the facts given:
‰ A present obligation exists because at the year end there is a detailed plan in place and
the closure has been announced in the press.
‰ An outflow of economic benefits is probable.
‰ A reliable estimate of Rs.300,000 has been made.
However, IAS37 specifically states in respect of restructuring that any provision should include
only direct expenses, not ongoing expenses such as staff relocation or retraining. Therefore a
provision of Rs.250,000 (300,000 – 50,000) should be made.

11.3 EARLEY INC


(a) IAS 10 (revised) Events After the Statement of financial position Date states that assets and
liabilities should be adjusted for events occurring after the statement of financial position date
that provide additional evidence relating to conditions existing at the statement of financial
position date. It specifically includes the example of bad debts, where evidence of bankruptcy
of a debtor occurs after the year end.
In this case, Nedengy appears to have recovered part of the debt and as such only
Rs.200,000 needs to be provided. It may be argued that the receivership has occurred as a
result of events occurring after the statement of financial position date, as a result of a change
in legislation for example, but this is unlikely.
IAS 18 Revenue states that when uncertainty arises about the collectability of an amount
already included in revenue, the amount should be recognised as an expense.
(b) It is likely that the fall in the value of the property will fit the IAS 10 (revised) definition of
adjusting events noted in (a) above, unless, again, it can be argued that the decline in the
property market occurred after the year-end.

© Emile Woolf International 192 The Institute of Chartered Accountants of Pakistan


Answers

IAS 36 Impairment of assets and IAS 16 Property, Plant and Equipment require that the
carrying amount of property, plant and equipment should be reviewed periodically in order to
assess whether the recoverable amount has fallen below the carrying amount. Where it has,
the property, plant and equipment should be written down to the recoverable amount, either
through the statement of profit or loss as an expense, or though other comprehensive income
to revaluation reserve in shareholder’s equity, but only to the extent that the balance on the
revaluation reserve relates to a previous revaluation surplus on the same asset.
(c) IAS 2 Inventories requires that inventories be stated at the lower on cost and net realisable
value. Net realisable value is the estimated selling price in the ordinary course of business
less the estimated costs of completion and the estimated costs necessary to make the sale.
Unless Earley was making a significant margin on the tricycles, it is likely that the reduction in
selling price of 30% will necessitate a write- down to net realisable value, especially
considering the transportation costs to Iraq which must be included. If the Iraqi option is
unlikely to proceed, it may be necessary to write the tricycles down to scrap value.
(d) Under IAS 10, the nationalisation is likely to be regarded as a non-adjusting event that merely
requires disclosure in the financial statements. IAS 27 Consolidated Financial Statements
and Accounting for Investments in Subsidiaries, requires that an investment in a enterprise
should be accounted for as an investment (under IAS 39: Financial Instruments: Recognition
and Measurement) from the date that it ceases to fall within the definition of a subsidiary and
does not become an associate. It seems here that Earley has neither control nor significant
influence, nor even an investment as the assets have been in fact, expropriated. The loss of
the investment should be accounted for in the year in which it occurred, but disclosed in the
current year.
If the loss of the subsidiary results in Earley no longer being a going concern, then the event
becomes an adjusting event.
(e) & Both of the events described are non-adjusting event which should be disclosed, but not
(f) adjusted for in the current year financial statements.

11.4 ACCOUNTING TREATMENTS


(a) IAS 37 Provisions contingent liabilities and contingent assets states that contingent gains
should not be recognised as income in the financial statements. The company has a debit
balance already in its books which indicates that it must be reasonably certain that at least
part of the claim will be paid. This element of the claim then is probably not a contingency at
all. The remaining part (the difference between the Rs.15,000 and the Rs.18,600) is, and
should be disclosed and not accrued.
(b) IAS 16 Property, Plant and Equipment requires that the carrying amount of property, plant
and equipment should be reviewed periodically in order to assess whether the recoverable
amount has fallen below the carrying amount. Where it has, the property, plant and equipment
should be written down to the recoverable amount through the statement of profit or loss as
an expense. In this case this would result in the recognition of an expense of Rs.200,000.
(280,000 – 80,000).
It may be the case that the amounts involved are so significant as to warrant separate
disclosure in the statement of profit or loss under IAS 8 Net Profit of Loss for the Period,
Fundamental Errors and Changes in Accounting Policies.
(c) IAS 37 states that contingent liabilities should not be recognised. Though a provision should
be made for amounts where the company has an obligation to pay them.
The question in this case is whether or there is an obligating event within the context of IAS
37. On balance it seems inappropriate to recognise a provision in respect of this amount but
the possible liability should be disclosed as a contingent liability.
(i) the nature of the contingency
(ii) the uncertainties surrounding the ultimate outcome
(iii) the likely effect, ie Rs.500,000 loss less likely tax relief.

© Emile Woolf International 193 The Institute of Chartered Accountants of Pakistan


Financial accounting and reporting II

(d) IAS 2 Inventories requires that inventories be stated at the lower on cost and net realisable
value. Net realisable value is the estimated selling price in the ordinary course of business
less the estimated costs of completion and the estimated costs necessary to make the sale.
In this case, cost is Rs.1,800 and net realisable value is Rs.1,600
(e) The company should set up a provision for Rs.100,040, ie should accrue for the 10% probable
liability. It should disclose the possible liability under contingent liabilities. The disclosure is as
noted in (c) except that the financial effect is Rs.300,120 (30% u Rs.1,000,400). The balance
should be ignored as it is a remote contingent liability.
Tutorial note
In (c) above it is not appropriate to provide for 20%receivableRs.500,000, ie Rs.100,000. This would
only be appropriate where the event is recurring many times over.
In (e) it is appropriate to use the percentages provided, as warranty work is provided for.

11.5 J-MART LIMITED


(a) Adjusting events:
Adjusting events are events that provide further evidence of conditions that existed at the
reporting date.
Examples of adjusting events include:
(i) The subsequent determination of the purchase price or of the proceeds of sale of non-
current assets purchased or sold before the year end.
(ii) The renegotiation of amounts owing by customers or the insolvency of a customer
(iii) Amounts received or receivable in respect of insurance or the insolvency of a
customer.
(iv) The settlement after the reporting date of a court case that confirms that the entity had
a present obligation at the reporting date.
(v) The receipt of the information after the reporting date indicating that an asset was
impaired at the reporting date.
(vi) The discovery of fraud or errors that show that the financial statements are incorrect.

Non-adjusting events:
Non-adjusting events are indicative of conditions that arose subsequent to the reporting date.
Examples of non-adjusting events might be:
(i) Losses of non-current assets or inventories as a result of a catastrophe such as fire
or flood
(ii) Closing a significant part of the trading activities if this was not begun before the year
end
(iii) The value of an investment falls between the reporting date and the accounts are
authorised
(iv) Announcement of dividend after year end.
(b) (i) The conditions attached to the sale give rise to a constructive obligation on the
reporting date.
A provision for the sales return should be recognised for 5% of June 2015 sales. The
related cost should also be reversed.
(ii) Since the law suit was already in progress at year-end and the amount of
compensation can also be estimated, it is an adjusting event.
A provision of Rs. 400,000 should be made.

© Emile Woolf International 194 The Institute of Chartered Accountants of Pakistan


Answers

(iii) There is no obligating event at the year end either for the costs of fitting the smoke
detectors or for fines under the legislation.
No provision should be recognised in this regard.
(iv) The obligating event is the communication of decision to the customers and
employees, which gives rise to a constructive obligation from that date, because it
creates a valid expectation that the division will be closed.
Since no communication has yet been made, no provision is required in this regard.
(v) The obligating event is the signing of the lease contract, which gives rise to a legal
obligation.
A provision is required for the unavoidable rent payments.
(vi) Since the declaration was announced after year-end, there is no past event and no
obligation at year-end and is therefore non-adjusting event.
Details of the dividend declaration must, however, be disclosed.

11.6 AKBER CHEMICALS LIMITED


(a) The event is an accident, and since it happened before the year end, it is a past event.
However, there is no present obligation since:
(i) there is no law requiring the company to clean the canal.
(ii) there is no constructive obligation to clean the river since:

‰ a public statement has not been made;


‰ there is no established pattern of past practice as this was the first time the
company faced such a situation.
Although the company has decided to clean up the river and even has a reliable estimate of
the costs thereof, no liability or provision should be recognised in the current year because:

‰ the decision was taken after year end; and

‰ the decision was not yet made public.


(b) It is a non-adjustable event because the event due to which the net realizable value (NRV) of
stock has fallen, arose after the reporting date.
However, if this event is material, the company should disclose the decline in NRV in its
financial statement for the year ended June 30, 2015.
(c) The company should make the provision because:
(i) the company has a present obligation because of past event
(ii) the claim of the customer is valid and is confirmed by the company's inspection team
which shows that an outflow will be required to settle the obligation.
(iii) the amount of outflow is reliably estimated i.e. Rs. 2 million.
Since the company is certain of recovery from the vendor, it should:
(i) disclose it as a separate asset.
(ii) recognise a receivable but the same should not exceed the amount of the related
provision i.e. rs. 2.0 million.

© Emile Woolf International 195 The Institute of Chartered Accountants of Pakistan


Financial accounting and reporting II

11.7 QALLAT INDUSTRIES LIMITED


(i) Provision must be made for estimated future claims by customers for goods already sold.
The expected value i.e. Rs. 10 million ([Rs. 150m x 2%] + [Rs. 70m x 10%]) is the best
estimate of the provision.
(ii) Warehouse A: It is an onerous contract. as the warehouse has been sublet at a loss of Rs.
200,000 per month. QIT should therefore create a provision for the onerous contract that
arises on vacating the warehouse. This is calculated as the excess of unavoidable costs of
the contract over the economic benefits to be received from it. Therefore, QIL should
immediately provide for the amount of Rs. 13.2 million. [5.5 years x 12 month x Rs. 200,000]
in its financial statements i.e. for the year ended June 30, 2015.
Warehouse B: It is not an onerous contract because the warehouse has been sublet at
profit. Hence this would require no adjustment.
(iii) A provision is to be made by QIL against a contingent liability as:
(a) There is a present obligation (legal or constructive) as a result of a past event; i.e.
accident occurred on June 15, 2015.
(b) It is probable that outflow of resources will be required to settle the obligation; and
(c) A reliable estimate can be made of the amount of the obligation.
The amount of provision shall be Rs. 2.0 million i.e. the most probable amount as determined
by the lawyer.
(iv) A provision of Rs. 0.4 million is required in relation to penalty for March 1 to June 30, 2015
because at the reporting date there is a present obligation in respect of a past event.
The reimbursement of penalty amount from the vendor shall be recognised when and only
when it is virtually certain that reimbursement will be received if the entity settles the
obligation. The reimbursement should be treated as a separate asset in the statement of
financial position. However, in profit and loss statement, the expense relating to a provision
may be netted off with the amount recognised as recoverable, if any.

11.8 SKYLINE LIMITED


(i) Although the debt owing by the customer existed at the reporting date, the customer’s inability
to pay did not exist at that point. This condition only arose in January 2016 after the fire.
Thus, this is a non-adjusting event. However, if it is material for the financial statements, the
following disclosure should be made.
‰ Nature of the event
‰ An estimate of its financial effect
(ii) The amount withdrawn before year end i.e. Rs. 1.5 million is an adjusting event as although it
was discovered after year end it existed at the year end. However, since 60% has been
recovered subsequently, Rs. 0.6 million would be provided.
The further withdrawal of Rs. 6.0 million is a non-adjusting event as it occurred after year end.
However, if the events are considered material the following disclosures should be made:
‰ Nature of the event
‰ The gross amount of contingency
‰ The amount recovered subsequently
(iii) SL should not recognise the contingent gain until it is realised. However, if recovery of
damages is probable and material to the financial statements, SL should disclose the following
facts in the financial statements:
‰ Brief description of the nature of the contingent asset
‰ An estimate of the financial effect.

© Emile Woolf International 196 The Institute of Chartered Accountants of Pakistan


Answers

(iv) SL should make a provision of the expected amount i.e. Rs. 1.2 million (Rs. 1.0 million x 60%
+ Rs. 1.5 million x 40%) because
‰ it is a present obligation as a result of past event;
‰ it is probable that an outflow of resources embodying economic benefits will be required
to settle the obligations; and
‰ a reliable estimate can be made of the amount.
In addition, SL should disclose the following in the notes to the financial statements:
‰ Brief nature of the contingent liability
‰ The amount of contingency
‰ An indication of the uncertainties relating to the amount or timing of any outflow.

11.9 WALNUT LIMITED


(i) This is an adjusting post reporting event as it provides evidence of conditions that existed at
the end of the reporting period. The reasons for the competitor’s price reduction will not have
arisen overnight, but will normally have occurred over a period of time, may be due to superior
investment in technology.
An inventory write down of Rs. 2.5 million should be recognised and the amount included as
inventory on the Statement of Financial Position reduced to Rs. 12.5 million.
(ii) The provision should be recognised because the obligating event is the communication of
event to the public which creates a valid expectation that the division will be closed.
However, the provision should only be recognised to the extent of redundancy costs. IAS
prohibits the recognition of future operating losses, staff training and profits on sale of assets.
(iii) This is a non-adjusting event because the burglary and theft of consumable stores occurred
after reporting date. However, if the event is material, it should be disclosed in the financial
statements unless the loss is recoverable from the insurance company.
(iv) The drop in value of investment in shares is a non-adjusting event. Since the legislation was
announced after the reporting date, the event is not a past event. However, if the amount is
material, it should be disclosed in the financial statements.
(v) This is an adjusting event as it provides evidence of conditions that existed at the end of the
reporting period. The insolvency of a debtor and the inability to pay usually builds up over a
period of time and it can therefore be assumed that it was facing financial difficulty at year-
end.
A bad debts expense of Rs. 1.5 million should be recognised in SOCI.
(vi) It is a non-adjusting event because the declaration was announced after the year-end and
there was no obligation at year end. Details of the bonus shares declaration must, however,
be disclosed.

11.10 ATTOCK TECHNOLOGIES LIMITED


(i) Since the event which caused the inventory to be sold at a loss occurred after the year
end, it is non-adjusting event. However, the effect of the event should be disclosed in the
financial statements for the year ended June 30, 2015.
(ii) It is an adjusting event in accordance with the requirement of IAS-10. The debtor’s
balance should be written down by 80% amount.
(iii) It is non-adjusting event as the subsequent reduction in price is due to an event,
introduction of competitive product, occurred after the reporting period.
(iv) Since this change was not enacted before the reporting date, it is a non-adjusting event.
However, a disclosure should be made for this change.
(v) Since the declaration was announced after the year-end and there was no obligation at
year-end it is a non-adjusting event. Details of the dividend declaration must, however, be
disclosed.

© Emile Woolf International 197 The Institute of Chartered Accountants of Pakistan


Financial accounting and reporting II

CHAPTER 12 – IAS 8: ACCOUNTING POLICIES, CHANGES IN ACCOUNTING


ESTIMATES AND ERRORS

12.1 WONDER LIMITED


2015 2014
(Restated)
Rs.m Rs.m
Wonder Limited
Extracts of Statement of financial position
For the year ended 30 June 2015
Property, plant and equipment 178.50 111.50
Retained earnings 158.65 95.05
Deferred tax liability 41.85 21.45
PPE: Year 2015: 189 - [20 - (20 × 10% × 1.75)] + [56/4 – 56/7] PPE: Year 2014: 130 - 18.5(Note X)
DTL: Year 2015: [(21.45 + (45 - 27) + {(6+2) × 30%}] DTL: Year 2014: 27 - 5.55 (Note X)

Wonder Limited
Extracts from the Statement of profit or loss for the year ended 30 June 2015
Profit before taxation 98.00 101.50
Taxation (34.40) (36.45)
Profit after taxation 63.60 65.05
PBT : Year 2015 : 90 + (20 × 10% ) + [(56/4) - (56/7)] PBT : Year 2014 : 120 - 18.5 (Note X)
Tax : Year 2015: 32 + [(6+2) × 30%] Tax : Year 2014 : 42 - 5.55 (Note X)

Wonder Limited
Extracts of statement of changes in equity for the year ended 30 June 2015

Retained
earnings
Rs.m
Balance as on 1 July 2013 (108-78) 30.00
Profit for the year ended 30 June 2014 (78 - 12.95 (Note X))- restated 65.05
Balance as at 30 June 2014 - restated 95.05
Profit for the year ended 30 June 2015 63.60
Balance as at 30-June 2015 158.65

Wonder Limited
Notes to the financial statements
For the year ended 31 December 2015
X Correction of error
During the year ended 30 June 2013, the repair works was erroneously debited to
machinery account. The effect of this error is as follows:

© Emile Woolf International 198 The Institute of Chartered Accountants of Pakistan


Answers

2014
Rs.m
Effect on the statement of profit or loss
(Increase) / decrease in expenses or losses
Repairs and maintenance (20.00)
Depreciation (20 × 10% × 9 ÷ 12) 1.50
Tax expenses (30% × (20-1.5)) 5.55
Decrease in profit for the year (12.95)

Effect on the statement of financial position

Increase / (decrease) in assets


Property, plant and equipment (20 – 1.5) (18.50)

(Increase) / decrease in liabilities


Deferred tax liability (Rs. 18.5 × 30%) 5.55

(Increase) / decrease in equity


Retained earnings (18.50 - 5.55) (12.95)

12.2 DUNCAN
Statement of changes in equity (extract)
Retained Retained
earnings earnings
2015 2014
Rs.000 Rs.000
Opening balance as reported 23,950 22,500
Change in accounting policy (W2) 450 400
––––––– –––––––
Re-stated balance 24,400 22,900
Profit after tax for the period (W1) 4,442 3,250
Dividends paid (2,500) (1,750)
––––––– –––––––
Closing balance 26,342 24,400
––––––– –––––––
Workings

(1) Revised profit


2015 2014
Rs.000 Rs.000
Per question 4,712 3,200
Add back: Expenditure for the year 600 500
Minus: Depreciation (870) (450)
–––––– ––––––
Revised profit 4,442 3,250
–––––– ––––––

© Emile Woolf International 199 The Institute of Chartered Accountants of Pakistan


Financial accounting and reporting II

(2) Prior period adjustment


The prior period adjustment is the reinstatement of the Rs.400,000 asset on 1 January 2014
and the Rs.450,000 asset at 1 January 2015. On 31 December 2015 the closing balance
above of Rs.26,342,000 can be reconciled as the original Rs.26,162,000 plus the
reinstatement of the remaining asset of Rs.180,000.

12.3 MOHANI MANUFACTURING LIMITED


Mohani Manufacturing (Private) Limited
Statement of changes in equity
For the year ended December 31, 2015
Retained
Earnings
Rs. in million
Balance at December 31, 2013 as previously reported (Rs. 89m – Rs. 21m) 68.00
Effect of change in accounting policy (Rs. 37m - Rs. 35.5m) (1.50)
Balance at December 31, 2013 – restated 66.50
Profit for the year ended December 31, 2014 - restated (W1) 39.70
Balance at December 31, 2014 – restated 106.20
Profit for the year ended December 31, 2015 (W2) 8.80
Balance at December 31, 2015 115.00

W1: Profit for the year ended December 31, 2014 (as restated) Rs. in million
Profit as previously reported 21.00
Incorrect recording of depreciation (Rs. 25 million – Rs. 10 million) 15.00

Reversal of FIFO method


Opening inventory 37.00
Closing inventory (42.30)
(5.30)
Application of weighted average method
Opening inventory (35.50)
Closing inventory 44.50
9.00
39.70
W2: Adjusted profit for year ended June 30, 2015
Profit as per draft financial statements 15.00
Adjustment in Opening Inventory
FIFO 42.30
Weighted average (44.50)
(2.20)
Adjustment in Closing Inventory
FIFO (58.40)
Weighted average 54.40
(4.00)
Adjusted profit 8.80

© Emile Woolf International 200 The Institute of Chartered Accountants of Pakistan


Answers

CHAPTER 13 – IAS 12: INCOME TAXES

13.1 FRANCESCA
Rs. Rs.
Opening liability 1,340,600
Capital allowances during the year 50,000,000
Depreciation charged during the year (45,000,000)
–––––––––––
5,000,000 u 30% 1,500,000
–––––––––––
Interest receivable in statement of profit or loss 50,000
Interest received in tax computation (45,000)
–––––––––––
Receivable in statement of financial position
5,000 u 30% 1,500
–––––––––––
Interest payable in statement of profit or loss 32,000
Interest paid in tax computation (28,000)
–––––––––––
Payable in balance sheet 4,000 u 30% (1,200)
–––––––––––

Development costs as allowable expense 500,600 u 30% 150,180

Revaluation 6,000,000
Carrying value (4,900,500)
–––––––––––
Revaluation surplus 1,099,500 x 30% 329,850
––––––––––– ––––––––––
Closing liability 3,320,930
––––––––––

Rs.
Charged to the revaluation reserve 329,850
Charged in the statement of profit or loss (balancing figure) 1,650,480
––––––––––
Total movement on the provision of (3,320,930 – 1,340,600) 1,980,330
––––––––––

13.2 SHEP (I)


(a) Corporate income tax liability - year ended 31st December 2015
Rs.
Profit per accounts 121,000
Add Depreciation 11,000
————
133,000
Less tax depreciation (15,000)
————
Taxable profits 117,000
————
Tax payable @ 30% 35,100
————

© Emile Woolf International 201 The Institute of Chartered Accountants of Pakistan


Financial accounting and reporting II

(b) Deferred tax liability Rs.


Carrying amount (48,000 + 12,000 = 60,000 – 11,000) 49,000
Tax base (48,000 + 12,000 = 60,000 – 15,000) 45,000
———
Temporary difference (4,000)
———

Deferred tax liability required @ 30% (1,200)


———
(c) Movement on the deferred tax liability
Balance b/f 
Statement of profit or loss (balancing figure) 1,200
———
Balance c/f 1,200
———
(d) Statement of profit or loss note
Current tax expense 35,100
Deferred tax expense 1,200
————
Tax expense 36,300
————

13.3 SHEP (II)


(a) Corporate income tax liability - year ended 31st December 2016
Rs.
Profit per accounts 125,000
Add Depreciation 14,000
Interest payable 500
Provision 1,200
Fine 6,000
————
146,700
Less tax allowance (given) (16,000)
Interest receivable (150)
————
Taxable profits 130,550
————
Tax payable @ 30% 39,165
————

(b) Deferred tax liability


Carrying Tax Temporary
amount base difference
Rs. Rs. Rs.
Tangible assets
Carrying amount (49bf – 14) 35,000
Tax base (45bf – 16) 29,000 6,000
Interest payable (25,000 x 8% x 3/12) (500)  (500)
Interest receivable (4,000 x 15% x 3/12) 150  150
Provision (1,200)  (1,200)
——— ——— ———
33,450 29,000 4,450
——— ——— ———

Deferred tax @30% 1,335


———

© Emile Woolf International 202 The Institute of Chartered Accountants of Pakistan


Answers

(c) Movement on the deferred tax liability Rs.


Balance b/f 1,200
Statement of profit or loss (balancing figure) 135
———
Balance c/f 1,335
———
(d) Statement of profit or loss note
Current tax expense 39,165
Deferred tax expense 135
————
Tax expense 39,300
————
(e) Tax reconciliation
Accounting profit 125,000
————
Accounting profit @ 30% 37,500
Tax effect of the fine (6,000 @ 30%) 1,800
————
Tax expense 39,300
————

13.4 SHEP (III)


(a) Corporate income tax liability - year ended 31st December 2017
Rs.

Profit per accounts 175,000


Add Depreciation 18,500
Interest payable (note) 
Provision 2,000
Entertainment 20,000
————
215,500
Less tax allowance (given) (24,700)
Interest receivable (note) 
Development costs (17,800)
Provision (500)
————
Taxable profits 172,500
————

Tax payable @ 30% 51,750


————
Note
There is no adjustment to profit for the interest paid and the interest receivable.
Consider the interest payable. The tax authority will disallow the closing accrual but will allow last
year’s accrual (that has been paid in this year) as a deduction. These amounts are equal so there is
no net effect.
Similar comments can be made about the interest receivable.

© Emile Woolf International 203 The Institute of Chartered Accountants of Pakistan


Financial accounting and reporting II

(b) Deferred tax liability


Carrying Tax Temporary
amount base difference
Rs. Rs. Rs.
Tangible assets
Carrying amount (35bf – 18.5) 16,500
Tax base (29bf – 24.7) 4,300 12,200
Interest payable (500)  (500)
Interest receivable 150  150
Provision (2,700)  (2,700)
Development expenditure 17,800  17,800
——— —— ———
31,250 4,300 26,950
——— —— ———
Deferred tax @ 30% 8,085
———
(c) Movement on the deferred tax liability Rs.
Balance b/f 1,335
Statement of profit or loss (balancing figure) 6,750
———
Balance c/f 8,085
———
(d) Statement of profit or loss note
Current tax expense 51,750
Deferred tax expense 6,750
————
Tax expense 58,500
————
(e) Tax reconciliation
Accounting profit 175,000
————
Accounting profit @ 30% 52,500
Tax effect of the fine (20,000 @ 30%) 6,000
————
Tax expense 58,500
————

13.5 SHEP (IV)


(a) Corporate income tax liability - year ended 31st December 2017
Rs.
Taxable profits (as before) 172,500
————
Tax payable @ 34% 58,650
————

(b) Deferred tax liability


Temporary difference (as before) 26,950
———
Deferred tax @ 34% 9,163
———

© Emile Woolf International 204 The Institute of Chartered Accountants of Pakistan


Answers

(c) Movement on the deferred tax liability Rs.


Balance b/f 1,335
Adjustment due to change in rate 178
———
Opening balance restated to 34% (1,335 x 34/30) 1,513
Statement of profit or loss (balancing figure) 7,650
———
Balance c/f 9,163
———

(d) Statement of profit or loss note


Current tax expense 58,650
Deferred tax expense relating to origination and reversal
of temporary differences 7,650
Deferred tax expense resulting from increase in tax rate 178
————
Tax expense 66,478
————
(e) Tax reconciliation
Accounting profit 175,000
————
Accounting profit @ 34% 59,500
Tax effect of the fine (20,000 @ 34%) 6,800
Increase in opening deferred tax balances due to change in rate 178
————
Tax expense 66,478
————

13.6 WAQAR LIMITED


a) Computation of current period income tax liability
2015 2014
Rs.m Rs.m
Accounting profit before tax 40.00 30.00

Less: Admissible deductions


Capital Gain (10.00) (8.00)
Tax depreciation on furniture and fittings
Rs. 40.5 x 10% (4.05)
Rs. 40.5 (1-10%) x 10% (3.65)
Tax depreciation on Machinery
Rs. 90 x 10% (9.00)
Rs. 90 (1-10%) x 10% (8.10)

Add: Inadmissible deductions


Accounting depreciation on machinery 25.00 25.00
Accounting depreciation on furniture and fittings 5.00 5.00
Taxable profit 48.25 38.95
Tax rate 30% 35%
Tax payable (current tax) 14.48 13.63

© Emile Woolf International 205 The Institute of Chartered Accountants of Pakistan


Financial accounting and reporting II

b) Deferred taxation computation


Tax base Temporary Deferred tax
NBV (W1)
(W1) difference liability
Working 2 Rs.m Rs.m Rs.m Rs.m
At December 31,2013
Machinery 175.00 90.00 85.00 29.75
Furniture and fittings 40.00 40.50 (0.50) (0.18)
Deferred tax liability at December
31,2013 (35%) 29.57

At December 31, 2014


Machinery 150.00 81.00 69.00 24.15
Furniture and fittings 35.00 36.45 (1.45) (0.51)
Deferred tax liability at December
31,2014 (35%) 23.64

WDV as at December 31, 2015


Machinery 125.00 72.90 52.10 15.63
Furniture and fittings 30.00 32.80 (2.80) (0.84)
Deferred tax liability at December
31,2015 (35%) 14.79
Working 1
Carrying amount and tax base of machinery NBV Tax base
Cost b/f 200.0 200.0
Accumulated depreciation b/f (25.0)
At 31 December 2013 175.0 90.0
200
Accounting depreciation ( /8 years) (25.0)
Tax depreciation (10% of WDV) (9.0)
At 31 December 2014 150.0 81.0
200
Accounting depreciation ( /8 years) (25.0)
Tax depreciation (10% of WDV) (8.1)
At 31 December 2015 125.0 72.9
Carrying amount and tax base of furniture and fittings NBV Tax base
Cost b/f 50.0 50.0
Accumulated depreciation b/f (10.0)
At 31 December 2013 40.0 40.5
Accounting depreciation (10% u 50) (5.0)
Tax depreciation (10% of WDV) (4.05)
At 31 December 2014 35.0 36.45
Accounting depreciation (10% u 50) (5.0)
Tax depreciation (10% of WDV) (3.65)
At 31 December 2015 30.0 32.8

© Emile Woolf International 206 The Institute of Chartered Accountants of Pakistan


Answers

c) Movement on deferred taxation account (W2) 2015 2014


At January 1 23.64 29.57
Change due to change in rate (23.64 u 5/35) (3.38) -
20.26
Change due to origination and reversal of temporary differences in
the period (balancing figure) (5.47) (5.93)
At December 31 14.79 23.64

d) Tax expense
Current tax 14.48 13.63
Deferred tax: -
Due to origination and reversal of temporary differences in the
period (3.38)
Due to change in rate (5.47) (5.93)
Tax expense 5.63 7.7

e) Tax reconciliation
Accounting profit 40.0 30.0
Tax rate 30% 35%
12.0 10.5
Tax effect of untaxed gain:
30% u 10.0 (3.0)
35% u 8.0 (2.8)
Decrease in opening deferred tax balances due to change in rate
(with rounding adjustment) (3.37)
Tax expense 5.63 7.7

13.7 SHAKIR INDUSTRIES


COMPUTATION OF TAX EXPENSE
FOR THE YEAR ENDED DECEMBER 31, 2015
2015
Rs. in
million
Profit before tax 15.80
Add: Inadmissible expenses
Accounting depreciation (Rs. 1.1 million + Rs. 0.7 million) 1.80
Financial charges on finance lease 0.15
Penalty paid to SECP 0.70
Provision for gratuity 2.40
5.05

© Emile Woolf International 207 The Institute of Chartered Accountants of Pakistan


Financial accounting and reporting II

Less: Admissible expenses Rs.m


Tax depreciation 1.65
Lease payments 0.65
Payment of gratuity 1.60
Borrowing cost capitalised 2.30
6.20
Taxable profit for the year 14.65
Current tax expense @ 35% 5.13

COMPUTATION OF DEFERRED TAX EXPENSE


FOR THE YEAR ENDED DECEMBER 31, 2015
Carrying Tax Temp
amount base difference
Rs.m Rs.m Rs.m
Fixed assets – Owned 16.70 13.85 2.85
Fixed assets – Leased 1.80 - 1.80
Capital work in progress 2.30 - 2.30
Provision for gratuity (0.7 + 2.4 – 1.6) (1.50) - (1.50)
Obligation against assets subject to finance lease (1.20) - (1.20)
Total 4.25
Deferred tax expense @ 35% 1.49

Rs. in
million
Deferred tax liability (Opening) 0.55
Deferred tax expense for the year (balancing figure) 0.94
Deferred tax liability as at December 31, 2015 (Rs. 4.25 million x 35%) 1.49

13.8 MARS LIMITED


(a) Date Particulars Debit Credit
Rupees

01.07.2014 Motor Vehicle - Cost 1,600,000


Obligations under finance lease 1,600,000
Capitalisation of the lease

01.07.2014 Obligations under finance lease 480,000


Bank 480,000
First lease payment made in advance

© Emile Woolf International 208 The Institute of Chartered Accountants of Pakistan


Answers

(a) Date Particulars Debit Credit


Rupees

30.06.2015 Finance charges 153,451


Accrued finance charges 153,451

Finance charge accrual for the year ended June 30, 2015
Working: (Rs. 1,600,000  480,000) u 13.701% = Rs. 153,451)

30.06.2015 Depreciation 400,000


Accumulated depreciation - Motor
Vehicle 400,000

Depreciation charge for the year ended June 30, 2015


Working: Rs. 1,600,000 ÷ 4 = Rs. 400,000.
Assuming that there is no reasonable certainty about transfer of ownership
at the end of lease term.

30.06.2015 Tax expense (W1) 1,492,035


Tax payable 1,492,035
Recognition of tax expense for the year ended June 30, 2015)

30.06.2015 Tax expense 22,035


Deferred tax (W2) 22,035
Recognition of deferred tax asset.

W1 Tax computation
Rs.
Accounting profit before tax 4,900,000
Add: Depreciation on leased assets 400,000
Add: Finance charges 153,451
Less: Lease payment (480,000)
Taxable profit 4,973,451
Tax @ 30% 1,492,035

W2 Deferred tax computation


Carrying Tax
Difference
amount base
Taxable temporary difference
Leased assets 1,200,000 - 1,200,000
Deductible temporary difference
Obligations under finance lease (1,120,000) - (1,120,000)
Accrued finance charges (153,451) (153,451)
Net taxable temporary difference (73,451)
Deferred tax @ 30% (Asset) 22,035

© Emile Woolf International 209 The Institute of Chartered Accountants of Pakistan


Financial accounting and reporting II

(b) Liabilities against assets subject to finance lease (W3)


2015
Rs.
Present value of minimum lease payments 1,120,000
Less: Current maturity shown under current liabilities (326,549)
793,451
Minimum lease payments (W3)
Not later than 1 year 480,000
Later than 1 year and not later than 5 years (480,000 × 2) 960,000
1,440,000
Less: future finance charges on finance lease (320,000)
1,120,000
Present value of finance lease liabilities (W3)
Not later than 1 year 326,549
Later than 1 year and not later than 5 years (371,289 + 422,162) 793,451
1,120,000

The minimum lease payment has been discounted at an interest rate of 13.701% to arrive
at their present value. Rentals are paid in annual instalments.

W3: Repayment Schedule


Opening Principal Interest Annual Closing
Years Balance repayment 13.701% payment Balance
Rs.m Rs.m Rs.m Rs.m Rs.m
2015 1,600,000 480,000 480,000 1,120,000
2016 1,120,000 326,549 153,451 480,000 793,451
2017 793,451 371,289 108,711 480,000 422,162
2018 422,162 422,162 57,838 480,000 -
320,000

13.9 BILAL ENGINEERING LIMITED


(a) Computation of current taxation
Rs.m Rs.m
Profit before tax 50.000
Add: Accounting depreciation 10.000
Financial charges on lease liability (1.00 – 0.3) × 13.701% 0.096
Amortization of research and development cost for the year 1.000
Less: Tax depreciation (7.000)
Annual instalment of lease payment (0.300)

Amortization of research and development cost (15 ×


0.9/10) (1.350)
Current year taxable income 52.446

© Emile Woolf International 210 The Institute of Chartered Accountants of Pakistan


Answers

Rs.m Rs.m
Tax liability for the year (52.446 × 35%) 18.356
Tax liability for prior periods (0.100 × 35%) 0.035
18.391

Deferred taxation
Accounting depreciation 10.000
Tax depreciation (7.000) 3.000

Financial charges on finance lease liability(1.00 – 0.3) × 13.701% 0.096


Annual instalment of lease payment allowed under tax (0.300) (0.204)

Amortization charged in accounts 1.000


Amortization cost claimed in tax (1.350) (0.350)

Excess of taxable income over accounting profit due to time


differences 2.446
Deferred tax credit at 35% (0.856)
Total tax expenses (current and deferred) 17.535

(b) Bilal Engineering Limited: Notes to the financial statements


for the year ended December 31, 2015

1.1 Relationship between tax expense and accounting profit 2015


Rs.m
Accounting profit before tax 50.000
Tax on accounting profit at 35% 17.500
Tax on expenses disallowed (Permanent Difference) 0.035
Effective tax rate/tax charge 17.535

(c) Journal entries Debit Credit


Rs.m Rs.m
1 Income tax expenses 18.391
Provision for taxation 18.391
(Tax provision for 2015)
2 Deferred tax asset 0.856
Tax expenses – deferred 0.856
(Deferred tax credit for 2015)

© Emile Woolf International 211 The Institute of Chartered Accountants of Pakistan


Financial accounting and reporting II

13.10 GALAXY INTERNATIONAL


28 : TAXATION 2015 2014
Rs.m Rs.m
Current - for the year (W – 1) 0.84 -
Deferred (W – 2) 6.95 (0.96)
7.79 (0.96)

28.1 : Relationship between tax expense and accounting profit


Profit/(Loss) before taxation 23.50 (1.75)

Tax at the applicable rate of 35% 8.23 (0.61)


Tax effect of exempt income (1.25 x 35%) (0.44) (0.35)
7.79 (0.96)

W1 : Computation of Current Tax


(Loss) / profit before tax as per books 23.50 (1.75)
Add: Allowable income / Disallowed expenses
Accounting depreciation 15.00 15.00
Provision for gratuity 2.20 1.70
Accrued expenses - 2.00

Less: Disallowed income / Allowable expenses


Tax depreciation (6.00) (45.00)
Interest income from SIBs (Exempt) (1.25) (1.00)
Accrued expenses (2.00)
Taxable income / (loss) 31.45 (29.05)
Tax liability (@ 35% 11.01 -
Tax loss to be brought forward (29.05 x 35%) (10.17) -
Tax payable 0.84 -

W -2: Computation of Deferred Tax


Timing differences (cumulative) on account of:
Depreciation (2015: 30-51, 2014: 15-45) 21.00 30.00
Accrued expenses - (2.00)
Provision for gratuity (3.90) (1.70 )
Tax losses - (29.05)
17.10 (2.75)

Deferred tax @ 35% 5.99 (0.96)


Add: Opening deferred tax (dr.) 0.96 -
Charge/(Reversal) for the year 6.95 (0.96)

© Emile Woolf International 212 The Institute of Chartered Accountants of Pakistan


Answers

13.11 APRICOT LIMITED


Taxation 2015 2014
Rs.m Rs.m
Current (W1) 20.48 10.76
Deferred (W2) (1.58) (21.35)
18.90 (10.59)

Relationship between tax expense and accounting profit 2015


Profit before taxation 60.00
Tax at the applicable rate of 35% 21.00
Less: Tax effect of exempt income (2.10)
18.90

W1: Computation of Current Tax


Profit before tax as per books 60.00 45.00
Add: Allowable income / Disallowed expenses
Accounting depreciation 10.00 9.00
Tax profit on sale of fixed assets 1.00 -
Bad debt expense 5.00 7.00
Less: Disallowed income / Allowable expenses
Tax depreciation (8.00) (7.00)
Accounting profit on sale of fixed assets (0.50) -
Capital gain (6.00) -
Bad debts written off (3.00) (4.00)
Taxable income 58.50 50.00
Tax liability (@ 35%) 20.48 17.50

W2: Computation of Deferred Tax


Fixed assets (2014: 95-90, 2015: 82.5-80) (W2.1) 0.87 1.75
Provision for bad debts (2014: 12×35%, 2015: 14×35%) [W2.2] (4.90) (4.20)
Closing balance of deferred tax (4.03) (2.45)
Less: Opening balance (2.45) (18.90)
Charge for the year (1.58) (21.35)

W2.1 Movement of Fixed Assets Accounting Tax


Opening balance 95.00 90.00
Disposal during the year (2.50) (2.00)
Depreciation for the year - 2015 (10.00) (8.00)
Closing balance 82.50 80.00

W2.2 Movement of provision for bad debts 2015 2014


Opening balance 12.00 9.00
Provision for the year 5.00 7.00
Write off during the year (3.00) (4.00)
Closing balance 14.00 12.00

© Emile Woolf International 213 The Institute of Chartered Accountants of Pakistan


Financial accounting and reporting II

CHAPTER 14: IAS 33: EARNINGS PER SHARE

14.1 AIRCON LTD


(a) Earnings Per Share
௉௥௢௙௜௧௔௙௧௘௥௧௔௫
=
ே௢Ǥ௢௙௦௛௔௥௘௦௜௦௦௨௘ௗ

1,854
2016 = Rs. = Rs.1.01
1,818
1,584 6 .06
2015 = x = Rs.1.69
900 6 .30
Workings
1. Calculation of theoretical ex-rights price
1 share at Rs.6.30 each 6.30
2 rights issue for every 1 at Rs.5.94 11.88
3 shares for 18.18
ଵ଼Ǥଵ଼
Price per share = = Rs.6.06

2. Weighted average number of shares


଺Ǥଷ଴
1 April – 30 Sept. 2015 = 900m x 6/12 x = 467.8
଺Ǥ଴଺
1 Oct. – 31 March 2016 = 2,700 x 6/12 = 1,350
1,818
(b) Report
To: Mr Hamad

From: Management Accountant

Date: 15 April 2016

Subject: Evaluating the changes in EPS of Aircon Ltd


The key factors which has led to changes in the EPS of Aircon Ltd. are as follows:
Revenue and profitability. Revenue increased by Rs.2,700 million (18%) last year, but the
gross profit and net profit ratios have not increased proportionately.
The gross profit percentage fell from 40% to 37% in 2016, while the net profit percentage
remained constant at 10%.
Factors responsible for the decline might be due to the inability of the entity to maintain good
profit margin coupled with the failure to also maintain good control over operating expenses.
The more funds realised from the rights issue did not lead to any significant increase in return
on capital employed which fell from 43% (2,880/6,606) in 2015 to 25% (3,240/12,780) in 2016.
Capital employed: raising over Rs.5,760 million of new finance was largely used to acquire
intangible assets.
It is hoped that this asset will start generating substantial returns in the near future.
EPS has therefore fallen from Rs.1.69 in 2015 to Rs.1.01 in 2016.
Signed
Management Accountant

© Emile Woolf International 214 The Institute of Chartered Accountants of Pakistan


Answers

APPENDIX TO THE REPORT


The ratios that are relevant to discussion and evaluation of changes in EPS of Aircon Ltd are
those that relate to profitability and return on capital employed.
The effect of the rights issue should also be considered in the discussion in relation to how the
funds raised through the shares were employed.
TABLE OF RATIOS

18,000  15,300
(i) Change in revenue = x 100 = 18% Increase
18,000

2016 2015
11,340 6,120
(ii) Costs of sales/revenue = 63% = 40%
18,000 15,300

6,600 6,120
(iii) Gross profit % = 37% = 40%
18,000 15,300

1,854
(iv) Net profit % = 10%
18,000 = 10%

3,420 3,420
(v) Operating expenses % = 19% = 22%
18,000 15,300

540 576
(vi) Interest payable/sales = 3% = 4%
18,000 15,300

846 720
(vii) Taxation/sales = 5% = 5%
18,000 15,300

3,240 2,880
(viii) Capital employed =25% = 43%
9,180  3,600 3,006  3,600

18,000 15,300
(ix) Assets/turnover = 1.41 = 2.32
12,780 6,606

Relevance of EPS to shareholders


(i) The EPS is used to compute the price earning (P/E) ratio, a major market indicator
to determine how successful a company has been operating.
(ii) The price earning figure is a multiple of the EPS, where the multiple represents the
number of years’ earnings required to recoup the price paid for the share.
(iii) Rising trend in EPS is a more accurate performance indicator than rising trend in
profit after tax. The investor should consider the future economic conditions of an
entity with some other ratios such as dividend cover and ROCE.
(iv) EPS is a measure of performance from the existing and potential investors’
perspective.
(v) EPS show the amount available to each ordinary shareholder thereby indicating the
potential returns on individual investment.
(vi) EPS is used to compare the activities of two entities in the same industry.

© Emile Woolf International 215 The Institute of Chartered Accountants of Pakistan


Financial accounting and reporting II

CHAPTER 15 – IAS 23: BORROING COST

15.1 SPIN INDUSTRIES LIMITED


Rupees
Commitment fee 125,000
Actual borrowing costs of specific loan (W1) 2,050,000
General borrowing costs (W1) 1,175,283
Less: Investment income (W2) (137,500)
Interest costs to be capitalised 3,212,783

W1
Outstanding
Outstanding Rate of Borrowing
Months outstanding month up to
amount (Rs.) interest cost (Rs.)
completion
Specific loan
Utilised till first
repayment 25,000,000 1-Sep-14 31-Jan-15 5 12% 1,250,000
Utilised after the first
repayment 20,000,000 1-Feb-15 31-May-15 4 12% 800,000
2,050,000
General Borrowings (W4)
Utilised after specific
loan exhausted on 2nd
payment to contractor
(W3) 8,125,000 1-Dec-14 31-May-15 6 12.08% 490,750
Principal payment of 12.08%
specific loan 5,000,000 1-Feb-15 31-May-15 4 201,333
rd
3 payment to 12.08%
contractor 12,000,000 1-Feb-15 31-May-15 4 483,200
rd
4 payment to 1-Jun-15 12.08%
contractor 9,000,000 31-May-15 0 -
1,175,283

W2: Investment income


Surplus fund available from 1-Sep-14 to 30-Nov-14 (Rs. 25m – Rs. 0.125m
– Rs. 8m – Rs. 10m) × 8% × 3/12 Rs.137,500

W3: Specific loan utilization


Commitment fee 125,000
Payment for obtaining permit 8,000,000
st
1 payment to contractor 10,000,000
nd
2 payment to contractor (balancing) 6,875,000
25,000,000

© Emile Woolf International 216 The Institute of Chartered Accountants of Pakistan


Answers

2nd payment to contractor (total) 15,000,000


Less: paid out of specific loan (as worked out above) 6,875,000
Paid from general borrowing 8,125,000

W4: Weighted average rate of borrowing


Weighted average Interest
amount of loan (Rs.) (Rs.)
From Bank A 25,000,000 Rs. 25,000,000 × 13% × 9/12 2,437,500
From Bank B 20,000,000 3,000,000
45,000,000 5,437,500
Weighted average rate of borrowing (Rs. 5,437,500 / 45,000,000) 12.08%

15.2 GRANITE CORPORATION


Borrowing costs to be capitalised
Workings 2015 2014
Commitment fee @ 1% - 700,000
Borrowing costs on specific loan 1 6,987,500 3,033,333
Borrowing costs on running finance 3 1,381,625 -
Less: Investment income 2 (2,099,001) (1,381,334)
6,720,124 2,351,999

W1 : Actual borrowing costs on specific loan


Outstanding

Outstanding

(Rs.) @ 13%
outstanding
Suspension

capitalised
Borrowing
cost to be
amount

months
month
(Rs.)

Net

From commencement on to June 30 70,000,000 4 0 3,033,333


Amount to be capitalised as on 30-Jun-2014 3,033,333

From June 30 to first principal repayment 70,000,000 2 0 2 1,516,667


After the 1st principal repayment 65,000,000 6 1 5 3,520,833
After the 2nd principal repayment to completion 60,000,000 3 0 3 1,950,000
Amount to be capitalised as on 30-Jun-2015 6,987,500

W2 : Investment income (All amounts in Rupees)


O/s amount
Available Used to reduce Invested in saving Total
up to
Funds running finance (14%) account @ 8% Income
completion
Amount Income Amount Income
Rs.(70m -
25m - 0.7m) 44,300,000 4 10,000,000 466,667 34,300,000 914,667 1,381,334
Investment income – 2014 1,381,334

© Emile Woolf International 217 The Institute of Chartered Accountants of Pakistan


Financial accounting and reporting II

O/s amount
Available Used to reduce Invested in saving Total
up to
Funds running finance (14%) account @ 8% Income
completion
Amount Income Amount Income
Rs. (70m -
25m - 0.7m) 44,300,000 2 10,000,000 233,333 34,300,000 457,333 690,666
Rs.(44.3 -
5m - 4.55m) 34,750,000 5 10,000,000 583,335 24,750,000 825,000 1,408,335
Investment income – 2015 2,099,001

W3 : Interest on running finance


2015

Borrowing cost to be
capitalised (Rs.) @
Net outstanding
No. of months
outstanding

Suspension

14%
months
Description Amount

2nd payment to contractor (Rs. 65m - 34.75m) 30,250,000 4 1 3 1,058,750


Payment of 2nd instalment
Principal 5,000,000 3 0 3 175,000
Interest (Rs. 65m x 13% x 6/12) 4,225,000 3 0 3 147,875
rd
3 payment to contractor 10,000,000 0 0 0 -
49,475,000 1,381,625

15.3 IMRAN LIMITED


(a) Specific borrowings
Rs.
Borrowing costs incurred:
13% bank loan outstanding for 10 months
(Rs. 32 million x 306/365 x 13%) 3,487,562
11% bank loan outstanding for 5 months
(Rs. 10 million x 153/365 x 11%) 461,096
Borrowing costs 3,948,658
Less: Interest that relates to suspension
13% bank loan: (Rs. 32 million x 61/365 x 13%) 695,233
61
11% bank loan (Rs. 10 million x /365 x 11%) 183,836
(879,068)
3,069,590
Less: Investment income on temporary investment of the borrowings (500,000)
2,569,590
(b) General borrowings
Phase 1 Phase 2 Phase 3
Building cost capitalised 20,000,000 18,000,000 16,000,000
Financed out of rights issue (15,000,000)
Financed from borrowing 5,000,000 18,000,000 16,000,000

© Emile Woolf International 218 The Institute of Chartered Accountants of Pakistan


Answers

Phase 1 Phase 2 Phase 3


Period to the year end
March 1 to December 31 306
April 1 to December 31 275
October 1 to December 31 92
Period of suspension (61) (61) 
Number of days for which borrowing
should be capitalised 245 214 92
Weighted average borrowing rate (W3) u 12.73% u 12.73% u 12.73%
Fraction of the year for which the rate
should be applied to costs incurred u 245/365 u 214/365 u 92/365
Capitalised borrowing 427,240 1,343,451 513,385

Total 2,284,076
Workings
W1: Average borrowings Rs.m
13% bank loan outstanding for 10 months
(Rs. 32 million x 306/365 days) 26,827,397
11% bank loan outstanding for 5 months
(Rs. 10 million x 153/365 days) 4,191,781
Average outstanding for the year 31,019,178
W2: Borrowing costs incurred (or from part a)
13% bank loan outstanding for 10 months
(Rs. 32 million x 306/365 x 13%) 3,487,562
11% bank loan outstanding for 5 months
(Rs. 10 million x 153/365 x 11%) 461,096
Borrowing costs 3,948,658
W3: Weighted average rate
Borrowing costs
/ Average outstanding for the year = 3,948,658 (W2)/31,019,178 (W1) = 12.73%
15.4 QURESHI STEEL LIMITED
Capital work in progress – Factory building Rs.000
Progress invoices received from the contractor (30,000+20,000+10,000+15,000) 75,000.00
(Rain damages paid would be chargeable to profit and loss account/ insurance
claim)

Borrowing costs to be capitalised:


Loan processing charges 500.00
Interest on bank loan W1 1,841.67
Interest on running finance W2 2,730.00
Interest income from surplus loan amount W4 (395.00)
Capital work in progress – June 30, 2015 79,676.67

© Emile Woolf International 219 The Institute of Chartered Accountants of Pakistan


Financial accounting and reporting II

W1: Interest on bank loan:


Rs.000
Interest amount Outstanding
Interest at 13%
From To Months loan amount

01-14-2014 31-05-2015 6 25,000 1625.00


01-06-2015 30-06 -2015 1 20,000 216.67
1,841.67

W2: Interest on running finance


Rs.000
Payments from Interest
Months
Paymentsn at 15%
Payments Invoice outstanding
Description et of Right Bank Running per
date amount up to
deductions issue loan finance annum
30-6-10
(W3)

01-07-14 Advanced
payment 10,000 10,000 10,000 12.00 1,500
st
15-10-14 1
progress
bill 30,000 25,500 15,000 10,500 8.50 1,116
nd
15-01-15 2
progress
bill 20,000 17,000 17,000 - - -
rd
15-04-15 3
progress
bill 10,000 8,500 7,500 1,000 2.50 31
31-05-15 Loan
interest 1,625 1,625 1.00 20
31-05-15 Loan
instalment 5,000 5,000 1.00 63
15,000 *24,500 29,125 2,730

*Loan amount of Rs. 25,000,000 less processing charges of Rs. 500,000


W3: Average rate of interest for running finance facility (9,000/60,000) 15%
W4: Interest income from surplus loan amounts:
Rs.000
Interest income Interest
Surplus loan
income at
amounts
From To Months 8%
01-14-14 15-01-15 1.5 24,500 (245)
16-01-15 15-04-15 3.0 7,500 (150)
(395)

© Emile Woolf International 220 The Institute of Chartered Accountants of Pakistan


Answers

CHAPTER 16 – ETHICAL ISSUES IN FINANCIAL REPORTING

16.1 ETHICAL ISSUES


The range of comments made by Arif raises questions over his ethical behaviour and professional
standards.
A chartered accountant should be unbiased when involved in preparing and reviewing financial
information. A chartered accountant should prepare financial statements fairly, honestly, and in
accordance with relevant professional standards and must not be influenced by considerations of
the impact of reported results.
Arif’s failings
Arif appears to be influenced by the need to achieve a specified level of profit. This is not
appropriate and calls his integrity into question.
In addition Arif’s professional competence seems to be suspect. His comment on not being up to
date on all of the little technicalities in IFRS s suggests that he has not maintained a level of
professional competence appropriate to his professional role.
ICAP members have a responsibility to engage in continuing professional development in order to
ensure that their technical knowledge and professional skills are kept up to date. Arif should seek
continuing professional development activities and improve his knowledge on ethical standards.
Furthermore, it might be expected that as Waheed’s superior he should set an example to Waheed
and guide him in his responsibilities. Clearly this is not happening.
As a member of ICAP Arif should be aware of the ICAP code of ethics. Arif should know of the
danger of self-interest threats and intimidation threats to himself and to others. His attempt to
influence the outcome of a fellow professional by applying such a threat to that individual is very
unprofessional.
Waheed’s ethical issues
Waheed faces a self-interest threat, in that there is the possibility of a bonus provided the earnings
per share figure remains the same as last year. Arif has also suggested that she can influence the
Board’s decision over employing him as a replacement finance director – another self-interest threat
to Waheed. Both of these threats must be ignored.
Arif’s comments imply that his application of professional responsibility is lacking. This may extend
into the way in which the current financial statements have been prepared. Waheed must be very
careful (as always) to carry out the review with all due care.
Waheed should first discuss his recommendations with Arif and remind his of his professional
responsibilities to ensure that the accounting standards are correctly followed. If the financial
statements are found to contain errors or incorrect accounting treatment then they must be
amended. If Arif refuses to amend the draft financial statements if necessary Waheed should
discuss the matter with other board members (including non- executives and the audit committee, if
possible). Further action might include consulting with ICAP.

16.2 SINDH INDUSTRIES LTD


(a) Financial reporting issues
Revenue
IFRS 15 Revenue from contract with customer sets out the rules to be followed in recognising
revenue.
The fact that the customer cannot cancel the contract is not relevant to the recognition of
revenue. Revenue from providing a service is recognised according to the stage of completion
subject to satisfying criteria set out in IFRS 15. In the absence of other information the
revenue in this contract should be recognised over the life of the contract as time progresses.
As the contract was only signed just before the year end, none of the revenue can be
recognised in 2015.

© Emile Woolf International 221 The Institute of Chartered Accountants of Pakistan


Financial accounting and reporting II

The credit for the amount received should be recognised as a liability. This represents the
obligation that the company has to provide the service over the next two years.
The fact that the customer cannot cancel the contract is not relevant to the recognition of
revenue. If Sindh Industries failed to provide the service they would be sued for restitution.
Therefore the revenue can only be recognised as the service is provided.
New factory
Borrowing costs directly attributable to construction of an asset which necessarily takes a
substantial period to get ready for its intended use should be capitalised as part of the cost of
that asset under IAS 23 Borrowing Costs. IAS 23 states that the capitalisation of borrowing
costs should commence when three conditions are all met for the first time: borrowing costs
are being incurred, expenditure is being incurred and activities to prepare the asset are being
undertaken. Although borrowing costs were incurred throughout the year and expenditure was
incurred from 1 February 2015 (the date the land was purchased), construction only started
on 1 June 2015. Therefore this is the date on which capitalisation commences.
Capitalisation ceases when substantially all of the activities required to make the asset ready
for use/sale have been completed, that is on 30 September 2015. (The actual date on which
the factory was brought into use is irrelevant.) Therefore the period of capitalisation should be
four months.
Where construction is financed from general borrowings, the calculation of the amount to be
capitalised should be based on the weighted average cost of borrowings. This is:
(Rs.1,000,000 × 9.75%) + (Rs.1,750,000 × 10%) + (Rs.2,500,000 × 8%)/ (Rs.1,000,000 +
Rs.1,750,000 + Rs.2,500,000) = 9%
Therefore the amount capitalised should be 9% × Rs.4.5 million (land Rs.1.8 million plus
construction costs Rs.2.7 million) × 4/12 = Rs.135,000. The total cost of the factory should be
measured at Rs.4,635,000 (Rs.1.8 million plus Rs.2.7 million, plus Rs.135,000). The amount
that has been recognised in the statement of financial position should be reduced by
Rs.315,000 (Rs.450,000 – Rs.135,000). Finance costs recognised in profit or loss should be
increased by Rs.315,000.
Land should not be depreciated because it has an indefinite life. Under IAS 16 Property, Plant
and Equipment depreciation charges should start when the asset becomes available for use,
from 1 October 2015 in this case.
Depreciation of Rs.35,000 ((Rs.2.7 million, plus (Rs.135,000 × 2.7/4.5) ÷ 20) × 3/12) should
be recognised in profit or loss for the year ended 31 December 2015 and the carrying amount
of the asset reduced by the same amount to Rs.4.6 million.
Useful life of the blast furnace
Depreciation of the blast furnace has been based on an estimated useful life of 20 years. This
is at variance with a report by a qualified expert. The asset valuation specialist treats the
furnace as being made up of two components, the main structure and the lining, which must
be replaced at regular five yearly intervals over the life of the asset. This is the approach
required by IAS 16. The uncertainties inherent in business mean that many items in financial
statements cannot be measured with certainty, but estimates should always be made using
the most up to date and reliable information. Where estimates have been prepared by
professionals with relevant qualifications, then it is nearly always most appropriate to use
those estimates. Therefore in accordance with the valuer’s report the main structure of the
furnace should be depreciated over 15 years from 1 January 2015 and the lining should be
depreciated over five years from that date.
The reassessment of the estimated lives of assets is a change in accounting estimate, rather
than a change in accounting policy (IAS 8 Accounting Policies, Changes in Accounting
Estimates and Errors). Changes in accounting estimate should be dealt with on a prospective
basis. This is achieved by including the effect of the change in profit or loss in current and
future periods. The additional depreciation should be calculated as:

© Emile Woolf International 222 The Institute of Chartered Accountants of Pakistan


Answers

Rs.000
Revised depreciation: main structure 140
((Rs.3.5m – Rs.1.4m)/15 years)
lining (Rs.1.4m/5 years) 280

420
Current depreciation (Rs.3.5m/20 years) (175)
Additional depreciation 245

IAS 8 requires the disclosure of the nature and amount of the effect of the change in the
estimate of useful lives on the profit for the year.

(b) Revised financial statements


Statement of profit or loss extract for the year ended 31 December 2015

Borrowing Blast
Draft Revenue costs furnace Revised
Rs.000 Rs.000 Rs.000 Rs.000 Rs.000
Profit before tax 2,500 (1,000) (315)+ (35) (245) 905

Statement of financial position at 31 December 2015

Borrowing Blast
Draft Revenue costs furnace Revised
Rs.000 Rs.000 Rs.000 Rs.000 Rs.000
Non-current assets
Property, plant and equipment 12,000 (315) + (35) (245) 11,405
Current assets 3,500 3,500
Total assets 15,500 14,905

Share capital 2,000 2,000


Retained earnings 6,000 (1,000) (315) + (35) (245) 4,405
Equity 8,000 6,405
Non-current liabilities 5,000 500 5,500
Current liabilities 2,500 500 3,000
Total equity and liabilities 15,500 15,905

(c) Ethical issues


It is noticeable that all the adjustments required reduce profit. This and the background to the
previous finance director’s resignation suggest serious problems.
It is not clear who actually prepared the draft financial statements. If they were prepared by
more junior staff in the absence of a finance director, some of the adjustments (for example,
the calculation of borrowing costs to be capitalised) could be the result of genuine errors or
lack of accounting knowledge. However, it seems reasonably clear that the managing director
has attempted to influence the treatment of the revenue and the estimated useful life of at
least one significant non-current asset. (Note: the directors have reviewed the useful lives of
several items of plant and machinery and it is possible that other assets besides the furnace
are being depreciated over unrealistically long periods.)

© Emile Woolf International 223 The Institute of Chartered Accountants of Pakistan


Financial accounting and reporting II

It seems almost certain that the previous finance director resigned as a result of pressure from
the managing director (and possibly from other members of the Board) to present the financial
statements in a favourable light. The directors intend to seek a stock market listing in the near
future. Therefore they have clear motives for manipulating the profit figure and also (perhaps)
for making controversial decisions before the financial statements come under much greater
scrutiny as a result of the listing. The job title of financial controller is also significant. It
suggests that the role has been downgraded and that the person holding it has less authority
than the rest of the Board.
Possible courses of action:
‰ Discuss with the managing director the financial reporting standards that apply to the
transactions and explain the implications of non-compliance. If the managing director is
himself a member of a professional body then it might be worth pointing out to him that
he himself is bound by an ethical code.
‰ Advise him that as a Chartered Accountant you are bound by the ICAP code of ethics,
and that you would not be prepared to compromise your views of the figures he has
prepared for career advancement.
‰ Consider speaking to the other directors (or audit committee if there is one) and
seeking their support.
‰ If all of these actions produce a negative response then it would be appropriate to
consult the ICAP ethical handbook and/or the Institute.
‰ If all else fails then consider seeking alternative employment.

© Emile Woolf International 224 The Institute of Chartered Accountants of Pakistan

You might also like